User:Gauravkumar4291/Notes/PYQ

From Wikipedia, the free encyclopedia
Source Date Subject Last Reading Reading

Count

Question Ans Reference 1
Insights Oct-20 Polity 08/01/2021 2 1) Consider the following statements regarding Parliamentary privileges.


1. The Constitution provides Parliamentary privileges to the members of both the houses of parliament and not their committees.

2. The Constitution has also extended the parliamentary privileges to those persons who are entitled to speak in the proceedings of a House of Parliament.

3. The parliamentary privileges do not extend to the president of India.

Which of the above statements is/are correct?

a) 1, 2

b) 2, 3

c) 2 only

d) 1, 3

b Parliamentary privileges are special rights, immunities and exemptions enjoyed by the two Houses of Parliament, their committees and their members.

The Constitution has also extended the parliamentary privileges to those persons who are entitled to speak and take part in the proceedings of a House of Parliament or any of its committees. These include the attorney general of India and Union ministers.

The parliamentary privileges do not extend to the president who is also an integral part of the Parliament.

Insights Oct-20 Polity 08/01/2021 2 2) Consider the following statements regarding Committee of Privileges.


1. The functions of this committee are semi-judicial in nature.

2. In the Lok Sabha, the Speaker nominates a committee of privileges.

3. In the Rajya Sabha, the deputy chairperson heads the committee of privileges.


Which of the above statements is/are correct?

a) 1, 2

b) 1, 3

c) 2, 3

d) 1, 2, 3

d In the Lok Sabha, the Speaker nominates a committee of privileges consisting of 15 members as per respective party strengths.

In the Rajya Sabha, the deputy chairperson heads the committee of privileges, that consists of 10 members.

The functions of this committee are semi-judicial in nature.

Insights Oct-20 Polity 08/01/2021 2 3) The Rajya Sabha has special powers in which of the following matters.


1. It can authorise the Parliament to make a law on a subject enumerated in the State List.

2. A resolution for the discontinuance of the national emergency can be passed only by the Rajya Sabha.

3. It can authorise the Parliament to create new All-India Services common to both the Centre and states.


Which of the above statements is/are correct?

a) 1, 2

b) 2, 3

c) 1, 3

d) 1, 2, 3

c Due to its federal character, the Rajya Sabha has been given two exclusive or special powers that are not enjoyed by the Lok Sabha:

1. It can authorise the Parliament to make a law on a subject enumerated in the State List (Article 249).

2. It can authorise the Parliament to create new All-India Services common to both the Centre and states (Article 312).

A resolution for the discontinuance of the national emergency can be passed only by the Lok Sabha and not by the Rajya Sabha.

Insights Oct-20 Polity 08/01/2021 2 4) Consider the following statements


1. Parliament can increase the area of any state without the consent of state legislature.

2. Parliament can alter the name of any state without the consent of state legislature.


Which of the above statements is/are correct?

a) 1 only

b) 2 only

c) Both 1 and 2

d) Neither 1 nor 2

c Article 3 authorises the Parliament to:

(a) form a new state by separation of territory from any state or by uniting two or more states or parts of states or by uniting any territory to a part of any state,

(b) increase the area of any state,

(c) diminish the area of any state,

(d) alter the boundaries of any state, and

(e) alter the name of any state without the consent of the state.

Constitution authorises the Parliament to form new states or alter the areas, boundaries or names of the existing states without their consent.

Insights Oct-20 Polity 08/01/2021 2 5) Which of the following are the federal features of the Indian Constitution?


1. Written Constitution

2. Division of powers

3. All States having an equal number of seats in the Rajya Sabha


Select the correct answer code:

a) 1 only

b) 1, 2

c) 1, 3

d) 2, 3

b The seats are allotted to the states in the Rajya Sabha on the basis of population. Hence, the number of representatives vary from state to state.
Insights Oct-20 Polity 08/01/2021 2 6) Consider the following statements regarding Parliamentary privileges.


1. The press can publish true reports of parliamentary proceedings without prior permission of the House.

2. A member of the parliament cannot be arrested during the session of Parliament if he/she is booked in civil case.

3. Courts can inquire into the validity of any proceedings in Parliament on the ground of an alleged irregularity of procedure.


Which of the above statements is/are correct?


a) 1, 2

b) 2 only

c) 3 only

d) 1, 3

a The 44th Amendment Act of 1978 restored the freedom of the press to publish true reports of parliamentary proceedings without prior permission of the House. But this is not applicable in the case of a secret sitting of the House.

The courts are prohibited to inquire into the proceedings of a House or its committees.

Members cannot be arrested during the session of Parliament and 40 days before the beginning and 40 days after the end of a session. This privilege is available only in civil cases and not in criminal cases or preventive detention cases.

Insights Oct-20 Polity 08/01/2021 2 7) Which of the following factors limit the sovereignty of Indian Parliament?


1. Written Nature of the Constitution

2. Federal System of Government

3. System of Judicial Review

4. Fundamental Rights

5. Fundamental Duties


Select the correct answer code:

a) 1, 2, 3, 5

b) 1, 3, 4, 5

c) 1, 2, 3, 4

d) 1, 2, 3, 4, 5

c The factors that limit the sovereignty of Indian Parliament are:

Written Nature of the Constitution: Parliament has to operate within the limits prescribed by the Constitution.

Federal System of Government: The law-making authority of the Parliament gets confined to the subjects enumerated in the Union List and Concurrent List and does not extend to the subjects enumerated in the State List (except in five abnormal circumstances and that too for a short period).

System of Judicial Review: The adoption of an independent Judiciary with the power of judicial review also restricts the supremacy of our Parliament.

Fundamental Rights: The authority of the Parliament is also restricted by the incorporation of a code of justiciable fundamental rights under Part III of the Constitution.

Fundamental duties does not limit the sovereignty of Indian Parliament.

Insights Oct-20 Polity 08/01/2021 2 8) Consider the following statements.


1. The Constitution contains lists that detail the subjects that each tier of government can make laws on.

2. Indian citizens are governed by laws and rules made by both Central and state governments at the same time.

3. Under federalism, the states derive their authority from the Centre.


Which of the above statements is/are correct?

a) 1, 3

b) 1, 2

c) 2, 3

d) 1, 2, 3

b Constitutional existence of several tiers of government is the first pre-requisite of a federal polity.

Under federalism, the states do not derive their authority from the Centre, but from the Constitution. While the centre may direct the states to perform certain actions, the fact is that they are an independent tier of government.

The Constitution contains lists that detail the issues that each tier of government can make laws on. All persons in India are governed by laws and policies made by each of these levels of government.

Insights Oct-20 Polity 08/01/2021 2 9) The famous Minerva Mills case was related to


a) Evolution of the basic structure doctrine of the Constitution of India

b) Supremacy to Directive Principles of State Policy over the Fundamental Rights

c) Expansion of the Right to Freedom of Speech and Expression

d) Both (a) and (b)

d Minerva Mills Ltd. and Ors. v. Union Of India and Ors. is a landmark decision of the Supreme Court of India that applied and evolved the basic structure doctrine of the Constitution of India.

The 42nd Amendment Act accorded the position of legal primacy and supremacy to the Directive Principles over the Fundamental Rights conferred by Articles 14, 19 and 31. However, this extension was declared as unconstitutional and invalid by the Supreme Court in the Minerva Mills case.

Insights Oct-20 Polity 08/01/2021 2 10) Which of the following is not an essential qualification for appointment as a Governor?


a) He should be a citizen of India

b) He should be a domicile of the State to which he is being appointed

c) He must have completed the age of 35 years

d) He must not be a member of either House of Parliament

b The Constitution lays down only two qualifications for the appointment of a person as a governor. These are:

1. He should be a citizen of India.

2. He should have completed the age of 35 years.

Additionally, two conventions have also developed in this regard over the years.

First, he should be an outsider, that is, he should not belong to the state where he is appointed, so that he is free from the local politics.

Second, while appointing the governor, the president is required to consult the chief minister of the state concerned, So that the smooth functioning of the constitutional machinery in the state is ensured.

However, both the conventions have been violated in some of the cases.

Insights Oct-20 Polity 08/01/2021 2 11) In the Indian Parliamentary model of Government


a) The Centre controls the States and Union territories

b) The Judiciary controls the Executive

c) The Executive is responsible to the Legislature

d) The Legislature controls the Judiciary

c The Constitution of India provides for a parliamentary form of government, both at the Centre and in the states. The parliamentary system of government is the one in which the executive is responsible to the legislature for its policies and acts.
Insights Oct-20 Polity 08/01/2021 2 12) Consider the following statements


1. The Constitution of India is neither rigid nor flexible.

2. Article 368 provides for amendment of the constitution by three ways: by a special majority of the Parliament, by a special majority of the Parliament and with the ratification by half of the total states and by simple majority of the Parliament.

3. Division of powers, written Constitution, supremacy of Constitution, flexibility of Constitution, independent judiciary and bicameralism are the features of a federation.

Which of the above statements is/are incorrect?

a) 2 only

b) 1, 2

c) 2, 3

d) 1, 3

c The Constitution of India is neither rigid nor flexible but a synthesis of both.

Article 368 provides for two types of amendments:

(a) Some provisions can be amended by a special majority of the Parliament, i.e., a two-third majority of the members of each House present and voting, and a majority (that is, more than 50 per cent), of the total membership of each House.

(b) Some other provisions can be amended by a special majority of the Parliament and with the ratification by half of the total states.

At the same time, some provisions of the Constitution can be amended by a simple majority of the Parliament in the manner of ordinary legislative process. Notably, these amendments do not come under Article 368.

The Constitution of India establishes a federal system of government. It contains all the usual features of a federation, viz., two government, division of powers, written Constitution, supremacy of Constitution, rigidity of Constitution, independent judiciary and bicameralism.

Flexibility of Constitution is a unitary or non-federal feature.

Insights Oct-20 Polity 08/01/2021 2 13) Which one of the following Directive Principles was not originally provided in the Constitution of India?


a) Organization of village panchayats

b) Safeguard forests and wild life

c) Uniform civil code for the citizens

d) Organization of agriculture and animal husbandry

b The 42nd Amendment Act of 1976 added four new Directive Principles to the original list. They require the State:

(i) To secure opportunities for healthy development of children (Article 39).

(ii) To promote equal justice and to provide free legal aid to the poor (Article 39 A).

(iii) To take steps to secure the participation of workers in the management of industries (Article 43 A).

(iv) To protect and improve the environment and to safeguard forests and wild life (Article 48 A)

Insights Oct-20 Polity 08/01/2021 2 14) Consider the following statement regarding Fundamental Rights.


1. Fundamental Rights in our Constitution are more elaborate than those found in the Constitution of any other country in the world, including the USA.

2. Fundamental Rights are named so because they are guaranteed and protected by the Parliament, the supreme law-making body in India.

Which of the above statements is/are correct?

a) 1 only

b) 2 only

c) Both 1 and 2

d) Neither 1 nor 2

a Part III of the Constitution is rightly described as the Magna Carta of India. It contains a very long and comprehensive list of ‘justiciable’ Fundamental Rights. In fact, the Fundamental Rights in our Constitution are more elaborate than those found in the Constitution of any other country in the world, including the USA.

The Fundamental Rights are named so because they are guaranteed and protected by the Constitution, which is the fundamental law of the land.

Insights Oct-20 Polity 08/01/2021 2 15) Consider the following statements regarding the constitution of India?


1. Constitution is the supreme law that determines the relationship between the citizens and government.

2. It specifies how the government will be constituted and also lays down limits on the powers of the government.

3. It expresses the aspirations of the people about creating a good society.

Select the correct answer code:

a) 1, 2

b) 2, 3

c) 1, 3

d) 1, 2, 3

d Constitution is the supreme law that determines the relationship among citizens and also the relationship between the citizens and government. Moreover:

• First, it generates a degree of trust and coordination that is necessary for different kind of people to live together;

• Second, it specifies how the government will be constituted, for e.g. by democratic elections.

• Third, it lays down limits on the powers of the government and tells us what the rights of the citizens are; and

• Fourth, it expresses the aspirations of the people, for e.g. by preamble and DPSP.

Insights Oct-20 Geography 08/01/2021 2 1) Consider the following statements regarding Amazon Rainforest.


1. The area covered by Amazon rainforest is larger than Australia.

2. The Andes mountains is located to the east of Amazon rainforest.

3. The entire Amazon rainforest area is located between equator and Tropic of Capricorn.


Which of the above statements is/are correct?

a) 1, 2

b) 1 only

c) 2, 3

d) 3 only

b Amazon rainforest covers approximately eight million square kilometres — an area larger than Australia.

Comprising about 40% of Brazil’s total area, it is bounded by the Guiana Highlands to the north, the Andes Mountains to the west, the Brazilian central plateau to the south, and the Atlantic Ocean to the east.

Insights Oct-20 Geography 08/01/2021 2 2)The Amazon basin is located in which of the following countries?

1. Bolivia

2. Venezuela

3. Suriname

4. French Guiana

5. Paraguay

Select the correct answer code:

a) 1, 2, 3

b) 1, 2, 3, 4

c) 1, 2, 3, 5

d) 1, 2, 3, 4, 5

b The Amazon basin is the part of South America drained by the Amazon River and its tributaries. It is located in the countries of (Amazon -: BBPECCVGF) Brazil, Bolivia, Peru, Ecuador, Colombia, Venezuela, Guyana and French Guiana (France).
Insights Oct-20 Geography 08/01/2021 2 3) Consider the following statements regarding Rainforests.


1. Rainforests are home to more than half of the world’s plant and animal species.

2. Rainforests thrive on every continent on earth.

3. The largest rainforests on Earth surround the Amazon River and the Congo River.

Which of the above statements is/are correct?

a) 1, 2

b) 1, 3

c) 2, 3

d) 1 only

b A rainforest is an area of tall, mostly evergreen trees and a high amount of rainfall.

They are home to more than half of the world’s plant and animal species—even though they cover just 6% of Earth’s surface.

Rainforests thrive on every continent except Antarctica. The largest rainforests on Earth surround the Amazon River in South America and the Congo River in Africa.

The tropical islands of Southeast Asia and parts of Australia support dense rainforest habitats.

Insights Oct-20 Geography 08/01/2021 2 4) Consider the following statements:


1. The soil develops on crystalline igneous rocks in areas of low rainfall.

2. They are generally poor in nitrogen, phosphorous and humus.

3. They are fertile when fine-grained and are poor in fertility when they are coarse-grained.

The above statements refer to:

a) Laterite soil

b) Black Soil

c) Red and Yellow Soil

d) None of the above

c Red soil develops on crystalline igneous rocks in areas of low rainfall in the eastern and southern part of the Deccan Plateau. Along the piedmont zone of the Western Ghats, long stretch of area is occupied by red loamy soil. Yellow and red soils are also found in parts of Odisha and Chhattisgarh and in the southern parts of the middle Ganga plain. The soil develops a reddish colour due to a wide diffusion of iron in crystalline and metamorphic rocks. It looks yellow when it occurs in a hydrated form. The fine-grained red and yellow soils are normally fertile, whereas coarse-grained soils found in dry upland areas are poor in fertility. They are generally poor in nitrogen, phosphorous and humus.
Insights Oct-20 Geography 08/01/2021 2 5) Which of the following best describes the ‘Marine upwelling’?


a) Periodic rise and fall of the ocean due to gravitational attraction of the moon.

b) Rise in the sea level due to the global warming.

c) Rise of deep cold water towards the surface of the ocean.

d) Storm surges hitting the coast due to cyclonic activity.

c Winds blowing across the ocean surface push water away. Water then rises up from beneath the surface to replace the water that was pushed away. This process is known as “upwelling.”

Upwelling occurs in the open ocean and along coastlines. The reverse process, called “downwelling,” also occurs when wind causes surface water to build up along a coastline and the surface water eventually sinks toward the bottom.

Water that rises to the surface as a result of upwelling is typically colder and is rich in nutrients. These nutrients “fertilize” surface waters, meaning that these surface waters often have high biological productivity. Therefore, good fishing grounds typically are found where upwelling is common.

Insights Oct-20 Geography 08/01/2021 2 6) Most Rainforests are structured in different layers. Which of the following factors are responsible for this?


1. Water

2. Sunlight

3. Air circulation

Select the correct answer code:

a) 1, 2

b) 2, 3

c) 1, 3

d) 1, 2, 3

d Most rainforests are structured in four layers: emergent, canopy, understory, and forest floor. Each layer has unique characteristics based on differing levels of water, sunlight, and air circulation. While each layer is distinct, they exist in an interdependent system: processes and species in one layer influence those in another.
Insights Oct-20 Geography 08/01/2021 2 7) Consider the following statements regarding Foreshocks and earthquake swarm.


1. Foreshocks are mild earthquakes that precede larger earthquakes in the same location.

2. An earthquake cannot be identified as a foreshock until after a larger earthquake in the same area occurs.

3. An earthquake swarm is a sequence of seismic events occurring in a local area within a relatively short period of time.

4. Foreshocks and earthquake swarm are mainly clustered in the north-eastern and north-western Himalayan region.


Which of the above statements is/are correct?

a) 1, 2, 3

b) 1, 3, 4

c) 1, 2, 3, 4

d) 2, 3, 4

c The recent earthquakes in different parts of India, albeit of small magnitudes, have occurred due to foreshocks and swarms, a top official of the Geological Survey of India (GSI) said.

Foreshocks -- mild tremor preceding earthquakes -- or swarm activities -- series of earthquakes -- indicate release of tectonic stress and strain during the continuous deformation process, and a detailed study of these jolts could help predict if a significant seismic event is in store. These minor earthquakes are mainly clustered in the north-eastern and north-western Himalayan region, around the Delhi-National Capital Territory (NCT), western Gujarat and Western Maharashtra areas which have been demarcated as seismic zones IV and V.

Insights Oct-20 Geography 08/01/2021 2 8) Most volcanoes and earthquakes in the world are located at


a) The inter-junction of high mountain stretches along the continental belts

b) Within the plates

c) Plate margins

d) Littoral zones of major oceans

c Plate margins witness several plate collisions, sliding, transformation etc that result into volcanism or earthquakes.

Most of them are found in the Ring of Fire.

Some earthquakes also occur within the plates but not as frequently as on the plate margins.

Insights Oct-20 Geography 08/01/2021 2 9) Consider the following statements regarding Earthquakes and Volcano.


1. Earthquake normally occurs along with the volcanoes.

2. Volcanoes occur only where ocean crust collides with continental crust.

3. Volcanoes are one of the best sources of sulphur.

Select the correct answer code:

a) 1, 3

b) 2, 3

c) 1, 2

d) 1, 2, 3

a Earth quakes are usually associated with volcanoes, since both are related to plate movements.

Volcanoes can also happen with ocean-ocean crust collision. Best example is South East Asian Archipelago.

Volcanoes are one of the best sources of sulphur, which is not abundant on other continental landforms.

Insights Oct-20 Geography 08/01/2021 2 10) The thickness of the troposphere is greater at the equator rather than at the poles because of

1. Rotation of the earth pushes the atmosphere near equator to greater heights

2. Convection currents leading to thermal expansion of the atmosphere at the equator

Which of the above statements is/are correct?

a) 1 only

b) 2 only

c) Both 1 and 2

d) Neither 1 nor 2

c The troposphere is thicker at the equator than at the poles because the equator is warmer. The convection currents of air expand the thickness of the troposphere (atmosphere).

Thus the simple reason is thermal expansion of the atmosphere at the equator and thermal contraction near the poles.

Also, the rotation of the earth causes centrifugal force which is strongest near the equator and pushes the atmosphere to greater heights.

The thickness of the troposphere also varies with season. The troposphere is thicker in the summer and thinner in the winter all around the planet.

Insights Oct-20 Geography 08/01/2021 2 11) Consider the following statements.


1. The expansion created by the “Big Bang” continues even to the present day.

2. After the Big Bang, the Universe became highly opaque and temperatures started rising till the atmosphere was formed.

Which of the above statements is/are incorrect?

a) 1 only

b) 2 only

c) Both 1 and 2

d) Neither 1 nor 2

b The Big Bang Theory considers the following stages in the development of the universe.

(i) In the beginning, all matter forming the universe existed in one place in the form of a “tiny ball” (singular atom) with an unimaginably small volume, infinite temperature and infinite density.

(ii) At the Big Bang the “tiny ball” exploded violently. This led to a huge expansion. It is now generally accepted that the event of big bang took place 13.7 billion years before the present. The expansion continues even to the present day. As it grew, some energy was converted into matter. There was particularly rapid expansion within fractions of a second after the bang. Thereafter, the expansion has slowed down. Within first three minutes from the Big Bang event, the first atom began to form.

(iii) Within 300,000 years from the Big Bang, temperature dropped to 4,500K (Kelvin) and gave rise to atomic matter. The universe became transparent.

Insights Oct-20 Geography 08/01/2021 2 12) Consider the following statements regarding Temperate Rainforests.


1. Temperate rainforests are found mostly in coastal, mountainous areas.

2. Cooler temperatures and a more stable climate slow down decomposition, allowing more material to accumulate.

3. Temperate rainforests are the most biologically diverse terrestrial ecosystems in the world.

Which of the above statements is/are correct?

a) 1, 3

b) 2 only

c) 1, 2

d) 2, 3

c Temperate rainforests are located in the mid-latitudes, where temperatures are much more mild than the tropics. Temperate rainforests are found mostly in coastal, mountainous areas. Rainfall in these forests is produced by warm, moist air coming in from the coast and being trapped by nearby mountains.

Cooler temperatures and a more stable climate slow down decomposition, allowing more material to accumulate. This productivity allows many plant species to grow for incredibly long periods of time.

Tropical rainforests are the most biologically diverse terrestrial ecosystems in the world.

Insights Oct-20 Geography 08/01/2021 2 13) Consider the following statements.

1. In India, oilseeds are largely grown under rainfed conditions.

2. Soybean and groundnut contributes to more than 80% of total oilseeds production in India.

3. Madhya Pradesh, Maharashtra and Rajasthan are the major Soybean growing states.

Which of the above statements is/are correct?

a) 1, 2

b) 1, 3

c) 2, 3

d) 1, 2, 3

b Soybean (34%), groundnut (27%), rapeseed & mustard (27%) contributes to more than 88% of total oilseeds production and >80% of vegetable oil with major share of mustard (35%), soybean (23%) and groundnut (25%).

Andhra Pradesh (groundnut) & Gujarat (groundnut), Haryana (Mustard), Karnataka (G.nut), M.P(Soybean), Maharashtra (Soybean), Rajasthan (Mustard & Soybean), Tamil Nadu (G. nut), U.P(Mustard), West Bengal (Mustard) contributing more than 95% of total oilseed production in the country.

Majority of the oilseeds are cultivated under rainfed ecosystem (70%).

Insights Oct-20 Geography 08/01/2021 2 14) Which of the following lake is not part of Great Lakes region of North America?


a) Lake Erie

b) Great Slave Lake

c) Michigan Lake

d) Lake Ontario

b The Great Lakes also called the Laurentian Great Lakes and the Great Lakes of North America, are a series of interconnected freshwater lakes located primarily in the upper mid-east region of North America, on the Canada–United States border, which connect to the Atlantic Ocean through the Saint Lawrence River. They consist of Lakes Superior, Michigan, Huron, Erie, and Ontario. (Great Lakes : HOMES)
Insights Oct-20 Geography 08/01/2021 2 15) Which of the following rivers flows through Kaziranga National Park.

1. Brahmaputra

2. Diphlu

3. Dharla

4. Mora Diphlu

5. Rangpo

Select the correct answer code:

a) 1, 2, 3

b) 1, 3, 4, 5

c) 1, 2, 3, 4, 5

d) 1, 2, 4

d Kaziranga (Mora Di-Mora Dha Brhama k Diphlu) is crisscrossed by four main riversBrahmaputra, Diphlu, Mora Diphlu and Mora Dhansiri and has numerous small water bodies.
Insights Oct-20 Geography 08/01/2021 2 16) Consider the following statements regarding Asafoetida.


1. Asafoetida or heeng is a perennial plant that stores most of its nutrients inside its deep fleshy roots.

2. It thrives in dry and cold desert conditions.

3. In India, it is grown extensively in the Himalayan region, mainly in the Ladakh region.

Which of the above statements is/are correct?

a) 1, 2

b) 1, 3

c) 2, 3

d) 1, 2, 3

a • Asafoetida, or heeng, is a common ingredient in most Indian kitchens.

Heeng is not cultivated in India.

• India imports Rs 600 crore worth of this pungent flavoured herb every year.

• It is a perennial plant. The plant stores most of its nutrients inside its deep fleshy roots.

• Asafoetida is endemic to Iran and Afghanistan, the main global suppliers.

• It thrives in dry and cold desert conditions.

Scientists at CSIR-Institute of Himalayan Bioresource, Palampur (IHBT), are on a mission to grow heeng in the Indian Himalayas. The first sapling has been planted in Himachal Pradesh’s Kwaring village in Lahaul valley

Insights Oct-20 Geography 08/01/2021 2 17) Consider the following statements regarding Asteroids.


1. Asteroids are rocky objects that orbit the Sun.

2. All the Asteroids are found in between Mars and Jupiter.

3. Some of the Near-Earth Asteroids are potentially hazardous.

Which of the above statements is/are correct?

a) 1, 2

b) 1, 3

c) 2, 3

d) 1, 2, 3

b Asteroids are rocky objects that orbit the Sun, much smaller than planets. They are also called minor planets.

Asteroids are divided into three classes. First, those found in the main asteroid belt between Mars and Jupiter.

The second group is that of trojans, which are asteroids that share an orbit with a larger planet. NASA reports the presence of Jupiter, Neptune and Mars trojans. In 2011, they reported an Earth trojan as well.

The third classification is Near-Earth Asteroids (NEA), which have orbits that pass close by the Earth. Those that cross the Earth’s orbit are called Earth-crossers. More than 10,000 such asteroids are known, out of which over 1,400 are classified as potentially hazardous asteroids (PHAs).

Insights Oct-20 Geography 08/01/2021 2 18) Consider the following statements regarding Ring of Fire.

1. The Ring of Fire is a direct result of plate tectonics/ movement and collisions of lithospheric plates.

2. About 90% of the world's earthquakes occur along the Ring of Fire.

3. This region is also prone to high volcanism.

Which of the above statements is/are correct?

a) 1, 2

b) 1, 3

c) 2, 3

d) 1, 2, 3

d The Ring of Fire (also known as the Rim of Fire or the Circum-Pacific belt) is a major area in the basin of the Pacific Ocean where many earthquakes and volcanic eruptions occur.

The Ring of Fire is a direct result of plate tectonics: the movement and collisions of lithospheric plates, especially subduction in the northern portion.

It is associated with a nearly continuous series of oceanic trenches, volcanic arcs, and volcanic belts and plate movements.

It has more than 75% of the world's active and dormant volcanoes.

About 90% of the world's earthquakes and about 81% of the world's largest earthquakes occur along the Ring of Fire.

Insights Oct-20 Geography 08/01/2021 2 19) Which of the following statement best describes ‘Doldrums’?


a) High pressure area in subtropical region where westerlies originate.

b) Belt of calm region in equatorial region where prevailing trade winds meet.

c) Tropical region in Indian ocean where cyclone often originate.

d) Frigid zone of the earth where there is little atmospheric circulation.

b Doldrums, also called equatorial calms, equatorial regions of light ocean currents and winds within the intertropical convergence zone (ITCZ), a belt of converging winds and rising air encircling Earth near the Equator. The northeast and southeast trade winds meet there; this meeting causes air uplift and often produces clusters of convective thunderstorms. They occur along the Equator in the Indian and western Pacific oceans and slightly north of the Equator off the African and Central American west coasts.
Insights Oct-20 Geography 08/01/2021 2 20) Consider the following statements regarding Fold Mountains.


1. Fold Mountains are created when large areas are broken and displaced vertically.

2. They are least likely to have conical peaks.

Which of the above statements is/are correct?

a) 1 only

b) 2 only

c) Both 1 and 2

d) Neither 1 nor 2

d Fold mountains are created where two or more of Earth’s tectonic plates are pushed together. At these colliding, compressing boundaries, rocks and debris are warped and folded into rocky outcrops, hills, mountains, and entire mountain ranges.

The Himalayan Mountains and the Alps are young fold mountains with rugged relief and high conical peaks. The Appalachians in North America and the Ural Mountains in Russia have rounded features and low elevation. They are very old fold mountains.

Block Mountains are created when large areas are broken and displaced vertically.

Insights Oct-20 Economics 08/01/2021 2 1) Consider the following statements regarding White Label ATM (WLA)

1. Automated Teller Machines (ATMs) set up, owned and operated by non-bank entities are called White Label ATMs

2. Non-bank ATM operators are authorised under the Payment & Settlement Systems Act, 2007 by the RBI.

3. White label ATMs accept cash deposits.

Which of the above statements is/are correct?

a) 1, 2

b) 1, 3

c) 1, 2, 3

d) 2 only

c ATMs set up, owned and operated by non-banks are called White label ATMs (WLAs). Non-bank ATM operators are authorised under the Payment & Settlement Systems Act, 2007 by the Reserve Bank of India (RBI).

For a customer, using a WLA is just like using an ATM of any bank.

Until recently, these white label ATMs were not allowed to accept cash deposits, but the latest guidelines have now allowed these ATM operators to provide this service. They can also offer bill payment services through their ATMs.

Insights Oct-20 Economics 08/01/2021 2 2) Consider the following statements regarding the Marginal Standing Facility (MSF).

1. MSF functions as the last resort for banks to borrow short-term funds over and above that available under the Liquidity Adjustment Facility Window (LAF).

2. MSF is an extraordinary rate at which banks can borrow money from the central bank at a much cheaper rate than repo rate.

Which of the above statements is/are incorrect?

a) 1 only

b) 2 only

c) Both 1 and 2

d) Neither 1 nor 2

b Marginal Standing Facility (MSF) was announced by the Reserve Bank of India (RBI) in its Monetary Policy (2011-12) and refers to the penal rate at which banks can borrow money from the central bank over and above what is available to them through the LAF window.

MSF, being a penal rate, is always fixed above the repo rate.

Insights Oct-20 Economics 08/01/2021 2 3) Inflation may result from

1. A reduction in the total productive capacity of the economy even as more and more people are employed

2. Oversupply of goods in the economy

Select the correct answer code:

a) 1 only

b) 2 only

c) Both 1 and 2

d) Neither 1 nor 2

a In simple terms, inflation is basically too much money chasing too few goods, or excess demand chasing limited supply. In both these cases, the prices of goods rises faster as individual consumers bid process higher in order to get the good.

Excess supply is likely to bring prices down, not high.

If income rises faster, demand for goods and services will also rise. On the other hand, if the economy is unable to satisfy the increased demand, for e.g. due to poor infrastructure, lack of production etc, the higher income will spiral the prices upwards and lead to high inflation.

Insights Oct-20 Economics 08/01/2021 2 4) Which of the following best describes ‘Anti-Dumping’ duty?

a) a protectionist tariff imposed on foreign imports that are priced below fair market value.

b) an import tax imposed on certain goods in order to prevent piracy.

c) tax imposed on the import of the goods pertaining to core industrial sectors.

d) None of the statements a, b and c are correct.

a An anti-dumping duty is a protectionist tariff that a domestic government imposes on foreign imports that it believes are priced below fair market value. Dumping is a process where a company exports a product at a price lower than the price it normally charges in its own home market.
Insights Oct-20 Economics 08/01/2021 2 5) When the Reserve Bank of India increases the repo rate by 50 basis points, which of the following likely to happen?

1. India's GDP growth rate increases drastically

2. Foreign Institutional Investors may bring more capital into our country

3. Scheduled Commercial Banks may increase their lending rates

Select the correct answer code:

a) 1, 2

b) 1, 3

c) 2, 3

d) 3 only

d Increase in repo rate by RBI would largely lead to increase in lending rates of banks.
Insights Oct-20 Economics 08/01/2021 2 6) Which of the following were the purpose of Fiscal Responsibility and Budget Management Act, 2003 (FRBMA)?

1. Eliminate revenue deficit of the country

2. Reduce fiscal deficit to 3% of the GDP

3. Improve overall management of the public funds

Select the correct answer code:

a) 1, 2

b) 1, 3

c) 2, 3

d) 1, 2, 3

d The Fiscal Responsibility and Budget Management Act, 2003 (FRBMA) is an Act of the Parliament of India to institutionalize financial discipline, reduce India's fiscal deficit, improve macroeconomic management and the overall management of the public funds by moving towards a balanced budget and strengthen fiscal prudence.

The main purpose was to eliminate revenue deficit of the country (building revenue surplus thereafter) and bring down the fiscal deficit to a manageable 3% of the GDP by March 2008. Since then, there have been several amendments to the Act essentially postponing the targets.

Insights Oct-20 Economics 08/01/2021 2 7) Consider the following statements regarding Long term repo operation (LTRO).

1. Long term repo operation is a tool under which the central bank provides one-year to three-year money to banks at the prevailing repo rate.

2. As per RBI guidelines, banks need not keep government securities as collateral for accessing funds under LTRO.

3. LTRO helped RBI ensure that banks reduce their marginal cost of funds-based lending rate, without reducing policy rates.

Which of the above statements is/are correct?

a) 1, 2

b) 1, 3

c) 2, 3

d) 1 only

b The Reserve Bank of India (RBI) has said it will conduct on-tap targeted long-term repo operations (LTRO) for an amount of Rs 1 lakh crore to ensure comfortable liquidity conditions in the system.

The LTRO is a tool under which the central bank provides one-year to three-year money to banks at the prevailing repo rate, accepting government securities with matching or higher tenure as the collateral.

How is it different from LAF and MSF?

While the RBI’s current windows of liquidity adjustment facility (LAF) and marginal standing facility (MSF) offer banks money for their immediate needs ranging from 1-28 days, the LTRO supplies them with liquidity for their 1- to 3-year needs. LTRO operations are intended to prevent short-term interest rates in the market from drifting a long way away from the policy rate, which is the repo rate.

LTRO helped RBI ensure that banks reduce their marginal cost of funds-based lending rate, without reducing policy rates.

Insights Oct-20 Economics 08/01/2021 2 8) Consider the following statements regarding Current account convertibility.

1. Current account convertibility means freedom to convert domestic currency into foreign currency and vice versa for trade in goods and invisibles.

2. Under current account convertibility for rupee, an exporter can sell the foreign currency he obtained from exporting a commodity at the market determined exchange rate in India.

3. There is partial Current account convertibility in India, so as to limit imports into the country.

Which of the above statements is/are correct?

a) 1, 2

b) 1, 3

c) 2, 3

d) 1, 2, 3

a Current account convertibility means freedom to convert domestic currency into foreign currency and vice versa for trade in goods and invisibles (services, transfers or income from investment). Individuals and entities can convert currencies in the foreign exchange market.

Current account convertibility is one part of currency convertibility.

When there is current account convertibility for rupee, an exporter can sell the US Dollars (or other foreign currency) he obtained from exporting a commodity at the market determined exchange rate in India. This means that there is no exchange controls (foreign exchange controls). Similarly, when an importer buys foreign currency from India’s foreign exchange market by exchanging rupee, it is current account convertibility.

In India, there is full current account convertibility since August 20, 1993.

Insights Oct-20 Economics 08/01/2021 2 9) Consider the following statements.

1. Headline inflation is a measure of inflation within an economy, including commodities which tend to be more volatile and prone to inflationary spikes.

2. Headline inflation present an accurate picture of an economy's inflationary trend since sector-specific inflationary spikes persist.

Which of the above statements is/are incorrect?

a) 1 only

b) 2 only

c) Both 1 and 2

d) Neither 1 nor 2

b Headline inflation is a measure of the total inflation within an economy, including commodities such as food and energy prices (e.g., oil and gas), which tend to be much more volatile and prone to inflationary spikes.

Headline inflation may not present an accurate picture of an economy's inflationary trend since sector-specific inflationary spikes are unlikely to persist.

Insights Oct-20 Economics 08/01/2021 2 10) Which of the following are part of capital receipts for the Government of India.

1. Loans raised by Government from RBI and public

2. Dividend on investments made by Government

3. Disinvestment receipts

4. Borrowings by Government through sale of Treasury Bills

Select the correct answer code:

a) 1, 3

b) 1, 3, 4

c) 2, 4

d) 1, 2, 3, 4

b The capital receipts are loans raised by Government from public, called market loans, borrowings by Government from Reserve Bank and other parties through sale of Treasury Bills, loans received from foreign Governments and bodies, disinvestment receipts and recoveries of loans from State and Union Territory Governments and other parties.

Revenue Budget consists of the revenue receipts of Government (tax revenues and other revenues like interest and dividend on investments made by Government, fees, and other receipts for services rendered by Government) and the expenditure met from these revenues.

Insights Oct-20 Economics 08/01/2021 2 11) Consider the following statements about Socialist economy.

1. It emphasises the collective ownership of the means of production.

2. Its only central goal is to attain economic efficiency by most optimum utilization of factors of production.

Which of the above statements is/are incorrect?

a) 1 only

b) 2 only

c) Both 1 and 2

d) Neither 1 nor 2

b Communist economy advocates state ownership of all properties including even labour and absolute power to state in running the economy.

Socialistic economy, on the other hand, emphasises the collective ownership of the means of production (property and assets). It also ascribes a large role to the state in running the economy.

A socialist economy’s central goal is the welfare of its subjects by progressive and equitable distribution of income, widespread employment, economic growth etc.

Economic efficiency is generally the goal of capitalist economies.

Insights Oct-20 Economics 08/01/2021 2 12) Consider the following statements regarding Universal Banking.

1. Universal banking is a system in which banks provide a wide variety of financial services, including commercial and investment services.

2. Universal Banking was conceptualized in India after the recommendation of SH Khan Committee.

3. They are exempted from the CRR and SLR requirements of the RBI.

Which of the above statements is/are correct?

a) 1, 3

b) 2, 3

c) 1, 2

d) 1, 2, 3

c Universal banking is a system in which banks provide a wide variety of financial services, including commercial and investment services.

Banks in a universal system may still choose to specialize in a subset of banking service, even though they technically offer much more to their client base.

The second Narasimham committee of 1998 gave an introductory remark on the concept of the Universal banking. However, the concept of Universal Banking conceptualized in India after the SH Khan Committee recommended it as a different concept.

Once the Financial Institution becomes a universal Bank, it would be compliant with the CRR and SLR requirements of the RBI.

Insights Oct-20 Economics 08/01/2021 2 13) Consider the following statements regarding Fiscal Responsibility and Budget Management (FRBM) Act, 2003.

1. The Act made Central government responsible for ensuring inter-generational equity in fiscal management and long-term macro-economic stability.

2. The Act envisages the setting of limits on the Central government’s debt and deficits.

3. The law contain an ‘escape clause’ under which Centre can exceed the annual fiscal deficit target.

Which of the above statements is/are correct?

a) 1, 2

b) 1, 3

c) 2, 3

d) 1, 2, 3

d What is the FRBM Act?

Enacted in August 2003, the legislation is aimed at making the Central government responsible for ensuring “inter-generational equity in fiscal management and long-term macro-economic stability”.

To achieve this, the Act envisages the setting of limits on the Central government’s debt and deficits as well as mandating greater transparency in fiscal operations of the Central government and the conduct of fiscal policy in a medium-term framework.

The rules for implementing the Act were notified in July 2004 and since then every Budget of the Union government has included a Medium Term Fiscal Policy Statement that specifies the annual revenue and fiscal deficit goals over a three-year horizon.

The States have also enacted their own respective Financial Responsibility Legislation.

Insights Oct-20 Economics 08/01/2021 2 14) In economics, the monetary base includes:

1. Total currency circulating in the public.

2. Currency that is physically held in the vaults of commercial banks.

3. Sum total of the capital of all financial institutions regulated by Central Bank.

4. Commercial banks' reserves held in the central bank.

Select the correct answer code:

a) 1, 2, 3

b) 1, 2, 4

c) 2, 3, 4

d) 1, 2, 3, 4

b In economics, the monetary base (also base money, high-powered money, reserve money) in a country is the total amount of bank notes and coins. This includes:

• the total currency circulating in the public,

• plus the currency that is physically held in the vaults of commercial banks,

• plus the commercial banks' reserves held in the central bank.

The monetary base should not be confused with the money supply, which consists of the total currency circulating in the public plus certain types of non-bank deposits with commercial banks.

Insights Oct-20 Economics 08/01/2021 2 15) The FRBM Act contain an ‘escape clause’ under which Centre can exceed the annual fiscal deficit target on which of the following grounds?

1. National security

2. National calamity

3. Collapse of agriculture

4. Decline in real output growth of a quarter by at least three percentage points below the average of the previous four quarters.

Select the correct answer code:

a) 1, 2, 3

b) 1, 3, 4

c) 1, 2, 3, 4

d) 2, 3, 4

c How does a relaxation of the FRBM work?

The law does contain what is commonly referred to as an ‘escape clause’. Under Section 4(2) of the Act, the Centre can exceed the annual fiscal deficit target citing grounds that include national security, war, national calamity, collapse of agriculture, structural reforms and decline in real output growth of a quarter by at least three percentage points below the average of the previous four quarters.

There have been several instances of the FRBM goals being reset. Most recently, presenting the Budget for 2020-21 in February, Finance Minister Nirmala Sitharaman had cited the recent reductions in corporate tax as structural reforms that would trigger the escape clause.

But the most significant FRBM deviation happened in 2008-09, in the wake of the global financial crisis. Simultaneously, the deficit goals for the States too were relaxed.

Insights Oct-20 Economics 08/01/2021 2 16) Consider the following statements.

1. In recent years, the credit growth in the banking system is higher than deposit growth.

2. Credit-deposit ratio is the ratio of how much a bank lends out of the deposits it has mobilized.

3. If the Credit-deposit ratio is too high, it means that banks might not have enough liquidity to cover any unforseen fund requirements, may affect capital adequacy and asset-liability mis-match.

Which of the above statements is/are correct?

a) 1, 3

b) 1, 2

c) 2, 3

d) 2 only

c In recent years, the credit growth in the banking system is lower than deposit growth.

Weak demand in the market and risk aversion in the banking system have kept the credit growth at nearly half the level of the last year. Deposits surged 12 per cent, compared to 10 per cent growth a year ago.

Credit-deposit ratio, popularly CD ratio, is the ratio of how much a bank lends out of the deposits it has mobilized.

CD ratio helps in assessing a bank's liquidity and indicates its health - if the ratio is too low, banks may not be earning as much as they could be. If the ratio is too high, it means that banks might not have enough liquidity to cover any unforseen fund requirements, may affect capital adequacy and asset-liability mis-match.

Insights Oct-20 Economics 08/01/2021 2 17) Which of the following are RBI’s main risk provision accounts?

1. Contingency Fund

2. Currency and Gold Revaluation Account (CGRA)

3. Investment Revaluation Account Foreign Securities (IRA-FS)

4. Investment Revaluation Account-Rupee Securities (IRA-RS)

Select the correct answer code:

a) 1, 2, 3

b) 1, 3, 4

c) 2, 3, 4

d) 1, 2, 3, 4

d The central bank’s main risk provision accounts are Contingency Fund, Currency and Gold Revaluation Account (CGRA), Investment Revaluation Account Foreign Securities (IRA-FS) and Investment Revaluation Account-Rupee Securities (IRA-RS).
Insights Oct-20 Economics 08/01/2021 2 18) Laffer curve is a relationship between

a) Tax revenue and tax rate

b) Tax rate and tax buoyancy

c) Tax rate and tax elasticity

d) Tax buoyancy and tax elasticity

a
In economics, the Laffer curve illustrates a theoretical relationship between rates of taxation and the resulting levels of the government's tax revenue. The shape of the curve is a function of taxable income elasticity – i.e., taxable income changes in response to changes in the rate of taxation.
Insights Oct-20 Economics 08/01/2021 2 19) Consider the following statements regarding Reserve Money.

1. Reserve money consists of vault cash in banks and deposits of commercial banks with RBI.

2. Banks use this reserve to meet the demand for cash by account holders.

Which of the above statements is/are correct?

a) 1 only

b) 2 only

c) Both 1 and 2

d) Neither 1 nor 2

c Banks hold a part of the money people keep in their bank deposits as reserve money and loan out the rest to various investment projects. Reserve money consists of two thingsvault cash in banks and deposits of commercial banks with RBI. Banks use this reserve to meet the demand for cash by account holders.
Insights Oct-20 Economics 08/01/2021 2 20) Consider the following statements about International Finance Corporation (IFC).

1. It is a sister organization of the IMF.

2. It is the largest global development institution focused exclusively on the private sector in developing countries.

3. Its goals are to increase sustainable agriculture opportunities, improve healthcare and education.

Which of the above statements is/are correct?

a) 1, 2

b) 1, 3

c) 2, 3

d) 1, 2, 3

c IFC—a sister organization of the World Bank and member of the World Bank Group—is the largest global development institution focused exclusively on the private sector in developing countries. The World Bank Group has set two goals for the world to achieve by 2030: end extreme poverty and promote shared prosperity in every country.

Functions:

• It offers an array of debt and equity financing services and helps companies face their risk exposures, while refraining from participating in a management capacity.

• The corporation also offers advice to companies on making decisions, evaluating their impact on the environment and society, and being responsible.

• It advises governments on building infrastructure and partnerships to further support private sector development.

Since 2009, the IFC has focused on a set of development goals that its projects are expected to target. Its goals are to increase sustainable agriculture opportunities, improve healthcare and education, increase access to financing for microfinance and business clients, advance infrastructure, help small businesses grow revenues, and invest in climate health.

Insights Oct-20 Arts 08/01/2021 2 1) Consider the following statements about Mauryan art and architecture.

1. Stone was the principal building material.

2. Lauria Nandangarh pillar in Champaran was built during the time of Ashoka.

Which of the above statements is/are correct?

a) 1 only

b) 2 only

c) Both 1 and 2

d) Neither 1 nor 2

b The palace of Chandragupta Maurya was inspired by the Achaemenid palaces at Persepolis in Iran. Wood was the principal building material.

During the time of Ashoka, the inscription of pillars – as a symbol of the state or to commemorate battle victories – assumed a great significance. He also used pillars to propagate imperial sermons as well.

Example: Lauria Nandangarh pillar in Champaran, Sarnath pillar near Varanasi, etc.

Insights Oct-20 Arts 08/01/2021 2 2) Consider the following pairs:

    State Dance                                           Traditions

1. Tamil Nadu                        :                  Karakkatam

2. Mizoram                            :                  Tamang Selo

3. Maharashtra                      :                   Koli

Which of the above pairs is/are matched correctly?

a) 1, 2

b) 1, 3

c) 2, 3

d) 2 only

b Karakattam is an ancient folk dance of Tamil Nadu performed in praise of the rain goddess Mariamman. The ancient Tamil epic says that this type of dance has derived from Bharatham and a mixture of multiple forms of Tamil dance forms.

Tamang Selo is a genre of Nepali Folk song sung by the Tamang people and widely popular amongst the Nepali speaking community in Nepal, India and around the world. It is usually accompanied by Tamang instruments, the Damphu, Madal and Tungna.

Koli dance is the folk dance of fisher community of Maharashtra. In this dance, we can see the dance movements of girls are inspired from their daily life of fishing. Sea waves, throwing net in the sea, catching fishes are some of those movements.

Insights Oct-20 Arts 08/01/2021 2 3) Which of the following traditions finds their name in UNESCO List of Intangible Cultural Heritage?

1. Kutiyattam

2. Yoga

3. Kalbelia folk dance of Rajasthan

4. Kalaripayatttu

Select the correct answer code:

a) 2, 3

b) 3, 4

c) 1, 2, 3, 4

d) 1, 2, 3

d From India the Intangible Cultural Heritages added into this list include:

• Tradition of Vedic chanting

• Ramlila, the traditional performance of the Ramayana

Kutiyattam, Sanskrit theatre

Ramman, religious festival and ritual theatre of the Garhwal Himalayas.

Mudiyettu, ritual theatre and dance drama of Kerala

Kalbelia folk songs and dances of Rajasthan

Chhau dance

Buddhist chanting of Ladakh: recitation of sacred Buddhist texts in the trans-Himalayan Ladakh region, Jammu and Kashmir.

Sankirtana, ritual singing, drumming and dancing of Manipur

• Traditional brass and copper craft of utensil making among the Thatheras of Jandiala Guru, Punjab

• Yoga

Nawrouz

• Kumbh Mela

Insights Oct-20 Arts 08/01/2021 2 4) Consider the following statements regarding Sangeet Natak Akademi.

1. Sangeet Natak Akademi is the first National Academy of the arts set-up by the Republic of India.

2. It is presently an Autonomous Body of the Ministry of Culture, Government of India and is fully funded by the Government for implementation of its schemes and programmes.

Which of the above statements is/are correct?

a)1 only

b) 2 only

c) Both 1 and 2

d) Neither 1 nor 2

c The Sangeet Natak Akademi - India's national academy for music, dance and drama - is the first National Academy of the arts set-up by the Republic of India. It was created by a resolution of the (then) Ministry of Education, Government of India, dated 31 May 1952 notified in the Gazette of India of June 1952. The Akademi became functional the following year, with the appointment of its first Chairman, Dr P.V. Rajamannar, and the formation of its all-India council of representatives, the General Council.

The Sangeet Natak Akademi is presently an Autonomous Body of the Ministry of Culture, Government of India and is fully funded by the Government for implementation of its schemes and programmes.

Insights Oct-20 Arts 08/01/2021 2 5) Jamdani weave is

1. Fine muslin on which decorative motifs are woven

2. Tradition most popular in Western India

3. Work that employs use of both cotton and gold threads

Select the correct answer code:

a) 1, 2, 3

b) 2 only

c) 1, 3

d) 2, 3

c Jamdani is a fine muslin textile produced for centuries in South Rupshi of Narayanganj district in Bangladesh. The historic production of jamdani was patronized by imperial warrants of the Mughal emperors. Under British colonialism, the Bengali jamdani and muslin industries rapidly declined due to colonial import policies favoring industrially manufactured textiles. In more recent years, the production of jamdani has witnessed a revival in Bangladesh. Jamdani is typically woven using a mixture of cotton and gold thread. In 2013, the traditional art of weaving jamdani was declared a UNESCO Intangible Cultural Heritage of Humanity. In 2016, Bangladesh received geographical indication (GI) status for Jamdani Sari.
Insights Oct-20 Arts 08/01/2021 2 6) Ancient India had a rich tradition of music. Which of these treatises were written to expound the various styles and aspects of music?

1. Natya Shashtra

2. Brihat Samhita

3. Brihaddesi

Select the correct answer code:

 a) 1, 3

b) 1 only

c) 1, 2, 3

d) 2, 3

a Brihat Samhita, is a treatise on astrology.

The history of the system of music that prevailed in India from ancient times, goes back to the Vedas. The earliest treatise we have on music is the Natya Sastra of Bharata. Other treatises on music after Bharata, such as the Brihaddesi of Matanga, Sangeeta Ratnakara of Sharangadeva, Sangeet Sudhakara of Haripala, Swaramelakalanidhi of Ramamatya, etc., provide us information about the different aspects of music and its development during the different periods.

Insights Oct-20 Arts 08/01/2021 2 7) Consider the following statements:

1. There were no evidence of musical instruments found at the sites of Indus valley civilization.

2. Jaimini Brahmana speaks collectively of dance and music

3. In Hindustani music, Dhrupad is the pure music without distraction of words.

Which of the above statements is/are correct?

a) 1, 2

b) 2, 3

c) 1, 3

d) 1, 2, 3

b Musical instruments like seven-holed flute and Ravanahatha, have been recovered from the sites of Indus Valley Civilization.

The Jaimini Brahmana speaks collectively of dance and music.

Dhrupad is pure music without distraction of words.

Insights Oct-20 Arts 08/01/2021 2 8) Which of the following statement about Saptak is correct?

a) Emotions which are evoked through singing.

b) The rhythmic groupings of beats.

c) The seven swaras.

d) A system of classification of the ragas in different groups.

c The seven swaras together are called Saptak or Sargam.

The emotions which are evoked through the singing and playing of instruments are called Rasas.

The rhythmic grouping of beats are called Tala.

A system of classification of the ragas in different groups are called Thaat.

Insights Oct-20 Arts 08/01/2021 2 9) Consider the following:

1. Temple at Deogarh near Jhansi

2. Giant copper statue of Buddha originally found at Sultanganj

3. Delhi Iron pillar

4. Bagh caves paintings

The above statements belong to which of the following dynasties of India?

a) Peshwas

b) Guptas

c) Paramara

d) Western Ganga

b The Delhi Iron pillar of the Gupta period is still free from rust though completely exposed to sun and rain for so many centuries.

The paintings of the Gupta period are seen at Bagh caves near Gwalior.

Moreover, the mural paintings of Ajantha mostly illustrate the life of the Buddha as depicted in the Jataka stories.

There was little influence of Gandhara style on Gupta art. But the beautiful statue of standing Buddha at Mathura reveals a little Greek style. The Buddha statue unearthed at Saranath was unique piece of Gupta art.

Insights Oct-20 Arts 08/01/2021 2 10) Consider the following statements.

1. Udayagiri caves are famous for the Hathigumpha inscription which is carved out in Devanagari script.

2. Stupas became larger and more decorative in the post- Mauryan period.

3. The Gandhara School of Art had the influence of Greek and Roman traditions.

Which of the above statements is/are correct?

a) 1, 2

b) 1, 3

c) 2, 3

d) 1, 2, 3

c Udayagiri caves are famous for the Hathigumpha inscription which is carved out in Brahmi script.

Stupas became larger and more decorative in the post- Mauryan period. Stone was increasingly used in place of wood and brick.

The Gandhara School of Art developed in the western frontiers of Punjab, near modern day Peshawar and Afghanistan. The Greek invaders brought with them the traditions of the Greek and Roman sculptors, which influenced the local traditions of the region.

Thus, Gandhara School also came to be known as Greco-Indian School of Art.

Insights Oct-20 Arts 08/01/2021 2 11) Which among the following Indus Valley civilization Sites belong to India?

1. Harappa

2. Rakhigarhi

3. Dholavira

4. Mohenjodaro

5. Ropar

Select the correct answer code:

a) 1, 2, 4

b) 3, 4, 5

c) 2, 3, 5

d) 1, 2, 3, 4, 5

c While Harappa and Mohenjodaro are situated in Pakistan, the important sites excavated in India are Lothal and Dholavira in Gujarat, Rakhigarhi in Haryana, Ropar in the Punjab, Kalibangan and Balathal in Rajasthan.
Insights Oct-20 Arts 08/01/2021 2 12) According to experts, which of the following statements regarding Kalibangan is/are correct?

1. Kalibangan in Rajasthan has given the evidence of the earliest ploughed agricultural field ever revealed through an excavation.

2. It is also a site which has given an evidence of earliest archaeologically recorded Earthquake.

Select the correct answer code:

a) 1 only

b) 2 only

c) Both 1 and 2

d) Neither 1 nor 2

c Kalibangan has given the evidence of the earliest (2800 BC) ploughed agricultural field ever revealed through an excavation as per Dr. B Lal. It is also a site which has given an evidence of earliest recorded “Earthquake”.
Insights Oct-20 Arts 08/01/2021 2 13) Aihole inscription, is known as the Cradle of Indian architecture, was issued under the reign of

a) Chalukyas

b) Pallavas

c) Rashtrakutas

d) Marathas

a Aihole was the first capital of Chalukyas where they built numerous temples dating back to the 6th century CE.

Many inscriptions found at Aihole, but the inscription which found at Meguti Temple popularly known as Aihole inscription, which has the significance in the history of India, witnessed for the many historical events of Chalukyas.

The Aihole inscription issued by Pulakeshin II gives the details of his reign.

Insights Oct-20 Arts 08/01/2021 2 14) Consider the following statements regarding Bardo Chham.

1. Bardo Chham is a folk dance performed in Arunachal Pradesh.

2. It is based on the stories of good and evil.

3. The performers wear masks representing different animals.

Which of the above statements is/are correct?

a) 1, 2

b) 1, 3

c) 2, 3

d) 1, 2, 3

d Bardo Chham

• Region/Area: Arunachal Pradesh

• Occasion: Performed during festivals

• Highlight: The performers wear masks representing different animals

• Purpose: Depiction of good over evil

• Bardo Chham is performed by the Sherdukpen tribe of the West Kameng district. The dancers perform to the beats of many percussion instruments.

Insights Oct-20 Arts 08/01/2021 2 15) Which of the following are the Folk dances of North East India.

1. Bihu

2. Bagurumba

3. Kummi

4. Dhol Cholom

Select the correct answer code:

a) 1, 2, 3

b) 1, 2, 4

c) 1, 2

d) 1, 2, 3, 4

b Bihu

• Region/Area: Assam

• Occasion: Performed during the Bihu festival

• Highlight: Lyrics used in the songs propagates love

Bagurumba

• Region/Area: Assam

• Occasion: Performed during various festivals

• Highlight: The dance is inspired from nature

Dhol Cholom

• Region/Area: Manipur

• Occasion: Performed during Holi festival

• Highlight: The dancers display acrobatic moves while playing dhol – a large drum

Kummi is a folk dance, popular in Tamil Nadu and Kerala.

Insights Oct-20 Arts 08/01/2021 2 16) Consider the following statements regarding Metal works during Harappan civilization.

1. Copper, tin, gold and silver were easily available locally, thus obviating the need to import them from distant places.

2. The earliest evidence of metallurgy in the Indian subcontinent comes from the site of Mehrgarh.

Which of the above statements is/are correct?

a) 1 only

b) 2 only

c) Both 1 and 2

d) Neither 1 nor 2

b While some of the raw materials that the Harappans used were available locally, many items such as copper, tin, gold, silver and precious stones had to be brought from distant places.

The earliest evidence of metallurgy in the Indian subcontinent comes from the site of Mehrgarh in Baluchistan dated to circa 6500 BCE, a part of early Harappan civilization. It is recovered in the form of a tiny copper bead from grave of a child.

Insights Oct-20 Arts 08/01/2021 2 17) Consider the following statements about Kalighat paintings which originated in Bengal.

1. The school produced paintings that were irreproducible by lithography.

2. The school restricted itself to the depiction of natural, nationalistic and secular themes.

Which of the above is/are incorrect?

a) 1 only

b) 2 only

c) Both 1 and 2

d) Neither 1 nor 2

c Kalighat Paintings

The paintings over a period of time developed as a distinct school of Indian painting. An important achievement of the Kalighat artistes was that they made simple paintings and drawings, which could easily be reproduced by lithography.

From the depiction of Hindu gods, goddesses, and other mythological characters, the Kalighat paintings developed to reflect a variety of themes.

The artists also chose to portray secular themes and personalities and in the process played a role in the Independence movement. They painted heroic characters like Tipu Sultan and Rani Lakshmibai.

Insights Oct-20 Arts 08/01/2021 2 18) Consider the following statements about Dhrupad, a genre in Hindustani classical music.

1. It finds mention in Ain-e-Akbari.

2. The phrases of Dhrupad alapa are slow and contemplative in the beginning, but the tempo increases in stages.

3. It is a sub-set of Khyal form of classical singing.

Which of the above statements is/are correct?

a) 1, 2

b) 2, 3

c) 2 only

d) 1, 2, 3

a Abul Fazl, courtier and chronicler at the court of the Emperor Akbar, defines the dhrupad verse form in his Ain-e-Akbari as "four rhyming lines, each of indefinite prosodic length.

The elaboration of Dhrupad alap is done using the syllables of a mantric phrase, it is slow in the beginning and raises tempo after some time.

Dhrupad and khyāl are the two forms of classical singing that exist today in North India.

Insights Oct-20 Arts 08/01/2021 2 19) Thumri is a common genre of semi-classical Indian music with its origin in

a) Odisha

b) Maharashtra

c) Uttar Pradesh

d) Rajasthan

c Originated in eastern part of Uttar Pradesh, the termthumri’ is derived from the Hindi verb thumakna which means “to walk with dancing steps so as to make the ankle-bells tinkle.”

The form is, thus, connected with dance, dramatic gestures, mild eroticism, evocative love poetry and folk songs of Uttar Pradesh, though there are regional variations.

The text is romantic or devotional in nature, and usually revolves around a girl’s love for Krishna.

Thumri is characterized by its sensuality, and by a greater flexibility with the raag.

The lyrics are usually in Uttar Pradesh dialects of Hindi called Awadhi and Braj Bhasha.

Insights Oct-20 Arts 08/01/2021 2 20) Known as the “Khajuraho of Vidarbha”, the temple of Markandadeo, is built in which of these styles?

a) Dravidian

b) Nagara

c) Vesara

d) Ahom

b Known as the “Khajuraho of Vidarbha”, the temple of Markandadeo is situated on the bank of River Wainganga in district Gadchiroli of Maharashtra.

The temples belong to the Nagara group of temples of North India. On stylistic grounds, their date ranges in between 9- 12th centuries CE. The temples belong to saiva, vaishnava and sakta faith. Most of the temples have a simple plan, with ardhamandapa, mandapa, antarala and garbhagriha forming the component of the entire set up.

Insights Oct-20 History 08/01/2021 2 1) Consider the following statements

1. Harappan Civilization got its name from the city that was discovered first.

2. In the late-Harappan stage the Indus culture rose to its peak and started spreading in southern India and across Indian Frontiers.

Which of the above statements is/are correct?

a) 1 only

b) 2 only

c) Both 1 and 2

d) Neither 1 nor 2

a The Indus civilisation is also known as the Harappan Civilisation, after its type site, Harappa, the first of its sites to be excavated early in the 20th century in what was then the Punjab province of British India and now is Pakistan.

Late Harappan: Around 1900 BCE signs of a gradual decline began to emerge, and by around 1700 BCE most of the cities had been abandoned.

Insights Oct-20 History 08/01/2021 2 2) Consider the following statements regarding Vedic civilization

1. Sabha and Samiti were legislative assemblies at tribal or territorial level.

2. The king was autocratic with main responsibility of being protector of the tribe and the cattle wealth.

3. Samiti was not open for women in early Vedic period.

Which of the above statements is/are correct?

a) 1, 2

b) 1 only

c) 1, 3

d) 1, 2, 3

b The king was not autocratic. He had to abide by the decision of assemblies called sabha, samiti. Sabha and samiti were legislative assemblies at tribal and territorial level. The samiti was a general assembly which was open for all including women.
Insights Oct-20 History 08/01/2021 2 3) Who among the following rulers founded the Vikramsila University?

a) Dharmapala

b) Bhoja

c) Gopala

d) Kumaragupta

a Vikramashila University was one of the two most important centres of learning in India during the Pala Empire, along with Nalanda. Vikramashila was established by King Dharmapala in response to a supposed decline in the quality of scholarship at Nalanda.
Insights Oct-20 History 08/01/2021 2 4) Ashokan inscriptions were majorly engraved in the Brahmi script and Kharoshthi script. With reference to the above scripts consider the following statements.

1. Brahmi script was written from left to right where as Kharoshthi script was written from right to left.

2. Brahmi script prevailed in the north western region of India whereas Kharoshthi script prevailed in the rest of the country.

Which of the above statements is/are incorrect?

a) 1 only

b) 2 only

c) Both 1 and 2

d) Neither 1 nor 2

b Brahmi script was written from left to right where as Kharoshthi script was written from right to left.

Kharoshthi script prevailed in the north western region of India whereas Brahmi script prevailed in the rest of the country.

Insights Oct-20 History 08/01/2021 2 5) With reference to the entry of European powers into India, which one of the following statements is not correct?

a) The Portuguese captured Goa in 1499

b) The English opened their first factory in South India at Masulipatam

c) In Eastern India, the English Company opened its first factory in Orissa in 1633

d) Under the leadership of Dupleix, the French occupied Madras in 1746

a The Portuguese captured Goa from rulers of Bijapur in 1510 and not in 1499.
Insights Oct-20 History 08/01/2021 2 6) Match List-I (ancient places) with List-II (their Present location):

        List-I                                           List-II

A. Banawali                                        1. Sind

B. Lothal                                             2. Rajasthan

C. Kalibangan                                     3. Gujarat

D. Kot Diji                                           4. Haryana

Select the correct answer code:

    A B C D

a) 4 3 2 1

b) 1 2 3 4

c) 2 3 4 1

d) 3 4 2 1

a
Insights Oct-20 History 08/01/2021 2 7) Consider the following statements regarding Harappan civilization.

1. The citadels were mainly inhabited by the common people.

2. The Great Bath was used for ritual bathing and it was made up of stone

Which of the above statements is/are correct?

a) 1 only

b) 2 only

c) Both 1 and 2

d) Neither 1 nor 2

d Citadels were inhabited by elite people and great bath was made up of burnt brick.
Insights Oct-20 History 08/01/2021 2 8) The easternmost Harappan site amongst the following is

a) Rakhigarhi

b) Chanhudaro

c) Dholavira

d) Ganweriwala

a
Insights Oct-20 History 08/01/2021 2 9) Consider the following statements about Government of India Act of 1858.

1. The Act abolished the East India Company, and transferred the powers of government, territories and revenues to the British Crown.

2. It ended the system of double government.

3. It made a beginning of representative institutions by associating Indians with the law-making process.

Which of the above statements is/are correct?

a) 1, 3

b) 1, 2

c) 2, 3

d) 1, 2, 3

b Government of India Act of 1858:

• The act known as the Act for the Good Government of India, abolished the East India Company, and transferred the powers of government, territories and revenues to the British Crown.

• It provided that India henceforth was to be governed by, and in the name of, Her Majesty. It changed the designation of the Governor-General of India to that of Viceroy of India. He (viceroy) was the direct representative of the British Crown in India. Lord Canning thus became the first Viceroy of India.

• It ended the system of double government by abolishing the Board of Control and Court of Directors.

Indian Councils Act of 1861 made a beginning of representative institutions by associating Indians with the law-making process.

Insights Oct-20 History 08/01/2021 2 10) Consider the following statements regarding Gandhi-Irwin Pact.

1. Gandhi-Irwin Pact was a political agreement signed after the second Round Table Conference in London.

2. It marked the end of a period of civil disobedience (satyagraha) in India against British rule that Gandhi and his followers had initiated with the Salt March.

Which of the above statements is/are correct?

a) 1 only

b) 2 only

c) Both 1 and 2

d) Neither 1 nor 2

b The 'Gandhi - Irwin Pact' was a political agreement signed by Mahatma Gandhi and Lord Irwin, Viceroy of India, on 5 March 1931 before the second Round Table Conference in London. It marked the end of a period of civil disobedience (satyagraha) in India against British rule that Gandhi and his followers had initiated with the Salt March.
Insights Oct-20 History 08/01/2021 2 11) Consider the following statements regarding Palaeolithic age.

1. It spans nearly one-third of human history.

2. Evidence of the existence of ostriches were found in India during this period.

3. Bone tools could be found, but stone tools were not made in this period.

Which of the above statements is/are correct?

a) 1, 2

b) 2 only

c) 2, 3

d) 1, 3

b The Palaeolithic period extends from 2 million years ago to about 12,000 years ago. This long stretch of time is divided into the Lower, Middle and Upper Palaeolithic. This long span of time covers 99% of human history.

Stone tools found during this period are generally tiny, and are called microliths.

Ostriches were found in India during the Palaeolithic period. Large quantities of ostrich egg shells were found at Patne in Maharashtra. Designs were engraved on some pieces, while beads were also made out of them.

Insights Oct-20 History 08/01/2021 2 12) Which among the following sites provide the earliest evidence of settled agriculture in the Indian subcontinent?

a) Inamgarh

b) Dholavira

c) Mehrgarh

d) Kalibangan

c Located in Kacchi plains of Baluchistan, Mehrgarh provides the earliest evidence of settled agriculture in the subcontinent and probably south Asia. It is considered a precursor to Indus Valley Civilization
Insights Oct-20 History 08/01/2021 2 13) In Ancient India, shrenis served which of these purposes?

1. Provided training to crafts persons

2. Procured raw material for merchants

3. Served as financial institutions for money deposits

4. Supported religious institutions by donations

Select the correct answer code:

a) 1 only

b) 2, 3, 4

c) 1, 3

d) 1, 2, 3, 4

d Many crafts persons and merchants formed associations known as shrenis.

These shrenis of crafts persons provided training, procured raw material, and distributed the finished product.

Then shrenis of merchants organised the trade.

Shrenis also served as banks, where rich men and women deposited money.

This was invested, and part of the interest was returned or used to support religious institutions such as monasteries.

Insights Oct-20 History 08/01/2021 2 14) The Gandhi-Irwin Pact included

1. Right to make salt in coastal villages for personal consumption and sale.

2. Right to peaceful and non-aggressive picketing

3. Acceptance of Gandhiji’s suggestion for enquiry into police excesses

4. Release of all political prisoners not convicted of violence

Select the correct answer code:

a) 1, 2, 3

b) 2, 4

c) 2, 3, 4

d) 1, 2, 4

b Gandhi-Irwin Pact placed the Congress on an equal footing with the government.

Irwin on behalf of the government agreed on—

1. immediate release of all political prisoners not convicted of violence;

2. remission of all fines not yet collected;

3. return of all lands not yet sold to third parties;

4. lenient treatment to those government servants who had resigned; right to make salt in coastal villages for personal consumption (not for sale);

6. right to peaceful and non-aggressive picketing; and

7. withdrawal of emergency ordinances.

The viceroy, however, turned down two of Gandhi’s demands—

(i) public inquiry into police excesses, and

(ii) commutation of Bhagat Singh and his comrades’ death sentence to life sentence.

Gandhi on behalf of the Congress agreed

(i) to suspend the civil disobedience movement, and

(ii) to participate in the next Round Table Conference

Insights Oct-20 History 08/01/2021 2 15) Arrange the following events in their correct chronological order:

1. Nehru Report

2. First RTC

3. First May Day in India

4. Dandi March

Select the correct code:

a) 3 1 4 2

b) 3 1 2 4

c) 1 3 4 2

d) 1 3 2 4

a In 1923, the first May day was celebrated in India in Madras.

1928Nehru Report

Dandi MarchMarch 12-April 6, 1930

First Round Table ConferenceNovember 1930 – January 1931

Insights Oct-20 History 08/01/2021 2 16) The advent or development of which of the following was termed as “Neolithic Revolution”?

a) Metallurgy

b) Agriculture

c) Megaliths

d) Painted pottery

b The Neolithic Revolution, also called the Agricultural Revolution, marked the transition in human history from small, nomadic bands of hunter-gatherers to larger, agricultural settlements and early civilization.
Insights Oct-20 History 08/01/2021 2 17) Consider the following statements.

1. According the Aitreya Brahmana a daughter has been described as a source of pride.

2. During later vedic period, gold and silver coins were used as media of exchange.

Which of the above statements is/are incorrect?

a) 1 only

b) 2 only

c) Both

d) None

a According the Aitreya Brahmana a daughter has been described as a source of misery.

Besides nishka of the Rig Vedic period, gold and silver coins like satamana and krishnala were used as media of exchange during later vedic period.

Insights Oct-20 History 08/01/2021 2 18) In the early decades after the Permanent Settlement, zamindars regularly failed to pay the revenue to the East India Company. What are the reasons for their failure?

1. The initial revenue demands were very high.

2. The prices of agricultural produce were depressed, which made it difficult for the ryots to pay their dues to the zamindar.

3. The revenue was invariable, regardless of the Harvest.

Select the correct answer code:

a) 1, 2

b) 1, 3

c) 1, 2, 3

d) 2, 3

c In the early decades after the Permanent Settlement, however, zamindars regularly failed to pay the revenue demand and unpaid balances accumulated. The reasons for this failure were various.

First: the initial demands were very high. The Company pegged the revenue demand high, arguing that the burden on zamindars would gradually decline as agricultural production expanded and prices rose.

Second: this high demand was imposed in the 1790s, a time when the prices of agricultural produce were depressed, making it difficult for the ryots to pay their dues to the zamindar.

Third: the revenue was invariable, regardless of the harvest, and had to be paid punctually.

Insights Oct-20 History 08/01/2021 2 19) During the time of British Raj, the term ‘Amlah’ refer to

a) Rich Peasants

b) Officer of the Zamindar

c) Village headman

d) Sharecropper

b At the time of rent collection, an officer of the zamindar, usually the Amlah, came around to the village. But rent collection was a perennial problem. Sometimes bad harvests and low prices made payment of dues difficult for the ryots. At other times ryots deliberately delayed payment.
Insights Oct-20 History 08/01/2021 2 20) Consider the following statements about Indian Councils Act of 1909.

1. It increased the size of the legislative councils, both Central and provincial.

2. For the first time, it provided for the association of Indians with the executive Councils of the Viceroy and Governors.

3. It introduced a system of communal representation for Muslims and Christians.

Which of the above statements is/ are correct?

a) 1, 2

b) 2, 3

c) 1, 3

d) 1, 2, 3

a Indian Councils Act of 1909 is also known as Morley-Minto Reforms.

It considerably increased the size of the legislative councils, both Central and provincial.

It provided (for the first time) for the association of Indians with the executive Councils of the Viceroy and Governors.

It introduced a system of communal representation for Muslims by accepting the concept of ‘separate electorate’. Under this, the Muslim members were to be elected only by Muslim voters.

Insights Oct-20 Environment 08/01/2021 2 1) Hot water discharged by industries is considered as a water pollutant because

1. Higher water temperature generally decreases the level of dissolved oxygen of water.

2. High temperature limits oxygen dispersion into deeper waters, contributing to anaerobic conditions in deep water.

Which of the above statements is/are correct?

a) 1 only

b) 2 only

c) Both 1 and 2

d) Neither 1 nor 2

c Elevated temperature typically decreases the level of dissolved oxygen of water. This can harm aquatic animals such as fish, amphibians and other aquatic organisms.

Thermal pollution may also increase the metabolic rate of aquatic animals, as enzyme activity, resulting in these organisms consuming more food in a shorter time than if their environment were not changed.

High temperature limits oxygen dispersion into deeper waters, contributing to anaerobic conditions. This can lead to increased bacteria levels when there is ample food supply. Many aquatic species will fail to reproduce at elevated temperatures.

Insights Oct-20 Environment 08/01/2021 2 2) Three of the following criteria have contributed to the recognition of Western Ghats, Sri Lanka and Indo Burma regions as hotspots of biodiversity

1. Species richness 2. Vegetation density 3. Endemism 4. Ethno-botanical importance 5. Threat perception 6. Adaption of flora and fauna to warm and humid conditions Which three of the above are correct criteria for recognizing biodiversity hotspots? a) 1, 2, 6 b) 2, 4, 6 c) 1, 3, 5 d) 3, 4, 6

c Biodiversity Hotspots

Vegetation density, botanical importance, adaptation of flora and fauna are not considered while recognizing hot spots of biodiversity.

Insights Oct-20 Environment 08/01/2021 2 3) Consider the following statements

1. Shola forest-grassland ecosystem is characterized by patches of forest of stunted evergreen shola trees in the valleys and grasslands of Tamil Nadu only. 2. Exotic invasive trees like pine, acacia and eucalyptus are largely seen in Shola forests. Which of the above statements is/are correct? a) 1 only b) 2 only c) Both 1 and 2 d) Neither 1 nor 2

b Shola forests are tropical Montane forests found in the valleys separated by rolling grasslands only in the higher elevations. They are found only in South India in the Southern Western Ghats.

• The Shola forests are generally said to be found in altitudes above 2000 metres of sea-level. Although they are found from altitudes higher than 1600 metres. • Shola forests are a native only to the Southern Western Ghats. • They are found only in the high-altitude mountains of the states Karnataka, Kerala and Tamil Nadu. • Exotic invasive trees like pine, acacia and eucalyptus are largely seen in Shola forests

Insights Oct-20 Environment 08/01/2021 2 4) Consider the following pairs

     List I                                                                       List II 1. Tipeshwar Wildlife Sanctuary           -                Gujarat 2. Bhandavgarh National Park             -                Madhya Pradesh 3. Raiganj Wildlife Sanctuary               -                West Bengal Which of the above pairs is/are correct? a) 1, 3 b) 2, 3 c) 1, 2 d) 1, 2, 3

b Tipeshwar Wildlife Sanctuary is located in Maharashtra.

Bhandavgarh National Park  -  Madhya Pradesh Raiganj Wildlife Sanctuary -  West Bengal

Insights Oct-20 Environment 08/01/2021 2 5) Snow Leopard is naturally found in which of the following states?

1. Jammu and Kashmir 2. Arunachal Pradesh 3. Sikkim 4. Uttar Pradesh Select the correct answer code: a) 1, 3 b) 2, 3 c) 1, 2, 3, 4 d) 1, 2, 3

d The snow leopard inhabits the higher Himalayan and trans-Himalayan landscape in Jammu and Kashmir, Himachal Pradesh, Uttarakhand, Sikkim, and Arunachal Pradesh.

Snow Leopard is not naturally found in Uttar Pradesh.

Insights Oct-20 Environment 08/01/2021 2 6) Consider the following statements regarding BirdLife International.

1. BirdLife International is an inter-governmental organisation that strives to conserve birds and their habitats. 2. BirdLife International's priority include preventing extinction of bird species and empowering conservationists worldwide. 3. It is the official International Union for Conservation of Nature’s Red List authority for birds. Which of the above statements is/are correct? a) 1, 2 b) 1, 3 c) 2, 3 d) 1, 2, 3

c BirdLife International is a global partnership of non-governmental organizations that strives to conserve birds and their habitats.

BirdLife International's priorities include preventing extinction of bird species, identifying and safeguarding important sites for birds, maintaining and restoring key bird habitats, and empowering conservationists worldwide. BirdLife International has identified 13,000 Important Bird and Biodiversity Areas and is the official International Union for Conservation of Nature’s Red List authority for birds. As of 2015, BirdLife International has established that 1,375 bird species (13% of the total) are threatened with extinction (critically endangered, endangered or vulnerable). BirdLife International publishes a quarterly magazine, BirdLife: The Magazine, which contains recent news and authoritative articles about birds and their conservation.

Insights Oct-20 Environment 08/01/2021 2 7) How does silting of water bodies affect aquatic life?

1. Coral polyps grow more favourably in silted and shallow water bodies. 2. Silt acts as a vehicle for certain pesticides into water bodies which adversely affect aquatic life. Select the correct answer code: a) 1 only b) 2 only c) Both 1 and 2 d) Neither 1 nor 2

b Hard bottom communities like corals and mussel banks (including oysters) are more sensitive to siltation. Siltation adversely affects coral population.

In rural areas the erosion source is typically soil degradation due to intensive or inadequate agricultural practices, leading to soil erosion, especially in fine-grained soils such as loess. The result will be an increased amount of silt and clay in the water bodies that drain the area often adding unwanted fertilizers to these bodies.

Insights Oct-20 Environment 08/01/2021 2 8) Light pollution can have which of the following effects on the ecology and biodiversity?

1. It adversely affects the migration of birds that navigate using the stars. 2. It is known to disturb the reproductive cycles of some animals. 3. It disturbs circadian rhythms in humans affecting the sleep pattern. Select the correct answer code: a) 1 only b) 2, 3 c) 1, 3 d) 1, 2, 3

d Light pollution is excessive brightness that causes visual discomfort.

• Skyglow is the brightening of the night sky over inhabited areas. It is caused by cluttered, bright, excessive and confusing groupings of light sources. • It affects the reproductive cycles of those animals that depend on sensing light movements or seasonal movements to reproduce. Since birds migrate using star light, it can confuse them, and even disorient night-flying insects. • Excessive blue light emitted from LEDs directly affects sleep pattern in Human by suppressing the production of the hormone melatonin, which mediates the sleep-wake cycle in humans.

Insights Oct-20 Environment 08/01/2021 2 9) Consider the following statements about Biodiversity loss.

1. Biodiversity loss describes the decline in the number, genetic variability, and variety of species, and the biological communities in a given area. 2. A loss in biodiversity can make plants and animals more vulnerable to pests and diseases. 3. Biodiversity loss is always anthropogenic. Which of the above statements is/are correct? a) 1, 3 b) 2, 3 c) 1, 2 d) 1, 2, 3

c Biodiversity loss describes the decline in the number, genetic variability, and variety of species, and the biological communities in a given area. This loss in the variety of life can lead to a breakdown in the functioning of the ecosystem where decline has happened.

An area’s biodiversity increases and decreases with natural cycles. Seasonal changes, such as the onset of spring, create opportunities for feeding and breeding, increasing biodiversity as the populations of many species rise. In contrast, the onset of winter temporarily decreases an area’s biodiversity, as warm-adapted insects die and migrating animals leave. In addition, the seasonal rise and fall of plant and invertebrate populations (such as insects and plankton), which serve as food for other forms of life, also determine an area’s biodiversity.

Insights Oct-20 Environment 08/01/2021 2 10) Consider the following statements about Fly Ash.

1. Fly ash is the end product of combustion during the process of power generation in the coal based thermal power plants. 2. Fly ash can be used in combination with other alkaline materials to transform sewage sludge into organic fertilizer or biofuel. Which of the above statements is/are incorrect? a) 1 only b) 2 only c) Both 1 and 2 d) Neither 1 nor 2

d Fly ash, the end product of combustion during the process of power generation in the coal based thermal power plants, is a proven resource material for many applications of construction industries and currently is being utilized in manufacturing of Portland Cement, bricks/blocks/tiles manufacturing, road embankment construction and low-lying area development, etc.

Fly ash contains heavy metals from coal, a large amount of PM 2.5 and black carbon (BC). Fly ash, in view of its alkalinity and water absorption capacity, may be used in combination with other alkaline materials to transform sewage sludge into organic fertilizer or biofuel.

Insights Oct-20 Environment 08/01/2021 2 11) Which of the following are considered as Ecosystem Services?

1. Erosion and flood control 2. Building of knowledge and the spreading of ideas 3. Creation of soils 4. Water purification Select the correct answer code: a) 1, 2, 3 b) 1, 3, 4 c) 2, 3, 4 d) 1, 2, 3, 4

d Four major categories of ecosystem services: provisioning, regulating, cultural and supporting services.

Provisioning Services: A provisioning service is any type of benefit to people that can be extracted from nature. Along with food, other types of provisioning services include drinking water, timber, wood fuel, natural gas, oils, plants that can be made into clothes and other materials, and medicinal benefits. Regulating Services: A regulating service is the benefit provided by ecosystem processes that moderate natural phenomena. Regulating services include pollination, decomposition, water purification, erosion and flood control, and carbon storage and climate regulation. Cultural Services: A cultural service is a non-material benefit that contributes to the development and cultural advancement of people, including how ecosystems play a role in local, national, and global cultures; the building of knowledge and the spreading of ideas; creativity born from interactions with nature (music, art, architecture); and recreation. Supporting Services: Ecosystems themselves couldn't be sustained without the consistency of underlying natural processes, such as photosynthesis, nutrient cycling, the creation of soils, and the water cycle.

Insights Oct-20 Environment 08/01/2021 2 12) Which of the following are the sources of Persistent Organic Pollutants (POPs)?

1. Improper use and/or disposal of agrochemicals 2. Elevated temperatures and combustion processes 3. Unwanted by-products of industrial processes Select the correct answer code: a) 1, 2 b) 1, 3 c) 2, 3 d) 1, 2, 3

d Sources of pollution from POPs includes :

improper use and/or disposal of agrochemicals and industrial chemicals, elevated temperatures and combustion processes, and unwanted by-products of industrial processes or combustion.

Insights Oct-20 Environment 08/01/2021 2 13) Which of the following can be threats to the biodiversity of a geographical area?

1. Global warming 2. Fragmentation of habitat 3. Invasion of alien species 4. Promotion of vegetarianism Select the correct answer code: a) 1, 2, 3 b) 2, 3 c) 1, 4 d) 1, 2, 3, 4

a Global Warming, fragmentation of habitat and invasion of alien species can be threats to the biodiversity of a geographical area.

Promotion of vegetarianism doesn’t threat biodiversity of geographical area.

Insights Oct-20 Environment 08/01/2021 2 14) Due to improper/indiscriminate disposal of old and used computers or their parts, which of the following are released into the environment as e-waste?

1. Beryllium 2. Cadmium 3. Chromium 4. Heptachlor 5. Mercury 6. Lead 7. Plutonium Select the correct answer code: a) 1, 3, 4, 6, 7 b) 1, 2, 3, 5, 6 c) 2, 4, 5, 7 d) 1, 2, 3, 4, 5, 6, 7

b Heptachor is an insecticide. Plutonium is radioactive and hence not used in day to day computer/electronic items.
Insights Oct-20 Environment 08/01/2021 2 15) Consider the following statements regarding Phosphorus and Phosphorus Cycle.

1. Phosphorus cycle is largely atmospheric and easily dissolves in water from air. 2. Phosphorus occurs as a mineral in phosphate rocks and enters the Phosphorus cycle from erosion and mining activities. 3. Phosphorus is responsible for excessive growth of rooted and free-floating microscopic plants in water bodies. Which of the above statements is/are correct? a) 1, 2 b) 2, 3 c) 1, 3 d) 1, 2, 3

b Phosphorous cycle is mainly terrestrial. The main storage for phosphorus is in the earth's crust. On land phosphorus is usually found in the form of phosphates.

It occurs in large amounts as a mineral in phosphate rocks and enters the cycle from erosion and mining activities. By the process of weathering and erosion phosphates enter rivers and streams that transport them to the ocean. Being an important nutrient, phosphorous promotes eutrophication in lakes. Along with nitrogen related compounds it leads to undesirable situations like algal bloom.

Insights Oct-20 Environment 08/01/2021 2 16) Which of the following contribute to the oxygen cycle on earth?

1. Oxidation of volcanic gases 2. Fixation of N2 by lightning 3. Chemical weathering Select the correct answer code: a) 1 only b) 2, 3 c) 1, 2 d) 1, 2, 3

d The largest reservoir of Earth's oxygen is within the silicate and oxide minerals of the crust and mantle (99.5%). Only a small portion has been released as free oxygen to the biosphere (0.01%) and atmosphere (0.36%). The main source of atmospheric free oxygen is photosynthesis.

The processes that lead to loss of oxygen are: • Aerobic respiration (largest – more than 75%) • Microbial oxidation • Combustion of fossil fuel (anthropogenic – around 6-7% loss only) • Photochemical oxidation • Fixation of N2 by lightning • Fixation of N2 by industry (anthropogenic) • Oxidation of volcanic gases • Chemical weathering • Surface reaction of O3

Insights Oct-20 Environment 08/01/2021 2 17) Consider the following statements regarding Alpha and beta diversity.

1. Alpha diversity is the diversity within an ecosystem which is generally described as the number of species. 2. Beta diversity is the geographic diversity which refers to the total diversity of a region. Which of the above statements is/are correct? a) 1 only b) 2 only c) Both 1 and 2 d) Neither 1 nor 2

a Alpha is the diversity within an ecosystem which is generally described as the number of species.

Beta diversity involves the comparison of different ecosystems in environmental gradients, for example, in a mountainous area within a coastal area. Beta diversity shows us the size of the change of species from one ecosystem to another. Gamma diversity refers to the total diversity of a region, i.e. the geographic diversity. It is the sum of the alpha diversity of various ecosystems.

Insights Oct-20 Environment 08/01/2021 2 18) Consider the following statements about Savanna type climate.

1. It is a transitional type of climate between the equatorial forest and the trade wind hot deserts. 2. It is characterized by high and year-round rainfall. Which of the above statements is/are incorrect? a) 1 only b) 2 only c) Both 1 and 2 d) Neither 1 nor 2

b • Savanna or Sudan climate is a transitional type of climate found between the equatorial forests & trade wind hot deserts.

• It is confined within the tropics (Tropic of Cancer & Tropic of Capricorn) & is best developed in Sudan, where dry & wet climate are most distinct, hence named Sudan climate. • It covers much of Africa (Kenya, Nigeria, Gambia) as well as large areas of Australia, South America (Brazilian highlands), and India. • Sudan climate is characterized by an alternate & distinct hot, rainy season (from May to Sep) & cool, dry season (Oct – April) in Northern Hemisphere & vice versa in Southern Hemisphere.

Insights Oct-20 Environment 08/01/2021 2 19) Consider the following statements.

1. Stenothermal organisms can tolerate and thrive in a wide range of temperatures. 2. Freshwater animals cannot live for long in sea water because of the osmotic problems. Which of the above statements is/are correct? a) 1 only b) 2 only c) Both d) Neither 1 nor 2

b A few organisms can tolerate and thrive in a wide range of temperatures (they are called eurythermal), but, a vast majority of them are restricted to a narrow range of temperatures (such organisms are called stenothermal).

Many freshwater animals cannot live for long in sea water and vice versa because of the osmotic problems, they would face.

Insights Oct-20 Environment 08/01/2021 2 20) The highly degraded organic matter rich in phosphorus, nitrogen and potassium in particular, resulting from the activity of earthworms is known as:

a) Compost bedding b) Humus c) Worm casting d) Vermicomposting

d Vermicomposting is a type of composting in which certain species of earthworms are used to enhance the process of organic waste conversion and produce a better end-product. It is a mesophilic process utilizing microorganisms and earthworms. Earthworms feeds the organic waste materials and passes it through their digestive system and gives out in a granular form (cocoons) which is known as vermicompost.

A wide range of organic residues, such as straw, husk, leaves, stalks, weeds etc. can be converted into vermicompost. Other potential feedstock for vermicomposting production are livestock wastes, poultry litter, dairy wastes, food processing wastes, organic fraction of MSW, bagasse, digestate from biogas plants etc.

Insights Nov-20 Polity 08/01/2021 2 1) Which of the following can be challenged in the courts as violating a Fundamental Right and hence, can be declared as void?

1. Constitutional amendment 2. Permanent laws enacted by the Parliament 3. Ordinances issued by the president 4. Statutory instruments like bye-law, rule, regulation or notification. Select the correct answer code: a) 1, 2, 3 b) 1, 3, 4 c) 2, 3, 4 d) 1, 2, 3, 4

d Not only a legislation but any of the following can be challenged in the courts as violating a Fundamental Right

and hence, can be declared as void. (a) Permanent laws enacted by the Parliament or the state legislatures; (b) Temporary laws like ordinances issued by the president or the state governors; (c) Statutory instruments in the nature of delegated legislation (executive legislation) like order, bye-law, rule, regulation or notification; and (d) Non-legislative sources of law, that is, custom or usage having the force of law. Supreme Court held in the Kesavananda Bharati case (1973) that a Constitutional amendment can be challenged on the ground that it violates a fundamental right that forms a part of the ‘basic structure’ of the Constitution and hence, can be declared as void.

Insights Nov-20 Polity 08/01/2021 2 2) The 42nd Amendment Act of 1976 transferred which of the following subjects to Concurrent List from State

List? 1. Education 2. Protection of wild animals and birds 3. Economic and social planning 4. Administration of justice Select the correct answer code: a) 1, 2 b) 1, 2, 3 c) 1, 2, 4 d) 1, 2, 3, 4

c The 42nd Amendment Act of 1976 : [5-EFoW&MAP]

transferred five subjects to Concurrent List from State List, that is, [EFW&MAP] (a) education, (b) forests, (c) weights and measures, (d) protection of wild animals and birds, and (e) administration of justice; constitution and organisation of all courts except the Supreme Court and the high courts.

Insights Nov-20 Polity 08/01/2021 2 3) Consider the following important parts of the constitution and the subject that they deal with:

1. Part IV: Fundamental Rights 2. Part V: The States 3. Part XI: Relations between the Union and the States Select the correct answer code: a) 1, 2 b) 3 only c) 2, 3 d) 1, 2, 3

b Part III Fundamental Rights Art. 12 to 35

Part IV Directive Principles Art. 36 to 51 Part V The Union Art. 52 to 151 Part VI The States Art. 152 to 237 Part XI Relations between the Union and the States Art. 245 to 263

Insights Nov-20 Polity 08/01/2021 2 4) Consider the following statements

1. The Rajya Sabha represents both the states and union territories of the Indian Union. 2. The representatives of states in the Rajya Sabha are elected by the members of state legislative assemblies and councils. Which of the above statements is/are correct? a) 1 only b) 2 only c) Both 1 and 2 d) Neither 1 nor 2

a The Rajya Sabha is the Upper House and the Lok Sabha is the Lower House. The former represents the states and union territories of the Indian Union, while the latter represents the people of India as a whole.

The representatives of states in the Rajya Sabha are elected by the elected members of state legislative assemblies. The election is held in accordance with the system of proportional representation by means of the single transferable vote.

Insights Nov-20 Polity 08/01/2021 2 5) Consider the following statements

1. A person shall not be a citizen of India if he/she has voluntarily acquired the citizenship of any foreign state. 2. Equal pay for equal work for men and women is one of the element/features of Fundamental Rights. 3. Constituent Assembly’s States Committee (Committee for Negotiating with States) was chaired by Jawaharlal Nehru. Which of the above statements is/are correct? a) 1, 3 b) 1 only c) 2, 3 d) 1, 2, 3

a Equal pay for equal work for men and women is one of the element/features of DPSP.
Insights Nov-20 Polity 08/01/2021 2 6) Which of the following in the Constitution reveal the secular character of the Indian State?

1. Preamble 2. Directive Principles of State Policy 3. Fundamental Rights Select the correct answer code: a) 1, 2 b) 1, 3 c) 2, 3 d) 1, 2, 3

d The term ‘secular’ was added to the Preamble of the Indian Constitution by the 42nd Constitutional Amendment Act of 1976. The Preamble secures to all citizens of India liberty of belief, faith and worship.

The State shall endeavour to secure for all the citizens a Uniform Civil Code (Article 44) is mentioned in DPSP –Part IV. The State shall not deny to any person equality before the law or equal protection of the laws (Article 14). The State shall not discriminate against any citizen on the ground of religion (Article 15). Equality of opportunity for all citizens in matters of public employment (Article 16). All persons are equally entitled to freedom of conscience and the right to freely profess, practice and propagate any religion (Article 25). Every religious denomination or any of its section shall have the right to manage its religious affairs (Article 26). Similarly Articles 27-30 also uphold values of secularism.

Insights Nov-20 Polity 08/01/2021 2 7) Consider the following statements.

1. Supreme Court has declared Right to decent environment including pollution free water as part of Article 21. 2. Parliament may by law provide for the adjudication of any dispute with respect to the use, distribution and control of waters of any inter-state river and river valley. 3. Development of inter-state rivers, water supplies, irrigation and canals come under the Union List. Which of the above statements is/are correct? a) 1, 3 b) 2, 3 c) 1, 2 d) 1, 2, 3

c Right to decent environment including pollution free water and air and protection against hazardous industries is part of Article 21.

While water supplies, irrigation and canals, drainage and embankments and storage fall in the State List, issues like development of inter-state rivers come under the Union List.

Insights Nov-20 Polity 08/01/2021 2 8) Which of the following are the mandatory provisions of the 73rd and 74th Constitutional amendment?

1. Regular direct elections to all local bodies 2. Mandatory reservation of seats for Dalits and Adivasis in every local body, proportionate to their share in the population. 3. Setting up of District Planning Committees that consolidate plans of rural and urban bodies. 4. Setting up of state-level election commission and finance commissions. Select the correct answer code: a) 1, 2, 3 b) 1, 3, 4 c) 2, 3, 4 d) 1, 2, 3, 4

d The key mandatory provisions of the 73rd and 74th amendments, are:

• Regular direct elections to all local bodies. • Setting up of state-level election commission and finance commissions. • Mandatory reservation of seats for Dalits and Adivasis in every local body, proportionate to their share in the population. • 33% reservation for women. • Setting up of District Planning Committees that consolidate plans of rural and urban bodies.

Insights Nov-20 Polity 08/01/2021 2 9) Consider the following statements regarding Question Hour.

1. Always the last hour of a sitting of Lok Sabha is devoted to the Questions and this hour is called the Question Hour. 2. Asking of questions is an inherent and unfettered parliamentary right of members. 3. Sometimes questions may lead to the appointment of a Commission or a Court of Enquiry when matters raised by Members are of wide public importance. Which of the above statements is/are correct? a) 1, 2 b) 1, 3 c) 2, 3 d) 1, 2, 3

c Generally, the first hour of a sitting of Lok Sabha is devoted to the Questions and this hour is called the Question Hour.

Asking of questions is an inherent and unfettered parliamentary right of members. It is during the Question Hour that the members can ask questions on every aspect of administration and Governmental activity. Government policies in national as well as international spheres come into sharp focus as the members try to elicit pertinent information during the Question Hour. It is through questions in the Parliament that the Government remains in touch with the people in as much as members are enabled thereby to ventilate the grievances of the public in matters concerning the administration. Sometimes questions may lead to the appointment of a Commission, a Court of Enquiry or even Legislation when matters raised by Members are grave enough to agitate the public mind and are of wide public importance.

Insights Nov-20 Polity 08/01/2021 2 10) Consider the following statements.

1. A Starred Question is not called for oral answer in the House and no supplementary question can be asked thereon. 2. Short Notice Questions involves matter of urgent public importance. 3. There are no provisions of Questions to Private Members. Which of the above statements is/are incorrect? a) 1 only b) 2 only c) 3 only d) 1, 3

d The questions are of four types:

(i) Starred Questions- A Starred Question is one to which a member desires an oral answer from the Ministerin the House and is required to be distinguished by him/her with an asterisk. Answer to such a question may be followed by supplementary questions by members. (ii) Unstarred Questions- An Unstarred Question is one to which written answer is desired by the member and is deemed to be laid on the Table of the House by Minister. Thus it is not called for oral answer in the House and no supplementary question can be asked thereon. (iii) Short Notice Questions- A member may give a notice of question on a matter of public importance and of urgent character for oral answer at a notice less than 10 days prescribed as the minimum period of notice for asking a question in ordinary course. (iv) Questions to Private Members- A Question may also be addressed to a Private Member, provided that the subject matter of the question relates to some Bill, Resolution or other matter connected with the business of the House for which that Member is responsible.

Insights Nov-20 Polity 08/01/2021 2 11) Consider the following statements regarding the interpretation of Secularism in the Indian context.

1. The state treats all religions equally. 2. The State does not participate in any religious matter whatsoever. 3. All Educational institutions, without any exception, are free to impart religious instruction. 4. The state makes no discrimination on the basis of religion in matters of employment. Which of the above statements is/are incorrect? a) 1, 2, 4 b) 1, 2, 3 c) 2, 3 d) 2, 3, 4

c The Constitution of India stands for a secular state. Hence, it does not uphold any particular religion as the official religion of the Indian State. The Preamble secures to all citizens of India liberty of belief, faith and worship. Hence, the Indian Constitution embodies the positive concept of secularism, i.e., giving equal respect to all religions or protecting all religions equally. But Article 28 says that no religious instruction shall be provided in any educational institution maintained by the State.

Article 15: Prohibition of discrimination on grounds of religion, race, caste, sex or place of birth. The Islamic Central Wakf Council and many Hindu temples of great religious significance are administered and managed by the Indian government.

Insights Nov-20 Polity 08/01/2021 2 12) The National Commission for Women perform which of the following functions?

1. Review the Constitutional and Legal safeguards for women 2. Recommend remedial legislative measures 3. Facilitate redressal of grievances 4. Advise the Government on all policy matters affecting women Select the correct answer code: a) 1, 2, 3 b) 1, 3, 4 c) 2, 3, 4 d) 1, 2, 3, 4

d The National Commission for Women was set up as statutory body in January 1992 under the National Commission for Women Act, to:

• review the Constitutional and Legal safeguards for women; • recommend remedial legislative measures; • facilitate redressal of grievances and • advise the Government on all policy matters affecting women.

Insights Nov-20 Polity 08/01/2021 2 13) Consider the following statements

1. Parliament under Article 368 can amend any part of the Constitution including the Fundamental Rights but without affecting the ‘basic structure’ of the Constitution. 2. Supreme Court is yet to define or clarify as to what constitutes the ‘basic structure’ of the Constitution. Which of the above statements is/are correct? a) 1 only b) 2 only c) Both 1 and 2 d) Neither 1 nor 2

c • The present position is that the Parliament under Article 368 can amend any part of the Constitution including the Fundamental Rights but without affecting the ‘basic structure’ of the Constitution.

• However the Supreme Court is yet to define or clarify as to what constitutes the ‘basic structure’ of the Constitution.

Insights Nov-20 Polity 08/01/2021 2 14) Consider the following statements regarding Fundamental Rights.

1. They limit the authority of the government, and arbitrary laws of the legislature. 2. Fundamental Rights are not absolute and cannot be suspended. 3. They secure vital political rights to the citizens of India. Which of the above statements is/are correct? a) 1, 2 b) 1, 3 c) 2, 3 d) 3 only

b They operate as limitations on the tyranny of the executive implies that they limit the authority of the government, and arbitrary laws of the legislature. They are justiciable in nature, that is, they are enforceable by the courts for their violation.

Rights such as equality to contest for political office, right against discrimination etc show the political and social equality of citizens. Fundamental Rights are not absolute and subject to reasonable restrictions. Further, they are not sacrosanct and can be curtailed or repealed by the Parliament through a constitutional amendment act. They can also be suspended during the operation of a National Emergency except the rights guaranteed by Articles 20 and 21.

Insights Nov-20 Polity 08/01/2021 2 15) Consider the following statements about pro-tem speaker.

1. Speaker of the previous Lok Sabha appoints the pro-tem speaker immediately before the first meeting of the newly elected house. 2. The office of the pro-tem speaker is in co-existence with the speaker of Lok Sabha. Which of the above statements is/are correct? a) 1 only b) 2 only c) Both 1 and 2 d) Neither 1 nor 2

d The speaker of the Lok Sabha/legislative assembly vacates the office immediately before the first meeting of the newly elected house.

Hence President/governor appoints the pro-tem speaker to preside over the sittings of the house. Usually the senior most member is elected as the pro-tem speaker. The president/governor will administer the oath of the office for the pro-tem speaker. When the house elects the new speaker the office of the pro-tem speaker ceases to exist. Hence the office of the pro-tem speaker is a temporary one which will be in existence for few days.

Insights Nov-20 Polity 08/01/2021 2 16) Consider the following statements regarding the Amendment of the Constitution.

1. Article 368 of the constitution deals with the powers of Parliament to amend the constitution and its procedure. 2. Constitution amendment bill can be introduced only in the Parliament and not in the state legislatures. 3. President can either withhold his assent to the Constitution amendment bill or return the bill for reconsideration. Which of the above statements is/are correct? a) 1, 3 b) 1 only c) 1, 2 d) 1, 2, 3

c Article 368 in Part XX of the Constitution deals with the powers of Parliament to amend the Constitution and its procedure.

• An amendment of the Constitution can be initiated only by the introduction of a bill for the purpose in either House of Parliament and not in the state legislatures. • The bill can be introduced either by a minister or by a private member and does not require prior permission of the president. • If the bill seeks to amend the federal provisions of the Constitution, it must also be ratified by the legislatures of half of the states by a simple majority, that is, a majority of the members of the House present and voting. • After duly passed by both the Houses of Parliament and ratified by the state legislatures, where necessary, the bill is presented to the president for assent. • The president must give his assent to the bill. He can neither withhold his assent to the bill nor return the bill for reconsideration of the Parliament.

Insights Nov-20 Polity 08/01/2021 2 17) Consider the following statements.

1. All laws that are inconsistent with or in derogation of any of the fundamental rights shall be void. 2. Ordinances issued by the president or the state Governors can be challenged in the courts on the ground of contravention of any of the Fundamental Rights. 3. A constitutional amendment is not a law and hence cannot be challenged in the Supreme Court. Which of the above statements is/are correct? a) 1, 2 b) 1 only c) 1, 3 d) 1, 2, 3

a Article 13 declares that all laws that are inconsistent with or in derogation of any of the fundamental rights shall be void.

The term ‘law’ in Article 13 has been given a wide connotation so as to include the following: (a) Permanent laws enacted by the Parliament or the state legislatures; (b) Temporary laws like ordinances issued by the president or the state governors; (c) Statutory instruments in the nature of delegated legislation (executive legislation) like order, bye-law, rule, regulation or notification; and (d) Non-legislative sources of law, that is, custom or usage having the force of law. Further, Article 13 declares that a constitutional amendment is not a law and hence cannot be challenged. However, the Supreme Court held in the Kesavananda Bharati case (1973) that a Constitutional amendment can be challenged on the ground that it violates a fundamental right that forms a part of the ‘basic structure’ of the Constitution and hence, can be declared as void.

Insights Nov-20 Polity 08/01/2021 2 18) As per the Constitution of India, the Union executive consists of

1. President of India 2. Vice President of India 3. Union Council of Ministers 4. Attorney General of India Select the correct answer code: a) 1, 2, 3 b) 1, 2, 4 c) 1, 3, 4 d) 1, 2, 3, 4

d Articles 52 to 78 in Part V of the Constitution deal with the Union executive.

The Union executive consists of the President, the Vice-President, the Prime Minister, the council of ministers and the attorney general of India.

Insights Nov-20 Polity 08/01/2021 2 19) Consider the following statements regarding the appointment of Chief Minister.

1. The Constitution contain specific procedure for the selection and appointment of the Chief Minister. 2. According to the Constitution, the Chief Minister should be selected from the Lower House. Which of the above statements is/are incorrect? a) 1 only b) 2 only c) Both 1 and 2 d) Neither 1 nor 2

c The Constitution does not contain any specific procedure for the selection and appointment of the Chief Minister. Article 164 only says that the Chief Minister shall be appointed by the governor.

According to the Constitution, the Chief Minister may be a member of any of the two Houses of a state legislature. Usually Chief Ministers have been selected from the Lower House (legislative assembly), but, on a number of occasions, a member of the Upper House (legislative council) has also been appointed as Chief Minister.

Insights Nov-20 Polity 08/01/2021 2 20) Consider the following statements regarding Leader of the Opposition in either House of the Parliament of India.

1. The position of Leader of the Opposition received statutory recognition through the Salary and Allowances of Leaders of Opposition in Parliament Act, 1977 2. When no party in the Lok Sabha secures required seats to form an opposition party and to designate a Leader of opposition, the matter is then decided by the President of India. Which of the above statements is/are correct? a) 1 only b) 2 only c) Both 1 and 2 d) Neither 1 nor 2

a The Leader of the Opposition is the politician who leads the official opposition in either House of the Parliament of India. To claim the status of "official opposition” in either house a party has to secure 55 seats (10%) of the seats in the Lok Sabha and likewise 25 (10%) of the seats in the Rajya Sabha.

It received statutory recognition through the Salary and Allowances of Leaders of Opposition in ParliamentAct, 1977 which defines the term "Leader of the Opposition" as that member of the Lok Sabha or the Rajya Sabha who, for the time being, is the Leader of that House of the Party in Opposition to the Government having the greatest numerical strength and recognised, as such, by the Chairman of the Rajya Sabha or the Speaker of the Lok Sabha. As per the Salary and Allowances of Leaders of Opposition in Parliament Act, 1977 by which the post has got official and statutory status, the majority required is decided by the heads of the houses, that is speaker and chairman as the case may be.

Insights Nov-20 Geography 08/01/2021 2 1) Consider the following statements regarding Sea floor.

1. Volcanic eruptions are common along the mid-oceanic ridges. 2. The age of the rocks decreases as one moves away from the crest. 3. The ocean crust rocks are much younger than the continental rocks. Which of the above statements is/are correct? a) 1, 2 b) 1, 3 c) 2, 3 d) 1, 2, 3

b Sea floor spreading is verified using these phenomena:

• It was realised that all along the mid-oceanic ridges, volcanic eruptions are common and they bring huge amounts of lava to the surface in this area. The sediments on the ocean floor are unexpectedly very thin. • The age of the rocks increases as one moves away from the crest. • The ocean crust rocks are much younger than the continental rocks. • The sediments on the ocean floor are unexpectedly very thin. • The deep trenches have deep-seated earthquake occurrences while in the mid-oceanic ridge areas, the quake foci have shallow depths.

Insights Nov-20 Geography 08/01/2021 2 2) Consider the following statements regarding Thermosphere.

1. In thermosphere temperature decreases very rapidly with increasing height. 2. Radio waves transmitted from the earth are reflected back to the earth by this layer. 3. The space shuttle and the International Space Station both orbit Earth within the thermosphere. Which of the above statements is/are correct? a) 1, 2 b) 1, 3 c) 2, 3 d) 1, 2, 3

c The thermosphere is a layer of Earth's atmosphere. The thermosphere is directly above the mesosphere and below the exosphere. It extends from about 90 km to between 500 and 1,000 km above our planet.

This layer helps in radio transmission. In fact, radio waves transmitted from the earth are reflected back to the earth by this layer. The space shuttle and the International Space Station both orbit Earth within the thermosphere! This is also where you’ll find low Earth orbit satellites. It is called the thermosphere because temperatures can reach up to 1,500 degrees Celsius. However, despite the high temperatures, the pressure is very low, so satellites don't suffer heat damage.

Insights Nov-20 Geography 08/01/2021 2 3) Which of the following conditions are good for fisheries production & catch?

1. Coastal upwelling of nutrient rich colder water 2. Abundance of phytoplankton 3. Meeting of warm and cold currents 4. Shallow continental shelves in cold regions Select the correct answer code: a) 1, 2, 3 b) 1, 3, 4 c) 2, 3, 4 d) 1, 2, 3, 4

d The following conditions are good for fisheries production & catch:

• Coastal upwelling of nutrient rich colder water • Abundance of phytoplankton • Meeting of warm and cold currents • Shallow continental shelves in cold regions

Insights Nov-20 Geography 08/01/2021 2 4) The speed of tsunami waves in Ocean largely depends on

a) Ocean depth b) Distance from mid-oceanic ridges c) Distance from the source of the wave d) Density of water

a Tsunamis are giant waves caused by earthquakes or volcanic eruptions under the sea. Out in the depths of the ocean, tsunami waves do not dramatically increase in height. But as the waves travel inland, they build up to higher and higher heights as the depth of the ocean decreases.

It depends on Ocean depth rather than the distance from the source of the wave. Tsunami waves may travel as fast as jet planes over deep waters, only slowing down when reaching shallow waters.

Insights Nov-20 Geography 08/01/2021 2 5) Which of the following are the conditions for the emergence of a tropical cyclone?

1. Large and continuous supply of warm and moist air. 2. Strong Coriolis force 3. Strong vertical wind 4. Unstable condition through the Troposphere Select the correct answer code: a) 1, 2, 3 b) 1, 2, 4 c) 2, 3, 4 d) 1, 2, 3, 4

b Some initial conditions for the emergence of a tropical cyclone are:

(i) Large and continuous supply of warm and moist air that can release enormous latent heat. (ii) Strong Coriolis force that can prevent filling of low pressure at the centre (absence of Coriolis force near the equator prohibits the formation of tropical cyclone between 0°-5° latitude). (iii) Unstable condition through the troposphere that creates local disturbances around which a cyclone develops. (iv) Finally, absence of strong vertical wind wedge, which disturbs the vertical transport of latent heat.

Insights Nov-20 Geography 08/01/2021 2 6) Consider the following statements regarding White dwarf.

1. White dwarfs can tell us about the age of the Universe. 2. The heaviest stars at the end of their lives turn into white dwarfs. Which of the above statements is/are correct? a) 1 only b) 2 only c) Both 1 and 2 d) Neither 1 nor 2

a Where a star ends up at the end of its life depends on the mass it was born with. Stars that have a lot of mass may end their lives as black holes or neutron stars. A low or medium mass star (with mass less than about 8 times the mass of our Sun) will become a white dwarf. A typical white dwarf is about as massive as the Sun, yet only slightly bigger than the Earth. This makes white dwarfs one of the densest forms of matter, surpassed only by neutron stars and black holes.

White dwarfs can tell us about the age of the Universe.

Insights Nov-20 Geography 08/01/2021 2 7) Which of these drainage patterns is formed when streams flow in different directions from a central peak or dome like structure?

a) Trellis b) Radial c) Rectangular d) Dendritic

b The Dendritic pattern develops where the river channel follows the slope of the terrain. The stream with its tributaries resemblesthe braches of a tree, thusthe name dendritic.

A river Joined by its tributaries, at approximately right angles, develops a trellis pattern. A trellis drainage pattern develops where hard and soft rocks exist parallel to each other. A rectangular drainage pattern develops on a strongly jointed rocky terrain. The radial pattern develops when streamsflow in different directionsfrom a central peak or dome like Structure.

Insights Nov-20 Geography 08/01/2021 2 8) Which of the following inhibit the development of cyclones in the North Indian Ocean?

1. Weak La Nina conditions along the equatorial Pacific Ocean. 2. Influence of Madden Julian Oscillation (MJO). 3. Vertical wind shear Select the correct answer code: a) 1, 2 b) 1, 3 c) 2, 3 d) 1, 2, 3

d October to December period is among the favourable months for the development of cyclones in the Bay of Bengal and the Arabian Sea. This year, however, October passed without witnessing a cyclonic storm.

Why were there no cyclone developments this year? • Ocean disturbances enter the Bay of Bengal from the South China sea side and head towards the Indian coast. This year, however, there was no system which intensified to form a cyclone. • Another reason is the weak La Nina conditions along the equatorial Pacific Ocean. • There was also the influence of Madden Julian Oscillation (MJO). MJO is kind of an eastward-moving cyclic weather event along the tropics that influences rainfall, winds, sea surface temperatures and cloud cover. They have a 30 to 60-day cycle. • Also, in November, the vertical wind shear created due to significant wind speed difference observed between higher and lower atmospheric level prevented the low-pressure systems and depression from strengthening into a cyclone.

Insights Nov-20 Geography 08/01/2021 2 9) Which one of the following places is located at the confluence of Alaknanda and Bhagirathi

a) Vishnuprayag b) Devaprayag c) Rudrapravag d) Karnaprayag

b Devaprayag – Confluence of Alaknanda and Bhagirathi

Vishnuprayag – Confluence of Alaknanda and Dhauliganga Rudraprayag – Confluence of Alaknanda and Mandakini Karanprayag – Confluence of Alaknanda and Pindar

Insights Nov-20 Geography 08/01/2021 2 10) Which of the following regions of the world is/are seismically active?

1. Alpine-Himalayan belt 2. Mid-Atlantic Ridges 3. Central Africa Select the correct answer code: a) 1, 2 b) 1, 2, 3 c) 1, 3 d) 1 only

a Seismic belt, narrow geographic zone on the Earth’s surface along which most earthquake activity occurs.

The two major seismic belts are the Circum-Pacific Belt, which surrounds the Pacific Ocean, and the Alpide Belt or Alpine-Himalayan orogenic belt, which stretches from the Azores through the Mediterranean and Middle East to the Himalayas and Indonesia, where it joins the Circum-Pacific Belt. A purely oceanic seismic belt lies along the mid-Atlantic ridge. Seismicity of West and Central Africa is low to moderate, as is normal with stable continental interiors.

Insights Nov-20 Geography 08/01/2021 2 11) Most of the world’s deserts are located in the western margins of continents in the subtropics. This is because?

1. The tropical easterly winds become dry by the time they reach the western margins of the continents. 2. Presence of warm ocean currents along the western shores of continents. Select the correct answer code: a) 1 only b) 2 only c) Both 1 and 2 d) Neither 1 nor 2

a The prevailing winds in the tropics are tropical easterly winds. The tropical easterly winds become dry by the time they reach the western margins of the continents and so they bring no rainfall.

Thus, the region becomes devoid of moisture which causes dry conditions leading to formation of deserts. The presence of cold ocean currents along the western shores of continents leads to the development of high pressure over the water surface. This high pressure leads to subsidence of air hinders cloud formation.

Insights Nov-20 Geography 08/01/2021 2 12) Which of the following factors affect the climate of a place?

1. Relief of a place 2. Altitude 3. Distance from Sea 4. Circulation of ocean currents 5. Wind flow Select the correct answer code: a) 1, 2, 3, 4 b) 2, 3, 4, 5 c) 2, 3, 4 d) 1, 2, 3, 4, 5

d Climate is affected by a multitude of factors like relief of a place, its altitude, distance from Sea, circulation of ocean currents and wind flow etc.
Insights Nov-20 Geography 08/01/2021 2 13) Conditions favourable for Delta Formation are

1. No strong current running at right angle to the river mouth. 2. Presence of Continental shelf. 3. Presence of large lake in the way of the river. 4. The coast should be sheltered preferably without tides. Which of the above statements is/are correct? a) 1, 2, 3 b) 1, 3, 4 c) 1, 2, 4 d) 1, 2, 3, 4

c Conditions favourable for the formation of deltas

• Lateral erosion and vertical erosion in the upper course of the river to provide extensive sediments to be eventually deposited as deltas. • The coast should be sheltered and preferably without tides. • The sea should be shallow near the delta region or else the load will disappear in the deep water. • There should not be the presence of large lakes in the river course to filter off the sediments. • The currents should be weak and no strong current should run at right angles to the river mouth as it can wash the sediment away.

Insights Nov-20 Geography 08/01/2021 2 14) Consider the following statements regarding Dharwar System of rocks.

1. It is the oldest system of rocks in India. 2. It contains metamorphic sedimentary rocks. 3. It hosts gold mines. Which of the above statements is/are correct? a) 1, 2 b) 1, 2, 3 c) 1, 3 d) 2, 3

d Dharwar system is later than the Archean system. The Dharwar period of rock formation has been largely fixed from 2500 million years ago to 1800 million years ago. Dharwar Rock System is special because it is the first metamorphic sedimentary rocks in India.

They are named Dharwar system because they were first studied in Dharwar region of Karnataka. But they are also found in Aravallis, Tamil Nadu, Chotanagpur plateau, Meghalaya, Delhi, and the Himalayas region.

Insights Nov-20 Geography 08/01/2021 2 15) With reference to Aurora or Northern lights, consider the following statements:

1. Auroras are the glowing lights at high latitude. 2. Auroras can be normally observed in stratosphere. 3. Change in the atmospheric temperature cause Auroras. Which of the above statement is/are correct? a) 1, 3 b) 1 only c) 2, 3 d) 1, 2, 3

b An aurora, sometimes referred to as polar lights, northern lights (aurora borealis) or southern lights (aurora australis), is a natural light display in the Earth’s sky, predominantly seen in the high latitude regions (around the Arctic and Antarctic).

Auroras are produced when the magnetosphere is sufficiently disturbed by the solar wind. It usually occurs in upper atmosphere (thermosphere/exosphere) due to Earth’s magnetic field.

Insights Nov-20 Geography 08/01/2021 2 16) Consider the following statements.

1. Earth’s crust is the thinnest of all the major layers of earth. 2. The crust is thicker on the continent than on the ocean floor. 3. The oceanic crust mainly consists of manganese and iron. Which of the above statements is/are correct? a) 1, 2 b) 1, 3 c) 2, 3 d) 1, 2, 3

a The uppermost layer over the earth’s surface is called the crust. It is the thinnest of all the layers. It is about 35 km. on the continental masses and only 5 km on the ocean floors.

The main mineral constituents of the continental mass are silica and alumina. It is thus called sial (si-silica and al-alumina). The oceanic crust mainly consists of silica and magnesium; it is therefore called sima (si-silica and mamagnesium).

Insights Nov-20 Geography 08/01/2021 2 17) Hot Spots within the earth help produce Geothermal Energy. What are these ‘Hot Spots’?

a) Areas of intense pressure inside the mantle b) Region in crust where hot molten rocks are trapped c) Regions of high volcanism on earth’s surface d) Areas of intense magnetic activity within the upper mantle

b Due to geological changes, molten rocks formed in the deeper hot regions of earth’s crust are pushed upward and trapped in certain regions called ‘hot spots’.

When underground water comes in contact with the hot spot, steam is generated. Sometimes hot water from that region finds outlets at the surface. Such outlets are known as hot springs.

Insights Nov-20 Geography 08/01/2021 2 18) Consider the following statements regarding Monsson rainfall.

1. Half of India’s rainfall is received from Northeast monsoon. 2. Northeast monsoon in India is almost confined to the Southern peninsula. 3. Maldives, Sri Lanka and Myanmar also record rainfall during Northeast monsoon. Which of the above statements is/are correct? a) 1, 2 b) 1, 3 c) 2, 3 d) 2 only

c India receives rainfall during two seasons. About 75 per cent of the country’s annual rainfall is received from the Southwest monsoon between June and September. The Northeast monsoon, on the other hand, occurs during October to December, and is a comparatively small-scale monsoon, which is confined to the Southern peninsula.

Also called the winter monsoon, the rainfall associated with the Northeast monsoon is important for Tamil Nadu, Puducherry, Karaikal, Yanam, coastal Andhra Pradesh, Kerala, north interior Karnataka, Mahe and Lakshadweep. Some South Asian countries such as Maldives, Sri Lanka and Myanmar, too, record rainfall during October to December. Tamil Nadu records about 48 per cent (447.4 mm) of its annual rainfall (943.7 mm) during these months, making it the key factor for undertaking agricultural activities and reservoir management in the state.

Insights Nov-20 Geography 08/01/2021 2 19) Consider the following statements.

1. Almost all the world deserts are confined within the 30 to 45 degrees parallels of latitude north and south of the equator. 2. Deserts lie in the trade wind belt on the western parts of the continents where Trade winds are offshore. Which of the above statements is/are correct? a) 1 only b) 2 only c) Both 1 and 2 d) Neither 1 nor 2

b The pattern that can be identified in the location of the world’s deserts is that almost all the deserts are confined within 15 to 30 degrees parallels of latitude north and south of the equator.

They lie in the trade wind belt on the western parts of the continents where Trade winds are off-shore. They are bathed by cold currents which produced a ‘desiccating effect’ so that moisture is not easily condensed into precipitation. Dryness or aridity is the key note. Such deserts are tropical hot deserts or ‘Trade wind deserts’. They include the Great Sahara Desert; Arabian, Iranian and Thar Deserts; Kalahari, Namib, and Atacama Deserts; the Great Australian Deserts and the deserts of the south-west U.S.A, and northern Mexico. In the continental interiors of the mid-latitudes, the deserts such as the Gobi and Turkestan are characterised by extremes of temperatures.

Insights Nov-20 Geography 08/01/2021 2 20) Warm moist summer, cool dry winter, great annual temperature range, rainfall throughout the year are the characteristic feature of which type of climate?

a) Hot, Wet Equatorial climate b) Sudan climate c) British type climate d) China type climate

d This type of climate is found on the eastern margins of continents in warm temperate latitudes, just outside the tropics. It has comparatively more rainfall than the Mediterranean climate in the same latitudes, coming mainly in the summer. It is, in fact, the climate of most parts of China –a modified form of monsoonal climate. It is thus also called the Temperate Monsoon or China Type of climate.

Climatic Condition: Warm moist summer and a cool, dry winter. The mean monthly temperature varies between 5°C and 25°C and is strongly modified by maritime influence. The relative humidity is a little high in mid-summer. Rainfall is more than moderate, anything from 25 inches to 60 inches. Another important feature is the fairly uniform distribution of rainfall throughout the year. There is rain every month, except in the interior of central China, where there is a distinct dry season.

Insights Nov-20 Economics 08/01/2021 2 1) Gross Domestic Product of India include

a) Value of goods and services provided by NRIs b) Gross capital investments made in India during a financial year c) Both a and b d) Neither a nor b

b GDP is the value of the final goods and services produced within a country within an year.

Indian nationals living abroad do not contribute to India’s GDP. They do however contribute to India’s GNP. But, the gross capital investments made during a financial year directly accounts as a part of GDP since it creates infrastructure which has an economic value.

Insights Nov-20 Economics 08/01/2021 2 2) Hard currency is usually preferred because

1. It is seen as politically and economically stable. 2. It is widely accepted around the world as a form of payment for goods and services. 3. It is not likely to depreciate or appreciate suddenly. Select the correct answer code: a) 1, 2 b) 1, 3 c) 1, 2, 3 d) 2, 3

c Hard currency refers to money that is issued by a nation that is seen as politically and economically stable. Hard currencies are widely accepted around the world as a form of payment for goods and services and may be preferred over the domestic currency.

A hard currency is expected to remain relatively stable through a short period of time, and to be highly liquid in the forex or foreign exchange (FX) market. The most tradable currencies in the world are the U.S. dollar (USD), European euro (EUR), Japanese yen (JPY), British pound (GBP), Swiss franc (CHF), Canadian dollar (CAD) and the Australian dollar (AUD). All of these currencies have the confidence of international investors and businesses because they are not generally prone to dramatic depreciation or appreciation.

Insights Nov-20 Economics 08/01/2021 2 3) Deposit insurance and credit guarantee corporation (DICGC) a wholly owned subsidiary of Reserve Bank of India insures which of the following deposits?

1. Local Area Banks 2. Payment Banks 3. Small Finance Banks 4. Inter-bank deposits 5. Regional Rural Banks Select the correct answer code: a) 1, 2, 3, 4 b) 1, 2, 3, 5 c) 1, 3, 5 d) 1, 2, 3, 4, 5

b Insured Banks

Public Sector Banks Private Sector Banks Foreign Banks Small Finance Banks Payment Banks Regional Rural Banks Local Area Banks State Co-operative banks District Central Co-op banks Urban Co-op banks The DICGC insures all deposits such as savings, fixed, current, recurring, etc. deposits except the following types of deposits • Deposits of foreign Governments; • Deposits of Central/State Governments; • Inter-bank deposits; • Deposits of the State Land Development Banks with the State co-operative bank; • Any amount due on account of and deposit received outside India • Any amount, which has been specifically exempted by the corporation with the previous approval of Reserve Bank of India

Insights Nov-20 Economics 08/01/2021 2 4) Consider the following statements regarding Consumer Price Index — Industrial Workers (CPI-IW).

1. CPI-IW is primarily used to regulate the dearness allowance of government employees and the workers in the industrial sectors. 2. It is also used in the determination and revision of minimum wages in scheduled employments. 3. The base year for CPI-IW is 2016. Which of the above statements is/are correct? a) 1, 2 b) 1, 3 c) 2, 3 d) 1, 2, 3

d The government recently launched a new series for Consumer Price Index — Industrial Workers (CPI-IW) changing the base year to 2016 from 2001.

The CPI-IW is primarily used to regulate the dearness allowance of government employees and the workers in the industrial sectors. It is also used in the determination and revision of minimum wages in scheduled employments. The new series is more representative in character and reflects the latest consumption pattern of the industrial workers. Labour Bureau, a unit of the labour ministry, has been compiling and releasing CPI-IW numbers since its inception in 1946. The base year for inflation gauges is typically changed from time to time to capture the complex churnings in the economy.

Insights Nov-20 Economics 08/01/2021 2 5) Consider the following statements regarding Yield curve.

1. The yield curve is a graph showing the relationship between interest rates earned on lending money for different durations. 2. The yield curve turns positive when near-term Treasurys yield more than their long-term counterparts. Which of the above statements is/are incorrect? a) 1 only b) 2 only c) Both d) None

b The yield curve is a graph showing the relationship between interest rates earned on lending money for different durations.

Normally, someone who lent to the government or a corporation for one year (by buying a one-year government or corporate bond) would expect to get a lower interest rate than someone who lent for five or ten years, making the yield curve upward-sloping. Recently In US the ten-year bond rate had fallen to the point at which the ten-year rate was below the two-year rate – so the yield curve was inverted. The yield curve turns negative when near-term Treasurys yield more than their long-term counterparts.

Insights Nov-20 Economics 08/01/2021 2 6) Consider the following statements regarding First Five-year Plan (FYP) in India.

1. The First Five-year Plan mainly focused on development of the primary sector. 2. The First Five-year Plan was followed by the Bombay Plan which became the basis for the Second Five Year Plan. Which of the above statements is/are correct? a) 1 only b) 2 only c) Both 1 and 2 d) Neither 1 nor 2

a The First Five-year Plan was launched in 1951 which mainly focused in development of the primary sector. The First Five-Year Plan was based on the Harrod–Domar model with few modifications.

The Bombay Plan is the name commonly given to a World War II-era set of proposals for the development of the post-independence economy of India. The plan was published in 1944/1945. Second Plan (1956–1961).

Insights Nov-20 Economics 08/01/2021 2 7) Printing currency is usually the last resort for the government in managing its deficit. This is because

1. Government expenditure using this new money boosts incomes and raises private demand in the economy, thus leading to inflation. 2. Excess supply of rupee will cause its depreciation. Select the correct answer code: a) 1 only b) 2 only c) Both 1 and 2 d) Neither 1 nor 2

c Government expenditure using this new money boosts incomes and raises private demand in the economy.

Thus, it fuels inflation. Easy monetary policy and high inflation are two of the leading causes of currency depreciation.

Insights Nov-20 Economics 08/01/2021 2 8) If the Cash Reserve Ratio (CRR) is increased, it may lead to which of the following?

1. Lesser availability of loanable funds with the banks 2. Immediate foreign institutional investment flows in the economy 3. Reduction in fiscal deficit of the government Select the correct answer code: a) 1 only b) 1, 2 c) 1, 3 d) None of the above

a CRR is one of the major weapons in the RBI’s arsenal that allows it to maintain a desired level of inflation, control the money supply, and also liquidity in the economy. The lower the CRR, the higher liquidity with the banks, which in turn goes into investment and lending and vice-versa. Higher CRR can also negatively impact the economy as lesser availability of loanable funds, in turn, slows down investment. It thereby reduces the supply of money in the economy.

Fiscal deficit depends on government’s receipts and expenditures. CRR does not have a direct bearing on fiscal deficit.

Insights Nov-20 Economics 08/01/2021 2 9) The Reserve Bank of India monitors which of following indicators in the banking sector?

1. Cash flow with banks 2. Credit activities of banks 3. Spurious transactions Select the correct answer code: a) 1, 3 b) 2, 3 c) 1, 2 d) 1, 2, 3

d Banks maintain a minimum cash balance out of the deposits they receive. The RBI monitors the banks in actually maintaining cash balance.

Similarly, the RBI sees that the banks give loans not just to profit-making businesses and traders but also to small cultivators, small scale industries, small borrowers etc. Banks need to report spurious transactions to Financial Intelligence Unit. The norms are regulated by RBI.

Insights Nov-20 Economics 08/01/2021 2 10) Which among the following is/are likely to result in current account surplus of Balance of Payments (BoP)?

1. Steep fall in global crude oil prices 2. Increase in the remittances received from abroad. 3. External commercial borrowing Select the correct answer code: a) 3 only b) 1, 2 c) 2, 3 d) 1, 2, 3

b External Commercial borrowing is a part of Capital account.
Insights Nov-20 Economics 08/01/2021 2 11) In India, which of the following are considered as Alternative Investment Funds (AIFs).

1. Social Venture Capital Funds 2. Real estate Funds 3. Hedge Funds 4. Private Equity (PE) Funds Select the correct answer code: a) 1, 2, 3 b) 1, 3, 4 c) 2, 3, 4 d) 1, 2, 3, 4

d Alternative Investment Fund (AIF) refers to an investment which differs from

conventional investment avenues such as stocks, debt securities, etc. Alternative Investment Fund is described under Regulation 2(1)(b) of the Regulation Act, 2012 of Securities and Exchange Board of India (SEBI). AIF can be established in the form of a company or a corporate body or a trust or a Limited Liability Partnership (LLP). Generally, high net worth individuals and institutions invest in Alternative Investment Funds as it requires a high investment amount, unlike Mutual Funds.

Insights Nov-20 Economics 08/01/2021 2 12) Consider the following statements regarding GDP deflator.

1. The GDP deflator is basically a measure of inflation. 2. It helps show the extent to which the increase in gross domestic product has happened on account of higher prices rather than increase in output. 3. It covers only those goods and services directly consumed by households. Which of the above statements is/are correct? a) 1 only b) 1, 2 c) 1, 3 d) 1, 2, 3

b The GDP deflator, also called implicit price deflator, is a measure of inflation. It is the ratio of the value of goods and services an economy produces in a particular year at current prices to that of prices that prevailed during the base year.

This ratio helps show the extent to which the increase in gross domestic product has happened on account of higher prices rather than increase in output. Since the deflator covers the entire range of goods and services produced in the economy — as against the limited commodity baskets for the wholesale or consumer price indices — it is seen as a more comprehensive measure of inflation. Changes in consumption patterns or introduction of goods and services are automatically reflected in the GDP deflator. This allows the GDP deflator to absorb changes to an economy’s consumption or investment patterns. Often, the trends of the GDP deflator will be similar to that of the CPI.

Insights Nov-20 Economics 08/01/2021 2 13) Consider the following statements regarding Currency Deposit Ratio (CDR).

1. CDR is calculated only with respect to the Term deposits. 2. It reflects people’s preference for liquidity. 3. It generally increases during the festive season. Which of the above statements is/are correct? a) 1, 2 b) 2, 3 c) 1, 3 d) 1, 2, 3

b The currency deposit ratio (cdr) is the ratio of money held by the public in currency to that they hold in bankdeposits. cdr = CU/DD.

If a person gets Re 1 she will put Rs 1/(1 + cdr) in her bank account and keep Rs cdr/(1 +cdr) in cash. It reflects people’s preference for liquidity. It is a purely behavioural parameter which depends, among other things, on the seasonal pattern of expenditure. For example, cdr increases during the festive season as people convert deposits to cash balance for meeting extra expenditure during such periods.

Insights Nov-20 Economics 08/01/2021 2 14) Consider the following statements regarding revenue receipts and capital receipts of the government.

1. Capital receipts are always debt creating unlike revenue receipts. 2. Capital receipts are non-redeemable unlike revenue receipts. Which of the above statements is/are incorrect? a) 1 only b) 2 only c) Both 1 and 2 d) Neither 1 nor 2

c The main difference between revenue receipts and capital receipts is that in the case of revenue receipts, government is under no future obligation to return the amount, i.e., they are non-redeemable. But in case of capital receipts which are borrowings, government is under obligation to return the amount along with Interest.

Capital receipts may be debt creating or non-debt creating. Examples of debt creating receipts are—Net borrowing by government at home, loans received from foreign governments, borrowing from RBI. Examples of non-debt capital receipts are—Recovery of loans, proceeds from sale of public enterprises (i.e., disinvestment), etc. These do not give rise to debt.

Insights Nov-20 Economics 08/01/2021 2 15) Consider the following statements regarding Cash Reserve Ratio (CRR).

1. It is the fraction of the total Net Demand and Time Liabilities (NDTL) of a Scheduled Commercial Bank in India that has to maintain as cash deposit with the RBI. 2. It applies uniformly to all banks in the country irrespective of an individual bank’s financial situation or size. 3. Banks are paid interest for parking the required cash under CRR. Which of the above statements is/are correct? a) 1 only b) 1, 2 c) 1, 3 d) 1, 2, 3

b Cash Reserve Ratio refers to the fraction of the total Net Demand and Time Liabilities (NDTL) of a Scheduled Commercial Bank held in India, that it has to maintain as cash deposit with the Reserve Bank of India (RBI).

The requirement applies uniformly to all banks in the country irrespective of an individual bank’s financial situation or size. In contrast, certain countries e.g. China stipulates separate reserve requirements for ‘large’ and ‘small’ banks. As per the RBI Act 1934, all Scheduled Commercial Banks (that includes public and private sector banks, foreign banks, regional rural banks and co-operative banks) are required to maintain a cash balance on average with the RBI on a fortnightly basis to cater to the CRR requirement. Presently, banks are not paid any interest on behalf of the RBI for parking the required cash. If a bank fails to meet its required reserve requirements, the RBI is empowered to impose a penalty by charging a penal interest rate.

Insights Nov-20 Economics 08/01/2021 2 16) The Agreement on Trade-Related Investment Measures (TRIMs), is related to

a) Current Account Convertibility b) Domestic regulations a country applies to foreign investors c) Most-favoured Nation principle d) Duty free International market access to developing countries

b The Agreement on Trade-Related Investment Measures (TRIMs) are rules that are applicable to the domestic regulations a country applies to foreign investors, often as part of an industrial policy. The agreement, concluded in 1994, was negotiated under the WTO's predecessor, the General Agreement on Tariffs and Trade (GATT), and came into force in 1995. The agreement was agreed upon by all members of the World Trade Organization.

Trade-Related Investment Measures is one of the four principal legal agreements of the WTO trade treaty. TRIMs are rules that restrict preference of domestic firms and thereby enable international firms to operate more easily within foreign markets. Policies such as local content requirements and trade balancing rules that have traditionally been used to both promote the interests of domestic industries and combat restrictive business practices are now banned.

Insights Nov-20 Economics 08/01/2021 2 17) Consider the following statements regarding Special Drawing Right (SDR).

1. The SDR is an international reserve asset, created by the IMF to supplement its member countries’ official reserves. 2. The value of the SDR is based on a basket of currencies of developed countries. Which of the above statements is/are correct? a) 1 only b) 2 only c) Both 1 and 2 d) Neither 1 nor 2

a The SDR is an international reserve asset, created by the IMF in 1969 to supplement its member countries’ official reserves. The value of the SDR is based on a basket of five currencies—the U.S. dollar, the euro, the Chinese renminbi, the Japanese yen, and the British pound sterling.
Insights Nov-20 Economics 08/01/2021 2 18) Which of the following may lead to Inflation

1. A reduction in the total productive capacity of the economy even as more and more people are employed 2. Oversupply of goods in the economy. Select the correct answer code: a) 1 only b) 2 only c) Both 1 and 2 d) Neither 1 nor 2

a Inflation is basically too much money chasing too few goods, or excess demand chasing limited supply. If income rises faster, demand for goods and services will also rise. On the other hand, if the economy is unable to satisfy the increased demand, for e.g. due to poor infrastructure, lack of production etc, the higher income will spiral the prices upwards and lead to high inflation.

Excess supply is likely to bring prices down.

Insights Nov-20 Economics 08/01/2021 2 19) A receipt is a capital receipt if it satisfies which of the following conditions?

1. The receipts must create a liability for the government. 2. The receipts must cause a decrease in the Government assets. Select the correct answer code: a) 1 only b) 2 only c) Both 1 and 2 d) Neither 1 nor 2

c A receipt is a capital receipt if it satisfies any one of the two conditions:

(i) The receipts must create a liability for the government. For example, Borrowings are capital receipts as they lead to an increase in the liability of the government. However, tax received is not a capital receipt as it does not result in creation of any liability. (ii) The receipts must cause a decrease in the assets. For example, receipts from sale of shares of public enterprise is a capital receipt as it leads to reduction in assets of the government.

Insights Nov-20 Economics 08/01/2021 2 20) With reference to World Trade Organization (WTO) affairs, which of the following best describes Special Safeguard Mechanism (SSM)?

a) Mechanism provided for the developed countries to raise tariffs to deal with cheap goods from the developing nations. b) Mechanism to restrict the imports of disease infected agricultural products from other countries. c) Mechanism to curb the imports of goods whose manufacturing process has violated International labour norms d) A tool that will allow developing countries to raise tariffs temporarily to deal with import surges or price falls.

d Special Safeguard Mechanism (SSM)

It is a tool that will allow developing countries to raise tariffs temporarily to deal with import surges or price falls.

Insights Nov-20 Arts 12/01/2021 2 1) The culture of wearing face masks for performing art can be found in

1. Indus Valley civilization 2. Natya Shashtra 3. Buddhist monasteries of Ladakh Select the correct answer code: a) 1, 2 b) 1, 3 c) 2, 3 d) 1, 2, 3

d From the delicate pastel coloured masks and shimmering head-dresses worn by Chhau dancers to the demon dance masks of the Buddhist monasteries of Ladakh to the inexpensive animal masks of papier-mâché available in our cities, India has a vast and ancient tradition of masks and make-up for rituals and theatre.

Excavations have revealed small hollow masks dating back to the Indus Valley Civilisation. In fact in Bihar a terracotta mask of the fourth century has also been excavated. The Natya Shastra speaks of masks and their use in theatre. Here it is mentioned that masks can be made of ground paddy husks applied to cloth.

Insights Nov-20 Arts 12/01/2021 2 2) Which of the following temple complexes are the examples of Nagara school of architecture?

1. Lingaraja Temple 2. Sun temple of Kornak 3. Shore Temple Select the correct answer code: a) 1, 2 b) 1, 3 c) 2, 3 d) 1, 2, 3

a Shore Temple is located near Chennai in Tamil Nadu.

Shore Temple is an example for Dravidian architecture.

Insights Nov-20 Arts 12/01/2021 2 3) Consider the following statements regarding the development of architecture during Mughal period.

1. Use of red sandstone is the chief feature of architecture during Akbar's time. 2. Humayun’s tomb is an example for Charbagh style. Which of the above statements is/are correct? a) 1 only b) 2 only c) Both 1 and 2 d) Neither 1 nor 2

c The chief feature of the architecture of Akbar's time was the use of red sandstone.

Humayun’s garden-tomb is an example of the charbagh (a four quadrant garden with the four rivers of Quranic paradise represented), with pools joined by channels.

Insights Nov-20 Arts 12/01/2021 2 4) Diwan-i-Khas in Red Fort is a

a) A gard b) Hall of Private Audiences c) Palace of Colour d) Pearl Mosque

b The Diwan-i-Khas or Hall of Private Audiences, was a chamber in the Red Fort of Delhi built in 1571 as a location for receptions. It was the location where the Mughal Emperor Akbar received courtiers and state guests. It was also known as the Shah Mahal.
Insights Nov-20 Arts 12/01/2021 2 5) The Senia tradition is

a) Musical tradition that invoked the name of Tansen, the celebrated musician of Akbar's Court b) Dance drama written by Keshab Chandra Sen, a social reformer and philosopher c) Musical tradition that emerged from the Mughal Sena d) None of the above

a The word "Senia" is related to Tansen. The word "Gharana" implies a style of music.

The followers of Tansen’s school of music are widely known as the followers of "Senia gharana" (i.e. "Seniya" style/school of music). Tansen was a vocalist but this tradition has also produced great Sitar Maestros. The "Senia" style of Sitar playing started with the legendary great master of Sitar, Ustad Maseet Sen from the family of Tansen, the originator of "Maseetkhani" style.

Insights Nov-20 Arts 12/01/2021 2 6) Which one of the following animals was not depicted on the seals and terracotta art of the Harappan culture?

a) Camel b) Rhinoceros c) Tiger d) Elephant

a Cow, camel, horse and lion were not depicted on seals. Unicorn (bull) was the animal most commonly represented on the seals.
Insights Nov-20 Arts 12/01/2021 2 7) Ashokan edicts have been written in which of the following languages?

1. Prakrit 2. Greek 3. Kharoshti 4. Aramaic Select the correct answer code: a) 1, 2, 3 b) 1, 3, 4 c) 1, 2, 4 d) 1, 2, 3, 4

c Ashokan inscriptions comprise of major rock edicts, minor rock edicts and pillar edicts. Different languages like Prakrit, Greek, Aramaic and different scripts like Brahmi, Kharoshti were used. The most widely used script was Brahmi. Kharoshti was widely used in the inscriptions found in modern-day Pakistan. Greek and Aramaic was used in inscriptions found in Afghanistan - for example, the bilingual Greek-Aramaic inscription found in Kandahar.
Insights Nov-20 Arts 12/01/2021 2 8) Consider the following statements.

1. The temple is the largest of the rock-cut Hindu temples at the Ellora Caves. 2. Most of the excavation of the temple is generally attributed to the eighth century Rashtrakuta king Krishna I. The above statements refer to a) Brihadishvara Temple b) Kailasha Temple c) Lingaraja Temple d) Vitthala temple

b The Kailasha or Kailashanatha temple is the largest of the rock-cut Hindu temples at the Ellora Caves, Maharashtra. A megalith carved from a rock cliff face, it is considered one of the most remarkable cave temples in the world because of its size, architecture and sculptural treatment, and "the climax of the rock-cut phase of Indian architecture".

The Kailasa temple (Cave 16) is the largest of the 34 Buddhist, Jain and Hindu cave temples and monasteries known collectively as the Ellora Caves. Most of the excavation of the temple is generally attributed to the eighth century Rashtrakuta king Krishna I, with some elements completed later. The temple architecture shows traces of Pallava and Chalukya styles built by Kannadiga kings.

Insights Nov-20 Arts 12/01/2021 2 9) What is common to these personalities - Tyagaraja, Muthuswami Dikshitar and Syama Sastri?

a) Theatre Presentation b) Carnatic music c) Temple Architecture d) Tamil Poetry and Prose

b The birth of the Musical Trinity - Tyagaraja, Muthuswami Dikshitar and Syama Sastri - at Tiruvarur between the years 1750 to 1850 A.D. ushered in an era of dynamic development in Carnatic music.

The Trinity were not only contemporaries among themselves but, also contemporaries of great composers of Western Music, as Beethoven, Mozart, Wagner and Haydn. It was the 'Golden Age' of music throughout the world. Carnatic music reached its pinnacle of artistic excellence during this period.

Insights Nov-20 Arts 12/01/2021 2 10) With reference to the history of philosophical thought in India, consider the following statements regarding Vaisheshika School

1. It believes in the physicality of the Universe and is considered to be the realistic and objective philosophy that governs the universe. 2. It developed the atomic theory, i.e. all material objects are made of atoms. Which of the above statements is/are correct? a) 1 only b) 2 only c) Both 1 and 2 d) Neither 1 nor 2

c The Vaisheshika school believes in the physicality of the Universe and is considered to be the realistic and objective philosophy that governs the universe. The Kanada who also wrote the basic text governing the Vaisheshika philosophy is often considered the founder of this school.

They argue that everything in the universe was created by the five main elements: fire, air, water, earth and ether (sky). These material elements are also called Dravya. As this school has a very scientific approach, they also developed the atomic theory, i.e. all material objects are made of atoms. This school was also responsible for the beginning of physics in Indian subcontinent. They are considered to be the propounder of the mechanical process of formation of this Universe.

Insights Nov-20 Arts 12/01/2021 2 11) Consider the following statements regarding Bharhut stupa.

1. The construction of Bharhut stupa was started and completed during Shunga period. 2. They contain numerous birth stories of the Buddha's previous lives, or Jataka tales. Which of the above statements is/are correct? a) 1 only b) 2 only c) Both 1 and 2 d) Neither 1 nor 2

b The Bharhut stupa (in MP) may have been first built by the Maurya king Ashoka in the 3rd century BCE, but many works of art were apparently added during the Shunga period, with many friezes from the 2nd century BCE.

They contain numerous birth stories of the Buddha's previous lives, or Jataka tales. Many of them are in the shape of large, round medallions. Queen Maya’s dream, preceding the birth of the Buddha, is also a major theme on the railing of the Bharhut“stupa”.

Insights Nov-20 Arts 12/01/2021 2 12) Consider the following statements.

1. Roman art style is more of ornamentation and decoration and is realistic in nature. 2. The Greeks art style is reflected in the muscular depictions of Gods. 3. Gandhara School integrates both Greek and Roman art styles. Which of the above statements is/are correct? a) 1, 2 b) 1, 3 c) 2, 3 d) 1, 2, 3

d There exists some difference between Greek and Roman styles, and Gandhara School integrates both the styles.

The idealistic style of Greeks is reflected in the muscular depictions of Gods and other men showing strength and beauty. On the other hand, Romans used art for ornamentation and decoration and is realistic in nature as opposed to Greek idealism.

Insights Nov-20 Arts 12/01/2021 2 13) Consider the following statements regarding Bhakti saints.

1. Saint Kabir is famous for his dohas (couplets) 2. Surdas wrote Ramacharithamanas, one of the most revered Hindu texts. Which of the above statements is/are correct? a) 1 only b) 2 only c) Both 1 and 2 d) Neither 1 nor 2

a Kabir was a 15th-century Indian mystic poet and saint, whose writings, according to some scholars, influenced Hinduism’s Bhakti movement. Kabir’s verses are found in Sikhism’s scripture Guru Granth Sahib. He is famous for his dohas (couplets).

Tulsidas wrote Ramacharithamanas

Insights Nov-20 Arts 12/01/2021 2 14) With reference to the Sangam age, Panar and Viraliyar were

a) Temple guards b) Bull fighters c) Singing bards d) Charity donors

c Poetry, music and dancing were popular among the people of the Sangam age. Liberal donations were given to poets by the kings, chieftains and nobles.

The royal courts were crowded with singing bards called Panar and Viraliyar. They were experts in folk songs and folk dances. The arts of music and dancing were highly developed. A variety of Yazhs and drums are referred to in the Sangam literature. Dancing was performed by Kanigaiyar. Koothu was the most popular entertainment of the people.

Insights Nov-20 Arts 12/01/2021 2 15) Consider the following statements regarding Bharatnatyam dance.

1. The accompanying song while performing it is generally dedicated to good harvests. 2. Its performance ends with a tillana which has its origin in the tarana of Hindustani music. Which of the above statements is/are correct? a) 1 only b) 2 only c) Both 1 and 2 d) Neither 1 nor 2

b As a solo dance, Bharatnatyam leans heavily on the abhinaya or mime aspect of dance - the nritya, where the dancer expresses the sahitya through movement and mime.

It is in the adoration of the Supreme Being. It is a vibrant dance performed to the accompaniment of musical syllables with a few lines of sahitya. The finale of the piece is a series of well designed rhythmic lines reaching a climax. A Bharatnatyam performance ends with a tillana which has its origin in the tarana of Hindustani music.

Insights Nov-20 Arts 12/01/2021 2 16) Which of the following most appropriately describes “Toranas”?

a) Collection of Chinars and Pietra duras seen in decorative works in Islami architecture b) Calligraphy inscribed on interior walls of royal durbars c) Gateways to Buddhist Stupas d) Style of dome-based architecture introduced by Turks

c Torana is a sacred or honorific gateway in Buddhist and Hindu architecture. It is made of wood or stone.

Toranas are associated with Buddhist stupas like the Great Stupa in Sanchi, as well as with Jain and Hindu structures, and also with several secular structures.

Insights Nov-20 Arts 12/01/2021 2 17) Consider the following statements regarding Ajanta Caves Paintings

1. The human & animal forms show a variety of graceful poses. 2. It depicts themes of court life and natural scenes 3. Most of the paintings were depicted after Gupta Period Which of the above statements is/are correct? a) 1, 3 b) 2, 3 c) 1, 2 d) 1, 2, 3

c The Ajanta Caves carved out of volcanic rock in the Maharashtra Plateau, situated near Aurangabad in Maharashtra.

Frescoes are paintings which are done on wet plaster in which colours become fixed as the plaster dries. They are found on the walls and ceilings at Ajanta. The human & animal forms show a variety of graceful poses. Various methods were used to create the illusion of depth. They depict themes of court life, feasting, processions, men and women at work, festivals and various natural scenes including animals, birds and flowers. Most of paintings were depicted well before Gupta’s time.

Insights Nov-20 Arts 12/01/2021 2 18) Rajaraja Chola I is associated with which of the following cultural heritages of India?

1. Brihadeeswara temple at Tanjore 2. The compilation of Thirumurai 3. Nagapattinam monastery Select the correct answer code: a) 1 only b) 2, 3 c) 1, 3 d) 1, 2, 3

d Rajaraja Chola I

He completed the construction of the famous Rajarajeswara temple or Brihadeeswara temple at Tanjore in 1010 A.D. He also helped in the construction of a Buddhist monastery at Nagapattinam. During his reign, the texts of the Tamil poets Appar, Sambandar and Sundarar were collected and edited into one compilation called Thirumurai. Further, He initiated a massive project of land survey and assessment in 1000 CE which led to the reorganization of the country into individual units known as valanadus.

Insights Nov-20 Arts 12/01/2021 2 19) Consider the following statements regarding Kuchipudi.

1. It is mentioned in Natya Shastra. 2. As per tradition, the dance must be accompanied by live vocals without instrumental music. 3. It is recognized as an Indian classical dance. Select the correct answer code: a) 1, 2 b) 2, 3 c) 1, 3 d) 1, 2, 3

c Kuchipudi

It has its roots in the ancient Hindu Sanskrit text of Natya Shastra. Like all major classical dances of India, it was also developed as a religious art linked to traveling bards, temples and spiritual beliefs. The Kuchipudi style was conceived by Siddhendra Yogi, a talented Vaishnava poet of 17th century. It begins with an invocation to Lord Ganesha followed by nritta (non-narrative and abstract dancing); shabdam (narrative dancing) and natya. The dance is accompanied by song which is typically Carnatic music. The singer is accompanied by musical instruments like mridangam, violin, flute and tambura. It is one of the major Indian classical dances. It originated in a village of Krishna district in modern Indian state of Andhra Pradesh. It derives its name from this village.

Insights Nov-20 Arts 12/01/2021 2 20) Which of the following pairs are correctly matched?

1. Dhyana Mudra: supreme enlightenment 2. Vitarka Mudra: fearlessness 3. Dharmachakra Mudra: Turning the Wheel of the Dharma Select the correct answer code: a) 1, 3 b) 3 only c) 2, 3 d) 1, 2, 3

b Abhaya Mudra: It indicates fearlessness.

Dharmachakra Mudra: It means ‘Turning the Wheel of the Dharma or Law’, i.e., setting into motion the wheel of Dharma. Uttarabodhi Mudra: It means supreme enlightenment. Dhyana Mudra: Indicates Meditation and is also called ‘Samadhi’ or ‘Yoga’ Mudra. Vitarka Mudra: It indicates teaching and discussion or intellectual debate.

Insights Nov-20 History 25/12/2020 1 1) Consider the following statements about the Rock Shelters of Bhimbetka.

1. Bhimbetka owes its name to the characters of Mahabharata. 2. Upper Palaeolithic as well as Medieval period rock paintings can be found in these caves. 3. Bhimbetka paintings are highly secular in nature. Which of the above statements is/are correct? a) 1, 3 b) 1, 2 c) 2, 3 d) 1, 2, 3

b Bhimbetka owes its name to the characters of the Mahabharata. It is believed that when the five brothers, called Pandavas, were banished from their kingdom, they came here and stayed in these caves, the massive rocks seating the gigantic frame of Bhima, the second Pandava.

The rock paintings have numerous layers belonging to various epochs of time, ranging from the Upper Palaeolithic, Mesolithic Age to the protohistoric, early historic and medieval periods. The most ancient scenes here believed to be commonly belonging to the Mesolithic Age. Animals such as bison, tiger, rhinoceros, wild boar, elephants, monkeys, antelopes, lizards, peacocks etc. have been abundantly depicted in the rock shelters. Popular religious and ritual symbols also occur frequently.

Insights Nov-20 History 25/12/2020 1 2) Consider the following statements about Mesolithic period.

1. Microliths were used by Mesolithic people for hunting. 2. Domestication of animals and primitive cultivation started during this period. 3. First human colonization of the Ganga plains took place during this period. Which of the above statements is/are correct? a) 1, 2 b) 2, 3 c) 1, 3 d) 1, 2, 3

d Mesolithic or Middle Stone Age which falls roughly from 10000 B.C. to 6000 B.C.

Mesolithic remains are found in Langhanj in Gujarat, Adamgarh in Madhya Pradesh and also in some places of Rajasthan, Uttar Pradesh and Bihar. In the sites of Mesolithic Age, a different type of stone tools is found. These are tiny stone artifacts, often not more than five centimeters in size, and therefore called microliths. The favourable climate, better rainfalls and warm atmosphere led to reduction in nomadism to seasonally sedentary settlement. They moved to new areas such as nearby rivers which provided water. First human colonization of the Ganga plains took place during this period.

Insights Nov-20 History 25/12/2020 1 3) Consider the following statements.

1. During the end of the eighteenth century and the early nineteenth century, a class of rich peasants known as jotedars consolidated their position in the villages and acquired vast areas of land. 2. Jotedars were loyal to the zamindars and helped them collect revenue from the ryots. Which of the above statements is/are correct? a) 1 only b) 2 only c) Both 1 and 2 d) Neither 1 nor 2

a While many zamindars were facing a crisis at the end of the eighteenth century, a group of rich peasants were consolidating their position in the villages. This class of rich peasants were known as jotedars. By the early nineteenth century, jotedars had acquired vast areas of land – sometimes as much as several thousand acres.

They controlled local trade as well as moneylending, exercising immense power over the poorer cultivators of the region. A large part of their land was cultivated through sharecroppers (adhiyars or bargadars) who brought their own ploughs, laboured in the field, and handed over half the produce to the jotedars after the harvest. They fiercely resisted efforts by zamindars to increase the jama of the village, prevented zamindari officials from executing their duties, mobilised ryots who were dependent on them, and deliberately delayed payments of revenue to the zamindar. The jotedars were most powerful in North Bengal. In some places they were called haoladars, elsewhere they were known as gantidars or mandals. Their rise inevitably weakened zamindari authority.

Insights Nov-20 History 25/12/2020 1 4) Arrange the following in the chronological order in which it was signed.

1. Treaty of Allahabad 2. Treaty of Bassein 3. Treaty of Srirangapatnam Select the correct answer code: a) 3-1-2 b) 1-2-3 c) 1-3-2 d) 3-2-1

c The Treaty of Allahabad was signed on 12 August 1765, between the Mughal Emperor Shah Alam II, son of the late Emperor Alamgir II, and Robert Clive, of the East India Company, as a result of the Battle of Buxar of 22 October 1764.

The Treaty of Seringapatam was signed on 18 March 1792 at the end of the Third Anglo-Mysore War. The Treaty of Bassein was a pact signed on 31 December 1802 between the British East India Company and Baji Rao II, the Maratha Peshwa of Pune in India after the Battle of Poona.

Insights Nov-20 History 25/12/2020 1 5) The Moderates were called so because

1. They had rejected the idea of participating in legislative councils. 2. They believed in confrontation rather than persuasion. Which of the above statements is/are correct? a) 1 only b) 2 only c) Both 1 and 2 d) Neither 1 nor 2

d Moderates were called so because they adopted peaceful and constitutional means to achieve their demands.

They had faith in British goodwill and justice. They organized in parties and associations, even extremists did the same, but the latter were more radical in their approach. Extremists believed in confrontation rather than persuasion.

Insights Nov-20 History 25/12/2020 1 6) Consider the following statements regarding Painted Grey Ware culture (PGW).

1. Painted Grey Ware culture (PGW) is a Stone Age culture. 2. It is contemporary to, and a successor of the Black and red ware culture. Which of the above statements is/are correct? a) 1 only b) 2 only c) Both 1 and 2 d) Neither 1 nor 2

b Painted Grey Ware culture (PGW) is an Iron Age culture of the Gangetic plain and the Ghaggar-Hakra valley, lasting from roughly 1200 BCE to 600 BCE.

PGW are extremely fine to touch, with a nice, smooth surface. Perhaps these were used on special occasions, for important people, and to serve special food. It is contemporary to, and a successor of the Black and red ware culture. It is succeeded by Northern Black Polished Ware from c. 700-500 BCE, associated with the rise of the great mahajanapada states and of the Magadha Empire.

Insights Nov-20 History 25/12/2020 1 7) Consider the following statements regarding the teachings of Jainism.

1. As per Mahavira, even inanimate objects have souls and varying degrees of consciousness. 2. Jainism believes in extreme penance and asceticism to attain ultimate knowledge. Which of the above statements is/are incorrect? a) 1 only b) 2 only c) Both 1 and 2 d) Neither 1 nor 2

d Mahavira regarded all objects, both animate and inanimate, have souls and various degrees of consciousness.

They possess life and feel pain when they are injured. Even the practice of agriculture was considered sinful as it causes injury to the earth, worms and animals. Similarly the doctrine of asceticism and renunciation was also carried to extreme lengths by the practice of starvation, nudity and other forms of self-torture.

Insights Nov-20 History 25/12/2020 1 8) Which of the following were annexed by the British citing the Doctrine of Lapse?

1. Satara 2. Sambalpur 3. Nagpur 4. Jhansi 5. Awadh Select the correct answer code: a) 1, 2, 3, 4 b) 2, 3, 4, 5 c) 1, 3, 4, 5 d) 1, 2, 3, 4, 5

d The final wave of annexations occurred under Lord Dalhousie who was the Governor-General from 1848 to 1856. He devised a policy that came to be known as the Doctrine of Lapse. The doctrine declared that if an Indian ruler died without a male heir his kingdom would “lapse”, that is, become part of Company territory. One kingdom after another was annexed simply by applying this doctrine: Satara (1848), Sambalpur (1850), Udaipur (1852), Nagpur (1853) and Jhansi (1854). Finally, in 1856, the Company also took over Awadh. This time the British had an added argument – they said they were “obliged by duty” to take over Awadh in order to free the people from the “misgovernment” of the Nawab! Enraged by the humiliating way in which the Nawab was deposed, the people of Awadh joined the great revolt that broke out in 1857.
Insights Nov-20 History 25/12/2020 1 9) Which of following movements was also known as the “Vande Matram movement”?

a) Civil disobedience movement b) Non-cooperation movement c) Swadeshi movement d) Quit India Movement

c Vandemataram Movement is also known as the Swadeshi Movement. It was an anti-Bengal partition movement. The Viceroy Lord Curzon divided Bengal province into two in 1905. Bengal people observed 16th October as a day of mourning when partition came into force. The song Vandemataram written by Bakim

Chandra Chatterjee became the prayer song for meetings on anti-partition movement.

Insights Nov-20 History 25/12/2020 1 10) The plan of Sir Stafford Cripps envisaged that after the Second World War

a) India should be granted complete independence b) India should be partitioned into two before granting independence c) India should be made a republic with the condition that she will join the Commonwealth d) India should be given Dominion status

d The main proposals of the Cripps mission were as follows.

1. An Indian Union with a dominion status would be set up; it would be free to decide its relations with the Commonwealth and free to participate in the United Nations and other international bodies. 2. After the end of the war, a constituent assembly would be convened to frame a new constitution. Members of this assembly would be partly elected by the provincial assemblies through proportional representation and partly nominated by the princes. 3. The British government would accept the new constitution subject to two conditions: (i) any province not willing to join the Union could have a separate constitution and form a separate Union, and (ii) the new constitution making body and the British government would negotiate a treaty to effect the transfer of power and to safeguard racial and religious minorities. 4. In the meantime, defence of India would remain in British hands and the governor-general’s powers would remain intact.

Insights Nov-20 History 25/12/2020 1 11) Which of the following were the practices associated with the Harappan Settlements?

1. Ploughing of crop fields 2. Mixed cropping 3. Practice of Irrigation 4. Consumption of animal products Select the correct answer code: a) 1, 2, 3 b) 1, 3, 4 c) 2, 3, 4 d) 1, 2, 3, 4

d Archaeologists have found evidence of a ploughed field at Kalibangan (Rajasthan).

The Harappans ate a wide range of animal products. Archaeologists have been able to reconstruct use of animals from finds of charred animal bones found at Harappan sites. These include those of cattle, sheep, goat, buffalo and pig. Most Harappan sites are located in semi-aridlands, where irrigation was probably required for agriculture. Traces of canals, water reservoirs and wells have been found at the Harappan sites indicate that agriculture was practiced.

Insights Nov-20 History 25/12/2020 1 12) Consider the following statements.

1. As per Ramanuja, God is Sagunabrahman. 2. As per Madhava, the world is not an illusion but a reality. Which of the above statements is/are correct? a) 1 only b) 2 only c) Both 1 and 2 d) Neither 1 nor 2

c In the twelfth century, Ramanuja preached Visishtadvaita.

According to him God is Sagunabrahman. The creative process and all the objects in creation are real but not illusory as was held by Sankaracharya. Therefore, God, soul, matter are real. But God is inner substance and the rest are his attributes. He also advocated prabattimarga or path of self-surrender to God. He invited the downtrodden to Vaishnavism. In the thirteenth century, Madhava from Kannada region propagated Dvaita or dualism of Jivatma and Paramatma. According to his philosophy, the world is not an illusion but a reality. God, soul, matter are unique in nature.

Insights Nov-20 History 25/12/2020 1 13) Consider the following statements regarding Subsidiary Alliance.

1. According to Subsidiary Alliance, Indian rulers were not allowed to have their independent armed forces and were protected by the Company. 2. If the Indian rulers failed to make the payment, then part of their territory was taken away as penalty. 3. Indian rulers regularly paid for the subsidiary forces and none of the Indian territories were ceded by the British. Which of the above statements is/are correct? a) 1, 2 b) 1, 3 c) 2, 3 d) 1, 2, 3

a According to the terms of this subsidiary alliance, Indian rulers were not allowed to have their independent armed forces.

They were to be protected by the Company but had to pay for the “subsidiary forces” that the Company was supposed to maintain for the purpose of this protection. If the Indian rulers failed to make the payment, then part of their territory was taken away as penalty. For example, when Richard Wellesley was Governor-General (1798-1805), the Nawab of Awadh was forced to give over half of his territory to the Company in 1801, as he failed to pay for the “subsidiary forces”. Hyderabad was also forced to cede territories on similar grounds.

Insights Nov-20 History 25/12/2020 1 14) The aim of Public Safety Bill, 1928 was to

a) Regulate the manufacture, sale, possession, and carrying of firearms. b) Allow British subjects to be tried by Indian judges. c) Deport the foreigners suspected of propagating socialist and communist ideas. d) Abolish oppressive social practices among various religious groups.

c Public safety bill aimed to stop Communist movement in India by cutting it off from British & foreign communist organizations. It called to deport the foreigners suspected of propagating socialist and communist ideas.
Insights Nov-20 History 25/12/2020 1 15) Consider the following statements about Champaran Satyagraha

1. The Champaran Satyagraha of 1917 was the first Satyagraha movement inspired by Gandhi in India. 2. Gandhi was requested by Rajendra Prasad to look into the problems of the indigo planters of Champaran. 3. Gandhi’s method of inquiry at Champaran was based on surveys by the volunteers. Which of the above statements is/are correct? a) 2 only b) 1, 3 c) 3 only d) 1, 2, 3

b Gandhi was so thoroughly persuaded by Rajkumar Shukla, an indigo cultivator from Champaran that he decided to investigate into the matter.

Gandhi’s method of inquiry at Champaran was based on surveys by the volunteers. The respondents who willingly gave statements should sign the papers or give thumb impressions. For those unwilling to participate, the reasons must be recorded by the volunteers. The principal volunteers in this survey were mostly lawyers like Babu Rajendra Prasad, Dharnidhar Prasad, Gorakh Prasad, Ramnawami Prasad, Sambhusaran and Anugraha Narain Sinha.

Insights Nov-20 History 25/12/2020 1 16) Consider the following statements regarding Mauryan Empire.

1. Slavery was largely absent in Mauryan Empire. 2. Mauryan government had equal control over all the regions of its empire. Which of the above statements is/are correct? a) 1 only b) 2 only c) Both 1 and 2 d) Neither 1 nor 2

d Slaves were employed in commercial activities. Mauryan government didn’t have equal control over all the regions of its empire, few regions were directly controlled under the Mauryan administration, while others were under the local kings, who accepted the over lordship of Mauryans.
Insights Nov-20 History 25/12/2020 1 17) Match List-I (Terms in Sangam Age) with List-II (Their meanings)

       List-I                                 List-II A. Enadi                       1. The lowest class B. Paraiyar                  2. The Ruling class C. Arasar                     3. Rich peasants D. Vellala                    4. Agricultural Labourers                                     5. Captains of the army Select the correct answer code:     A B C D a) 5 4 2 3 b) 3 4 5 2 c) 3 1 4 2 d) 5 1 4 3

a
Insights Nov-20 History 25/12/2020 1 18) Curzon brought in the Indian Universities Act which

a) Removed social sciences from the curriculum of private universities to curb nationalism b) Did away with the territorial jurisdiction of the Universities c) Brought all the universities in India under the control of the government d) Nationalized all higher educational institutions in India

c Curzon took a serious view of the fall in the standard of education and discipline in the educational institutions.

In his view the universities had degenerated into factories for producing political revolutionaries. To set the educational system in order, he instituted in 1902, a Universities Commission to go into the entire question of university education in the country. On the basis of the findings and recommendations of the Commission, Curzon brought in the Indian Universities Act of 1904, which brought all the universities in India under the control of the government.

Insights Nov-20 History 25/12/2020 1 19) By the time Gandhiji arrived in India

1. He had already forged of technique of Satyagraha in South Africa 2. Indian National Congress (INC) was already established in India 3. The first Swadeshi movement had already been waged Select the correct answer code: a) 1, 2 b) 2, 3 c) 1, 3 d) 1, 2, 3

d It was in South Africa that Mahatma Gandhi first forged the distinctive techniques of non-violent protest known as satyagraha.

The India that Mahatma Gandhi came back to in 1915 was rather different from the one that he had left in 1893. Although still a colony of the British, it was far more active in a political sense. The Indian National Congress now had branches in most major cities and towns. Through the Swadeshi movement of 1905-07 it had greatly broadened its appeal among the middle classes.

Insights Nov-20 History 25/12/2020 1 20) Ahmadabad Mill Strike 1918 led by Gandhiji was based on the issue of

a) Inhuman treatment of mill workers by the management b) Objection of management on participation of workers in the freedom struggle c) Plague Bonus of the previous year to workers d) Large-scale layoffs of mill workers

c Ahmedabad Mill Strike (1918)

First Hunger Strike:In March 1918, Gandhi intervened in a dispute between cotton mill owners of Ahmedabad and the workers over the issue of discontinuation of the plague bonus. Gandhi asked the workers to go on a strike and demand a 35 per cent increase in wages. Gandhi advised the workers to remain non-violent while on strike. He himself undertook a fast unto death (his first) to strengthen the workers’ resolve. The result was that the strike was successful and the workers got a 35% wage increase.

Insights Nov-20 Environment 25/12/2020 1 1) Consider the following statements.

1. The energy pyramid of an ecosystem is always upright and narrows to the top. 2. The biomass in the upper trophic levels is generally very high as compared to the lower trophic levels. Which of the above statements is/are correct? a) 1 only b) 2 only c) Both 1 and 2 d) Neither 1 nor 2

a Since each higher trophic level receives only a fraction of energy of the lower trophic levels, the energy pyramid is narrow at the top.

But, generally (barring some aquatic ecosystems) lower trophic levels have higher biomass as compared to the higher trophic levels.

Insights Nov-20 Environment 25/12/2020 1 2) Consider the following statements regarding Mediterranean Forests.

1. Mediterranean Forests are found in only those countries bordering Mediterranean Sea. 2. They have thick barks and wax coated leaves which help them reduce transpiration. 3. Mediterranean Forests are known as'Orchards of the world' for their fruit cultivation. Which of the above statements is/are correct? a) 1, 2 b) 1, 3 c) 2, 3 d) 1, 2, 3

c Mediterranean forests are found in areas around Mediterranean Sea, Central Chile, South-West USA, Australia, Africa.

They have thick barks and wax coated leaves which help them reduce transpiration. Due to these characteristics, Mediterranean trees adapt themselves to dry summers. Mediterranean Forests are known as'Orchards of the world' for their fruit cultivation. People have removed the natural vegetation here in order to cultivate what they want to. So, Citrus fruits such as oranges, figs, olives and grapes are commonly cultivated here.

Insights Nov-20 Environment 25/12/2020 1 3) Oil spills can be treated with bioremediation by using some species of

1. Archea 2. Plants 3. Bacteria 4. Algae 5. Fungi Select the correct answer using the codes below. a) 1, 3, 4, 5 b) 1, 2, 3, 4 c) 1, 2, 5 d) 1, 2, 3, 4, 5

d Bioremediation refers to the use of specific microorganisms or plants to metabolize and remove harmful substances. These organisms are known for their biochemical and physical affinity to hydrocarbons among other pollutants. Various types of bacteria, archaea, algae, fungi, and some species of plants are all able to break down specific toxic waste products into safer constituents.
Insights Nov-20 Environment 25/12/2020 1 4) The higher the amount of biological oxygen demand (BOD) in a certain segment of water

1. The greater the amount of organic matter available for oxygen consuming bacteria. 2. Higher could be the level of pollution in the water Select the correct answer code: a) 1 only b) 2 only c) Both 1 and 2 d) Neither 1 nor 2

c Biochemical Oxygen Demand or Biological Oxygen Demand, is a measurement of the amount of dissolved oxygen (DO) that is used by aerobic microorganisms when decomposing organic matter in water.

Biochemical Oxygen Demand is an important water quality parameter because it provides an index to assess the effect discharged wastewater will have on the receiving environment. The higher the BOD value, the greater the amount of organic matter or “food” (read nutrients) available for oxygen consuming bacteria.

Insights Nov-20 Environment 25/12/2020 1 5) Consider the following statements regarding ocean acidification.

1. It largely occurs due to the high absorption of nitrogenous based acidic compounds. 2. Introduction of sea grasses can reduce the impact of acidification. Which of the above statements is/are correct? a) 1 only b) 2 only c) Both 1 and 2 d) Neither 1 nor 2

b Ocean acidification refers to a reduction in the pH of the ocean over an extended period of time, caused primarily by uptake of carbon dioxide (CO2) from the atmosphere. Nitrogenous compounds contribute fraction to ocean acidification.

Sea grass has ability to control ocean acidification.

Insights Nov-20 Environment 25/12/2020 1 6) Arrange the following ecosystems in the decreasing order of biomass productivity (g per metre square per year).

1. Coral reefs 2. Open ocean 3. Tropical rainforest 4. Deserts Select the correct answer code: a) 1-3-2-4 b) 3-1-2-4 c) 3-1-4-2 d) 1-3-4-2

b
Insights Nov-20 Environment 25/12/2020 1 7) Which of the following are the harmful environmental effects of Oxides of Nitrogen?

1. It damage the leaves of plants and retard the rate of photosynthesis. 2. It is toxic to living tissues also. 3. It is harmful to various textile fibres and metals. Select the correct answer code: a) 1, 2 b) 1, 3 c) 2, 3 d) 1, 2, 3

The irritant red haze in the traffic and congested places is due to oxides of nitrogen.

Higher concentrations of NO2 damage the leaves of plants and retard the rate of photosynthesis. Nitrogen dioxide is a lung irritant that can lead to an acute respiratory disease in children. It is toxic to living tissues also. Nitrogen dioxide is also harmful to various textile fibres and metals.

Insights Nov-20 Environment 25/12/2020 1 8) Which of the following are Tropical Grasslands?

1. Savanna 2. Campos 3. Prairie 4. Llanos 5. Steppe Select the correct answer code: a) 1, 2, 3 b) 1, 2, 4 c) 1, 2, 5 d) 2, 3, 4, 5

b Grasslands are known by different names in different regions.

Tropical Grasslands are: • East Africa- Savanna • Brazil- Campos • Venezuela- Llanos Temperate Grasslands are: • Argentina- Pampas • N. America- Prairie • S. Africa- Veld • C. Asia- Steppe • Australia- Down

Insights Nov-20 Environment 25/12/2020 1 9) Consider the following statements.

1. Fungi cannot be used in the bioremediation of radioactively polluted sites. 2. Fungi can act as bio-fertilizers by actively competing for nutrients and space with pathogenic microorganisms. Which of the above statements is/are correct? a) 1 only b) 2 only c) Both 1 and 2 d) Neither 1 nor 2

b In agriculture, fungi are useful if they actively compete for nutrients and space with pathogenic microorganismssuch as bacteria or other fungi. For example, certain fungi may be used to eliminate or suppress the growth of harmful plant pathogens, such as insects, mites, weeds, nematodes.

Fungi have been shown to bio mineralize uranium oxides, suggesting they may have application in the bioremediation of radioactively polluted sites.

Insights Nov-20 Environment 25/12/2020 1 10) Which of the following are the key elements that lead to variation in the physical and chemical conditions of different habitats?

1. Temperature 2. Pathogens 3. Soil 4. Predators 5. Competitors 6. Light Select the correct answer code: a) 1, 3, 6 b) 1, 2, 3, 5, 6 c) 1, 3, 4, 6 d) 1, 2, 3, 4, 5, 6

d What are the key elements that lead to so much variation in the physical and chemical conditions of different habitats?

The most important ones are temperature, water, light and soil. We must remember that the physicochemical (abiotic) components alone do not characterise the habitat of an organism completely; the habitat includes biotic components also – pathogens, parasites, predators and competitors – of the organism with which they interact constantly.

Insights Nov-20 Environment 25/12/2020 1 11) Which of the following are recognised as the Ramsar sites in India?

1. Harike Lake 2. Wular Lake 3. Keoladeo National Park 4. Chilika Lake Select the correct answer code: a) 1, 2, 3 b) 2, 3, 4 c) 1, 2, 3, 4 d) 1, 2, 4

Some of the Ramsar sites in India include

Chilika Lake in Odisha Keoladeo National Park in Rajasthan Harike Lake in Punjab Loktak Lake in Manipur Wular Lake in Jammu and Kashmir

Insights Nov-20 Environment 25/12/2020 1 12) Arrange the following ecosystems in decreasing order of net primary productivity

1. Tropical seasonal forest 2. Tropical rain forest 3. Temperate Grassland 4. Woodland and Shrubland Select the correct answer code: a) 2-1-3-4 b) 2-1-4-3 c) 2-4-3-1 d) 1-2-3-4

b Net primary productivity, or NPP, is gross primary productivity minus the rate of energy loss to metabolism and maintenance. In other words, it's the rate at which energy is stored as biomass by plants or other primary producers and made available to the consumers in the ecosystem.

Net primary productivity varies among ecosystems and depends on many factors. These include solar energy input, temperature and moisture levels, carbon dioxide levels, nutrient availability, and community interactions. Tropical rain forests receive overall more rainfall than seasonal forests and grasslands. Also, a forest will certainly be more productive than grassland.

Insights Nov-20 Environment 25/12/2020 1 13) Consider the following statements regarding Acclimatization.

1. Acclimatization occurs in a short period of time and within the organism's lifetime, whereas adaptation is a development that takes place over many generations. 2. Humans who migrate permanently to a higher altitude naturally acclimatize to their new environment by developing an increase in the number of red blood cells to increase the oxygen carrying capacity of the blood. Which of the above statements is/are correct? a) 1 only b) 2 only c) Both 1 and 2 d) Neither 1 nor 2

c Acclimatization is the process in which an individual organism adjusts to a change in its environment (such as a change in altitude, temperature, humidity, photoperiod, or pH), allowing it to maintain performance across a range of environmental conditions. Acclimatization occurs in a short period of time (hours to weeks), and within the organism's lifetime (compared to adaptation, which is a development that takes place over many generations).

Humans who migrate permanently to a higher altitude naturally acclimatize to their new environment by developing an increase in the number of red blood cells to increase the oxygen carrying capacity of the blood, in order to compensate for lower levels of oxygen intake.

Insights Nov-20 Environment 25/12/2020 1 14) Consider the following statements.

1. In the aquatic environment, the sediment-characteristics often determine the type of benthic animals that can thrive there. 2. Like humans, plants also have mechanisms to maintain internal temperature. 3. Very small animals are rarely found in polar regions since thermoregulation is energetically expensive for these animals. Which of the above statements is/are correct? a) 1, 2 b) 1, 3 c) 2, 3 d) 1, 2, 3

b The mechanisms used by most mammals to regulate their body temperature are similar to the ones that we humans use. We maintain a constant body temperature of 37 deg C. In summer, when outside temperature is more than our body temperature, we sweat profusely.

The resulting evaporative cooling, similar to what happens with a desert cooler in operation, brings down the body temperature. In winter when the temperature is much lower than 37 deg C, we start to shiver, a kind of exercise which produces heat and raises the body temperature. Plants, on the other hand, do not have such mechanisms to maintain internal temperatures. Thermoregulation is energetically expensive for many organisms. This is particularly true for small animals like shrews and humming birds. Heat loss or heat gain is a function of surface area. Since small animals have a larger surface area relative to their volume, they tend to lose body heat very fast when it is cold outside; then they have to expend much energy to generate body heat through metabolism. This is the main reason why very small animals are rarely found in polar regions.

Insights Nov-20 Environment 25/12/2020 1 15) A substantial increase in atmospheric aerosols is likely to

1. Affect rainfall patterns 2. Cut down the destruction of stratospheric ozone significantly 3. Increase global warming by large proportions Select the correct answer code: a) 1 only b) 1, 3 c) 1, 2, 3 d) 1, 2

a Aerosols are minute particles suspended in the atmosphere. When these particles are sufficiently large, we notice their presence as they scatter and absorb sunlight. Their scattering of sunlight can reduce visibility (haze) and redden sunrises and sunsets.

Aerosols interact both directly and indirectly with the Earth's radiation budget and climate. As a direct effect, the aerosols scatter sunlight directly back into space. As an indirect effect, aerosols in the lower atmosphere can modify the size of cloud particles, changing how the clouds reflect and absorb sunlight, thereby affecting the Earth's energy budget. Aerosols also can act as sites for chemical reactions to take place (heterogeneous chemistry). The most significant of these reactions are those that lead to the destruction of stratospheric ozone. During winter in the polar regions, aerosols grow to form polar stratospheric clouds. The large surface areas of these cloud particles provide sites for chemical reactions to take place. These reactions lead to the formation of large amounts of reactive chlorine and, ultimately, to the destruction of ozone in the stratosphere.

Insights Nov-20 Environment 25/12/2020 1 16) Bioluminescence is the property of a living organism to produce and emit light. Which of the following organisms show the property of bioluminescence?

1. Animals 2. Plants 3. Fungi 4. Bacteria Select the correct answer code: a) 1, 2, 3 b) 1, 3, 4 c) 2, 3, 4 d) 1, 2, 3, 4

d Bioluminescence is the property of a living organism to produce and emit light. “Animals, plants, fungi and bacteria show bioluminescence.
Insights Nov-20 Environment 25/12/2020 1 17) Consider the following statements regarding Parasitism.

1. Parasitism occurs when two organisms interact, but while one benefits, the other experiences harm. 2. Tapeworm attaching itself to the intestine of a cow is an example of Parasitism. 3. The parasite always kill the host. Which of the above statements is/are correct? a) 1, 2 b) 1, 3 c) 1 only d) 2, 3

a A parasite is an organism that lives in or on another living organism, deriving nutrients from it. In this relationship the parasite benefits, but the organism being fed upon, the host, is harmed. The host is usually weakened by the parasite as it siphons resources the host would normally use to maintain itself. The parasite, however, is unlikely to kill the host. This is because the parasite needs the host to complete its reproductive cycle by spreading to another host.

The reproductive cycles of parasites are often very complex, sometimes requiring more than one host species. A tapeworm is a parasite that causes disease in humans when contaminated, undercooked meat such as pork, fish, or beef is consumed. The tapeworm can live inside the intestine of the host for several years, benefiting from the food the host is bringing into its gut by eating. The parasite moves from species to species as it requires two hosts to complete its life cycle.

Insights Nov-20 Environment 25/12/2020 1 18) Which of the following are the Biodiversity Hotspots in India?

1. Western Ghats 2. Himalayas 3. Indo-Bhutan region 4. Sundaland Select the correct answer code: a) 1, 2, 3 b) 2, 3, 4 c) 1, 2, 4 d) 1, 2, 3, 4

c India has some of the world's most biodiverse regions. It hosts 4 biodiversity hotspots: the Himalayas, the Western Ghats, the Indo-Burma region and the Sundaland (Includes group of Islands).
Insights Nov-20 Environment 25/12/2020 1 19) To avoid unfavourable conditions, which of the following enter into diapause, a stage of suspended development.

a) Zooplankton b) Bears c) Snails d) Fish

a In animals, the organism, if unable to migrate, might avoid the stress by escaping in time. The familiar case of bears going into hibernation during winter is an example of escape in time. Some snails and fish go into aestivation to avoid summer–related problems-heat and dessication. Under unfavourable conditions many zooplankton species in lakes and ponds are known to enter diapause, a stage of suspended development.
Insights Nov-20 Environment 25/12/2020 1 20) Ecological balance may be disturbed due to the

1. Introduction of new species 2. Natural hazards 3. Anthropogenic pollution Select the correct answer code: a) 1, 3 b) 1, 2 c) 2, 3 d) 1, 2, 3

d Exotic species may disturb the local food chain or create scarcity of competitive resources.

Natural hazards like tsunami or cyclone can uproot coastal mangroves. In the plants, any disturbance in the native forests such as clearing the forest causes a change in the species distribution.

Insights Dec-20 Polity 06/01/2021 1 1) Which of the following provisions of constitution can be amended through simple majority?

1. Rules of procedure in Parliament. 2. Conferment of more jurisdiction on the Supreme Court. 3. Delimitation of constituencies. 4. Power of Parliament to amend the Constitution and its procedure Select the correct answer code: a) 1, 2, 3 b) 1, 3, 4 c) 2, 3, d) 1, 2, 3, 4

a Power of Parliament to amend the Constitution and its procedure can be amended by special majority of Parliament and consent of states

A number of provisions in the Constitution can be amended by a simple majority of the two Houses of Parliament outside the scope of Article 368. These provisions include: 1. Admission or establishment of new states. 2. Formation of new states and alteration of areas, boundaries or names of existing states. 3. Abolition or creation of legislative councils in states. 4. Second Schedule—emoluments, allowances, privileges and so on of the president, the governors, the Speakers, judges, etc. 5. Quorum in Parliament. 6. Salaries and allowances of the members of Parliament. 7. Rules of procedure in Parliament. 8. Privileges of the Parliament, its members and its committees. 9. Use of English language in Parliament. 10. Number of puisne judges in the Supreme Court. 11. Conferment of more jurisdiction on the Supreme Court. 12. Use of official language. 13. Citizenship—acquisition and termination. 14. Elections to Parliament and state legislatures. 15. Delimitation of constituencies. 16. Union territories. 17. Fifth Schedule—administration of scheduled areas and scheduled tribes. 18. Sixth Schedule—administration of tribal areas.

Insights Dec-20 Polity 06/01/2021 1 2) The Parliament cannot amend which of these provisions that form the ‘basic structure’ of the Constitution

1. Freedom and dignity of the individual 2. Principle of equality 3. Independence of Judiciary 4. Rule of law Select the correct answer code: a) 1, 2, 3 b) 1, 3, 4 c) 2, 3, 4 d) 1, 2, 3, 4

d From the various judgements, the following have emerged as ‘basic features’ of the Constitution:

1. Supremacy of the Constitution 2. Sovereign, democratic and republican nature of the Indian polity 3. Secular character of the Constitution 4. Separation of powers between the legislature, the executive and the judiciary 5. Federal character of the Constitution 6. Unity and integrity of the nation 7. Welfare state (socio-economic justice) 8. Judicial review 9. Freedom and dignity of the individual 10. Parliamentary system 11. Rule of law 12. Harmony and balance between Fundamental Rights and Directive Principles 13. Principle of equality 14. Free and fair elections 15. Independence of Judiciary 16. Limited power of Parliament to amend the Constitution 17. Effective access to justice 18. Principles (or essence) underlying fundamental rights. 19. Powers of the Supreme Court under Articles 32, 136, 141 and 142 20. Powers of the High Courts under Articles 226 and 227

Insights Dec-20 Polity 06/01/2021 1 3) Consider the following statements regarding Lok Sabha.

1. Lok Sabha has a normal term of 5 years, after which it automatically dissolves. 2. Once the Lok Sabha is dissolved before the completion of its normal tenure, the dissolution is irrevocable. 3. Revocation of National Emergency is the exclusive power of Lok Sabha. Which of the above statements is/are correct? a) 1 only b) 1, 2 c) 1, 3 d) 1, 2, 3

b The dissolution of the Lok Sabha may take place in either of two ways:

1. Automatic dissolution, that is, on the expiry of its tenure of five years or the terms as extended during a national emergency; or 2. Whenever the President decides to dissolve the House, which he is authorised to do. Once the Lok Sabha is dissolved before the completion of its normal tenure, the dissolution is irrevocable. A proclamation of emergency may be revoked by the President at any time by a subsequent proclamation. Such a proclamation does not require the parliamentary approval. Further, the President must revoke a proclamation if the Lok Sabha passes a resolution disapproving its continuation.

Insights Dec-20 Polity 06/01/2021 1 4) The Constitution of India lays down which of the following conditions for the Governor’s office?

1. The Governor should not belong to the state where is appointed. 2. The Governor should not be a member of either House of Parliament or a House of the state legislature. 3. The Governor’s appointment must be made after consulting the chief Minister of the state. Select the correct answer code: a) 1, 2 b) 1, 3 c) 2, 3 d) 2 only

d The Governor should not be a member of either House of Parliament or a House of the state legislature. If any such person is appointed as governor, he is deemed to have vacated his seat in that House on the date on which he enters upon his office as the governor.

Two conventions have developed over the years. First, he should be an outsider, that is, he should not belong to the state where he is appointed, so that he is free from the local politics. Second, while appointing the governor, the president is required to consult the chief minister of the state concerned, so that the smooth functioning of the constitutional machinery in the state is ensured. However, both the conventions have been violated in some of the cases.

Insights Dec-20 Polity 06/01/2021 1 5) Consider the following statements regarding Election Commission of India

1. The Election commission shall consist of the chief election commissioner and such number of other election commissioners as the parliament may from time to time fix. 2. In case of difference of opinion amongst the chief election commissioner and other election commissioners the matter is referred to the Supreme Court. 3. The conditions of service and tenure of office of election commissioners shall be determined by the president. Which of the above statements is/are incorrect? a) 1 only b) 1, 2 c) 1, 3 d) 2, 3

b Article 324 of the Constitution has made the provisions with regard to the composition of election commission.

・ • The Election Commission shall consist of the chief election commissioner and such number of other election commissioners, if any, as the president may from time to time fix. ・ • The conditions of service and tenure of office of the election commissioners and the regional commissioners shall be determined by the president. ・ • In case of difference of opinion amongst the Chief election commissioner and/or two other election commissioners, the matter is decided by the Commission by majority.

Insights Dec-20 Geography 06/01/2021 1 1) Consider the following statements.

1. Much of the Earth’s mantle consists of magma. 2. Magma is a mixture of minerals and also contains small amounts of dissolved gases. 3. The high pressure under Earth’s crust keep magma in a solid state. Which of the above statements is/are correct? a) 1, 2 b) 1, 3 c) 2, 3 d) 1, 2, 3

a Magma is extremely hot liquid and semi-liquid rock located under Earth’s surface. Earth has a layered structure that consists of the inner core, outer core, mantle, and crust. Much of the planet’s mantle consists of magma. This magma can push through holes or cracks in the crust, causing a volcanic eruption. When magma flows or erupts onto Earth’s surface, it is called lava.

Like solid rock, magma is a mixture of minerals. It also contains small amounts of dissolved gases such as water vapor, carbon dioxide, and sulfur. The high temperatures and pressure under Earth’s crust keep magma in its fluid state.

Insights Dec-20 Geography 06/01/2021 1 2) The thinnest layer of our planet is

a) Mantle b) Crust c) Outer Core d) Inner Core

b Earth has 4 layers:

・ • The outer crust that we live on ・ • The plastic-like mantle ・ • The liquid outer core ・ • The solid inner core The crust is by far the thinnest of the layers of earth. The thickness varies depending on where you are on earth, with oceanic crust being 5-10 km and continental mountain ranges being up to 30-45 km thick. Thin oceanic crust is denser than the thicker continental crust and therefore 'floats' lower in the mantle as compared to continental crust.

Insights Dec-20 Geography 06/01/2021 1 3) Consider the following statements.

1. About 98 per cent of the total crust of the earth is composed of titanium, hydrogen, phosphorous, manganese and sulphur. 2. All three rock types (igneous, sedimentary, and metamorphic) can be found in Earth’s crust. 3. The oldest rocks on our planet are part of the continental crust. Which of the above statements is/are correct? a) 1, 2 b) 1, 3 c) 2, 3 d) 3 only

c The most abundant elements in the Earth’s crust include (listed here by weight percent) oxygen, silicon, aluminum, iron, and calcium.

All three rock types (igneous, sedimentary, and metamorphic) can be found in Earth’s crust. The oldest rocks on our planet are part of the continental crust and date back approximately 4 billion years in age. Ocean crust is constantly recycled through our planet’s system of plate tectonics and only dates back to approximately 200 million years ago.

Insights Dec-20 Geography 06/01/2021 1 4) Which of the following sources of information are considered as the direct sources of information about the Earth’s interior?

1. Drilling projects in Mining areas 2. Volcanic eruption 3. Meteors that at times reach the earth 4. Gravitation Select the correct answer code: a) 1, 2, 3 b) 1, 2, 4 c) 1, 2 d) 2, 3, 4

c Direct Sources: The most easily available solid earth material is surface rock or the rocks we get from mining areas. Besides mining, scientists have taken up a number of projects to penetrate deeper depths to explore the conditions in the crustal portions. Many deep drilling projects have provided large volume of information through the analysis of materials collected at different depths. Volcanic eruption forms another source of obtaining direct information.

Indirect Sources: Another source of information are the meteors that at times reach the earth. The other indirect sources include gravitation, magnetic field, and seismic activity.

Insights Dec-20 Geography 06/01/2021 1 5) Consider the following statements.

1. Earth’s crust has a density higher than the mantle part of Earth. 2. The inner core of the Earth is in solid state. Which of the above statements is/are incorrect? a) 1 only b) 2 only c) Both 1 and 2 d) Neither 1 nor 2

a The portion of the interior beyond the crust is called the mantle. It has a density higher than the crust’s (3.4 g/cm3).

The outer core of Earth is in liquid state while the inner core is in solid state. The core is made up of very heavy material mostly constituted by nickel and iron. It is sometimes referred to as the nife layer.

Insights Dec-20 Geography 06/01/2021 1 6) Consider the following statements regarding Karst topography.

1. It is commonly associated with carbonate rocks. 2. Such topography occurs only in tropical and temperate environments. 3. Nearly a quarter of the world’s population depends upon water supplied from karst areas. Which of the above statements is/are correct? a) 1, 2 b) 1, 3 c) 2, 3 d) 1, 2, 3

b The term karst describes a distinctive topography that indicates dissolution (also called chemical solution) of underlying soluble rocks by surface water or ground water. Although commonly associated with carbonate rocks (limestone and dolomite) other highly soluble rocks such as evaporates (gypsum and rock salt) can be sculpted into karst terrain.

Understanding caves and karst is important because ten percent of the Earth’s surface is occupied by karst landscape and as much as a quarter of the world’s population depends upon water supplied from karst areas. Though most abundant in humid regions where carbonate rock is present, karst terrain occurs in temperate, tropical, alpine and polar environments.

Insights Dec-20 Geography 06/01/2021 1 7) Consider the following statements regarding Badland Topography.

1. Presence of Hard archaic granite rock system. 2. Characterized by steep slopes and minimal vegetation. 3. The geologic forms of Canyons, ravines, gullies, buttes and mesas are common in badlands. Which of the above statements is/are correct? a) 1, 2 b) 1, 3 c) 2, 3 d) 1, 2, 3

c Badlands are a type of dry terrain where softer sedimentary rocks and clay-rich soils have been extensively eroded by wind and water. They are characterized by steep slopes, minimal vegetation, lack of a substantial regolith, and high drainage density. They can resemble malpaís, a terrain of volcanic rock. Canyons, ravines, gullies, buttes, mesas, hoodoos and other such geologic forms are common in badlands.
Insights Dec-20 Geography 06/01/2021 1 8) Consider the following statements.

1. Moraines are accumulations of dirt and rocks that have fallen onto the glacier surface or have been pushed along by the glacier as it moves. 2. Loess is a type of silt which forms fertile topsoil in some parts of the world. Which of the above statements is/are correct? a) 1 only b) 2 only c) Both 1 and 2 d) Neither 1 nor 2

c Moraines are accumulations of dirt and rocks that have fallen onto the glacier surface or have been pushed along by the glacier as it moves. The dirt and rocks composing moraines can range in size from powdery silt to large rocks and boulders. A receding glacier can leave behind moraines that are visible long after the glacier retreats.

Loess consists of tiny mineral particles brought by wind to the places where they now lie. It is a product of past glacial activity in an area. It is a sedimentary deposit of mineral particles which are finer than sand but coarser than dust or clay, deposited by the wind. Loess is a type of silt which forms fertile topsoil in some parts of the world.

Insights Dec-20 Geography 06/01/2021 1 9) Which of these geographical features is most likely to have the lowest mean elevation?

a) Plains b) Deltas c) Plateaus d) Islands

b
Insights Dec-20 Geography 06/01/2021 1 10) Lithospheric plates move around very slowly – just a few millimetres each year. This is because of

a) Rotation of earth b) Ocean currents c) Gravitational Force acting on the plates d) Movement of magma inside the earth

d The lithosphere is broken into a number of plates known as the Lithospheric plates. You will be surprised to know that these plates move around very slowly – just a few millimetres each year. This is because of the movement of the molten magma inside the earth.
Insights Dec-20 Geography 06/01/2021 1 11) Consider the following statements.

1. The rocks equidistant on either side of the crest of mid-oceanic ridges show remarkable similarities in terms of period of formation, chemical compositions and magnetic properties. 2. The age of the oceanic rocks decrease as one move away from the crest of mid-oceanic ridge. Which of the above statements is/are correct? a) 1 only b) 2 only c) Both 1 and 2 d) Neither 1 nor 2

a The rocks equidistant on either sides of the crest of mid-oceanic ridges show remarkable similarities in terms of period of formation, chemical compositions and magnetic properties. Rocks closer to the mid-oceanic ridges have normal polarity and are the youngest. The age of the rocks increases as one moves away from the crest.
Insights Dec-20 Geography 06/01/2021 1 12) The earth movements are divided on the basis of the forces which cause them. Which of the following are considered as endogenic forces?

1. Glaciers 2. Sea-waves 3. Landslides 4. Diastrophic forces Select the correct answer code: a) 1, 2, 3 b) 3, 4 c) 1, 3, 4 d) 2, 3, 4

b The earth movements are divided on the basis of the forces which cause them. The forces which act in the interior of the earth are called as Endogenic forces and the forces that work on the surface of the earth are called as Exogenic forces.
Insights Dec-20 Geography 06/01/2021 1 13) Which of the following evidences supported the continental drift?

1. Jig-Saw-Fit of shorelines of Africa and South America 2. Rocks of Same Age Across the Oceans 3. Tillite deposits 4. Placer Deposits Select the correct answer code: a) 1, 2, 3 b) 1, 3, 4 c) 2, 3, 4 d) 1, 2, 3, 4

d Evidence in Support of the Continental Drift

・ • The Matching of Continents (Jig-Saw-Fit): The shorelines of Africa and South America facing each other have a remarkable and unmistakable match. ・ • Rocks of Same Age Across the Oceans: The belt of ancient rocks of 2,000 million years from Brazil coast matches with those from western Africa. ・ • Tillite: It is the sedimentary rock formed out of deposits of glaciers. The Gondawana system of sediments from India is known to have its counter parts in six different landmasses of the Southern Hemisphere. Overall resemblance of the Gondawana type sediments clearly demonstrates that these landmasses had remarkably similar histories. ・ • Placer Deposits: The occurrence of rich placer deposits of gold in the Ghana coast and the absolute absence of source rock in the region is an amazing fact. The gold bearing veins are in Brazil and it is obvious that the gold deposits of the Ghana are derived from the Brazil plateau when the two continents lay side by side

Insights Dec-20 Geography 06/01/2021 1 14) Which of the following elements makes the highest share in the Earth’s crust by weight?

a) Iron b) Silicon c) Oxygen d) Aluminum

c The Major Elements of the Earth’s Crust:

Elements By Weight(%) 1. Oxygen 46.60 2. Silicon 27.72 3. Aluminum 8.13 4. Iron 5.00 5. Calcium 3.63 6. Sodium 2.83 7. Potassium 2.59 8. Magnesium 2.09 9. Others 1.41

Insights Dec-20 Geography 06/01/2021 1 15) The event of extinction of Dinosaurs was witnessed in which of the following period?

a) Jurassic b) Triassic c) Pleistocene d) Cretaceous

d
Insights Dec-20 Geography 06/01/2021 1 16) Consider the following statements regarding Shield volcanoes.

1. The shield volcanoes are the largest of all the volcanoes on the earth. 2. The Hawaiian volcanoes are the examples of Shield volcanoes. 3. These volcanoes are very steep in nature. Which of the above statements is/are correct? a) 1, 2 b) 1, 3 c) 2 only d) 2, 3

c Shield Volcanoes:

Barring the basalt flows, the shield volcanoes are the largest of all the volcanoes on the earth. The Hawaiian volcanoes are the most famous examples. These volcanoes are mostly made up of basalt, a type of lava that is very fluid when erupted. For this reason, these volcanoes are not steep. They become explosive if somehow water gets into the vent; otherwise, they are characterised by low-explosivity.

Insights Dec-20 Geography 06/01/2021 1 17) Consider the following statements.

1. In an Earthquake, greatest damage usually occurs closest to the epicentre and the strength of the earthquake decreases away from the centre. 2. The velocity of earthquake waves remains constant as they travel through materials with different densities. Which of the above statements is/are correct? a) 1 only b) 2 only c) Both 1 and 2 d) Neither 1 nor 2

a When the Lithospheric plates move, the surface of the earth vibrates. The vibrations can travel all-round the earth. These vibrations are called earthquakes. The place in the crust where the movement starts is called the focus. The place on the surface above the focus is called the epicentre. Vibrations travel outwards from the epicentre as waves. Greatest damage is usually closest to the epicentre and the strength of the earthquake decreases away from the centre.

The velocity of earthquake waves changes as they travel through materials with different densities. The denser the material, the higher is the velocity. Their direction also changes as they reflect or refract when coming across materials with different densities.

Insights Dec-20 Geography 06/01/2021 1 18) Which of the following are the reasons that can be attributed to the frequent landslides and debris avalanches in Himalayan region?

1. The Himalayas are tectonically stable. 2. They are mostly made up of sedimentary rocks and unconsolidated deposits. 3. The slopes are very steep. Select the correct answer code: a) 1, 2 b) 1, 3 c) 2, 3 d) 1, 2, 3

c The reasons that can be attributed to the frequent landslides and debris avalanches in Himalayan region are

(i) The Himalayas are tectonically active. Thus, tectonically unstable. (ii) They are mostly made up of sedimentary rocks and unconsolidated and semi-consolidated deposits. (iii) The slopes are very steep.

Insights Dec-20 Geography 06/01/2021 1 19) Consider the following statements regarding Earthquake Waves.

1. P-waves move faster than S-waves. 2. P-waves travel only through solid medium whereas S-waves travel through both solid and liquid medium. 3. The surface waves are more destructive than body waves. Which of the above statements is/are correct? a) 1, 2 b) 1, 3 c) 2, 3 d) 1, 2, 3

Earthquake waves are basically of two types — body waves and surface waves. Body waves are generated due to the release of energy at the focus and move in all directions travelling through the body of the earth.

There are two types of body waves. They are called P and S-waves. P-waves move faster and are the first to arrive at the surface. These are also called ‘primary waves’. The P-waves are similar to sound waves. They travel through gaseous, liquid and solid materials. S-waves arrive at the surface with some time lag. These are called secondary waves. An important fact about S-waves is that they can travel only through solid materials. The surface waves are the last to report on seismograph. These waves are more destructive. They cause displacement of rocks, and hence, the collapse of structures occurs

Insights Dec-20 Geography 06/01/2021 1 20) Consider the following statements regarding Earthquakes and Volcano.

1. Earthquake normally occurs along with the volcanoes. 2. Volcanoes occur only where ocean crust collides with continental crust. 3. Volcanoes are one of the best sources of sulphur. Which of the above statements is/are correct? a) 1, 3 b) 2, 3 c) 1, 2 d) 1, 2, 3

a Earth quakes are usually associated with volcanoes, since both are related to plate movements.

Volcanoes can also happen with ocean-ocean crust collision. Best example is South East Asian Archipelago. Volcanoes are one of the best sources of sulphur.

Insights Dec-20 Geography 06/01/2021 1 21) The climate and weather of a region largely depends on

1. Distance from equator 2. Height above Mean Sea Level (MSL) 3. Distance from International Date Line 4. Proximity with major water bodies Select the correct answer code: a) 1, 2, 3 b) 1, 3, 4 c) 1, 2, 4 d) 1, 4

c • Places farther from equator are likely to have lesser rainfall and a colder climate.

• Altitude affects the temperature of a place. Places at higher Mean Sea Level (MSL) are more likely to be colder than those at lower MSL. • Distance from International Date Line indirectly refers to the longitude of a place, which has no bearing on a place’s weather or climate. • Places close to large lakes or sea are more likely to have moderate weather due to the blowing of land and sea breezes.

Insights Dec-20 Geography 06/01/2021 1 22) In the course of its evolution, the early atmosphere on earth largely contained

1. Water vapour 2. Nitrogen 3. Carbon dioxide 4. Methane 5. Ammonia 6. Very little of free oxygen Select the correct answer code: a) 1, 2, 3, 4, 5 b) 1, 3, 4, 5, 6 c) 1, 2, 4, 5, 6 d) 1, 2, 3, 4, 5, 6

d During the cooling of the earth, gases and water vapour were released from the interior solid earth.

This started the evolution of the present atmosphere. The early atmosphere largely contained water vapour, nitrogen, carbon dioxide, methane, ammonia and very little of free oxygen.

Insights Dec-20 Geography 06/01/2021 1 23) Consider the following statements.

1. The location of the thermal equator is not identical to that of the geographic Equator. 2. The location of thermal equator is most affected by the level of insolation received in both hemispheres. Which of the above statements is/are correct? a) 1 only b) 2 only c) Both 1 and 2 d) Neither 1 nor 2

a The location of the thermal equator is not identical to that of the geographic Equator.

This is not due to the Level of insolation received in both hemispheres. They are virtually equal. What creates the difference is the distribution of land and water that causes changes in the way heat is distributed and transported across the planets. Land dominated areas will tend to have a large variation in temperature and a higher average temperature under the same conditions (as compared to a water dominated area). Thus, the thermal equator lies a bit north of the geographical equator.

Insights Dec-20 Geography 06/01/2021 1 24) Factors that affect temperature at a particular region include

1. Circulation of planetary and local winds 2. Distance of the region from poles or equator 3. Movement of Ocean waves 4. Altitude and terrain of the place Select the correct answer code: a) 1, 2, 3 b) 1, 2, 4 c) 1, 2, 3, 4 d) 2, 3, 4

b Water moves ahead from one place to another through ocean currents while the water in the waves does not move, but the wave trains move ahead. If water is not moved from one place to the other, there is no circulation of temperature, nutrients, water density or salinity.
Insights Dec-20 Geography 06/01/2021 1 25) Consider the following statements.

1. The mid-day sun never shines overhead on any latitude beyond the Tropic of Cancer and the Tropic of Capricorn. 2. The areas bounded by the Tropic of Cancer and the Arctic Circle in the Northern Hemisphere, and the Tropic of Capricorn and the Antarctic Circle in the Southern Hemisphere, have moderate temperatures. Which of the above statements is/are incorrect? a) 1 only b) 2 only c) Both 1 and 2 d) Neither 1 nor 2

d The mid-day sun never shines overhead on any latitude beyond the Tropic of Cancer and the Tropic of Capricorn. The angle of the sun’s rays goes on decreasing towards the poles. As such, the areas bounded by the Tropic of Cancer and the Arctic Circle in the Northern Hemisphere, and the Tropic of Capricorn and the Antarctic Circle in the Southern Hemisphere, have moderate temperatures. These are, therefore, called Temperate Zones.
Insights Dec-20 Geography 06/01/2021 1 26) Consider the following statements:

1. On summer solstice, the northern hemisphere will have its longest day and the shortest night. 2. On winter solstice, the southern hemisphere will have its longest day and the shortest night. Which of the above statements is/are correct? a) 1 only b) 2 only c) Both 1 and 2 d) Neither 1 nor 2

c The summer solstice, also known as midsummer, occurs when one of the Earth's poles has its maximum tilt toward the Sun. It happens twice yearly, once in each hemisphere. For that hemisphere, the summer solstice is when the Sun reaches its highest position in the sky and is the day with the longest period of daylight. On summer solstice, the northern hemisphere will have its longest day and the shortest night.

The winter solstice occurs when one of the Earth's poles has its maximum tilt away from the Sun. It happens twice yearly, once in each hemisphere. On winter solstice, the southern hemisphere will have its longest day and the shortest night.

Insights Dec-20 Geography 06/01/2021 1 27) Clear and sunny skies are usually associated with

a) High pressure zones b) Low pressure zones c) A zone of intense pressure fluctuations d) None of the above

a In areas having lower temperature, the air is cold. It is therefore heavy. Heavy air sinks and creates a high pressure area. High pressure is associated with clear and sunny skies.
Insights Dec-20 Geography 06/01/2021 1 28) Consider the following statements regarding Jet streams.

1. Jet streams are a narrow belt of high-altitude westerly winds in the troposphere. 2. These blow at a constant speed throughout the year. 3. They are limited to tropical latitudes with occasional movements in the sub-tropics. Which of the above statements is/are correct? a) 1, 2 b) 1, 3 c) 1 only d) 2, 3

c Jet streams are a narrow belt of high altitude (above 12,000 m) westerly winds in the troposphere.

Their speed varies from about 110 km/h in summer to about 184 km/h in winter. A number of separate jet streams have been identified. The most constant are the mid-latitude and the sub-tropical jet stream.

Insights Dec-20 Geography 06/01/2021 1 29) Consider the following statements.

1. Fold mountains are created when two plates collide, whereas Block Mountains are created when large areas are broken and displaced vertically. 2. The Himalayan Mountains are young fold mountains, whereas the Alps are block mountains. Which of the above statements is/are incorrect? a) 1 only b) 2 only c) Both 1 and 2 d) Neither 1 nor 2

b There are three types of mountains- Fold Mountains, Block Mountains and the Volcanic Mountains.

The Himalayan Mountains and the Alps are young fold mountains with rugged relief and high conical peaks. Fold mountains are created when two plates collide which is the case with Himalayas. The Aravali range in India is one of the oldest fold mountain systems in the world. Block Mountains are created when large areas are broken and displaced vertically. This didn’t happen with Himalayas.

Insights Dec-20 Geography 06/01/2021 1 30) Consider the following statements with reference to the ‘Continental Drift Theory’ proposed by Alfred Wegener.

1. In the beginning, all the continents formed a single continental mass called Laurasia. 2. The movement responsible for the drifting of the continents was caused by pole-fleeing force and tidal force. Which of the above statements is/are correct? a) 1 only b) 2 only c) Both 1 and 2 d) Neither 1 nor 2

b According to Wegener, all the continents formed a single continental mass and mega ocean surrounded the same. The super continent was named PANGAEA, which meant all earth. The mega-ocean was called PANTHALASSA, meaning all water. He argued that, around 200 million years ago, the super continent, Pangaea, began to split. Pangaea first broke into two large continental masses as Laurasia and Gondwanaland forming the northern and southern components respectively. Hence, statement 1 is incorrect.

Wegener suggested that the movement responsible for the drifting of the continents was caused by pole-fleeing force and tidal force. The polar-fleeing force relates to the rotation of the earth and the tidal force is due to the attraction of the moon and the sun that develops tides in oceanic waters.

Insights Dec-20 Geography 06/01/2021 1 31) Western Cyclonic Disturbances

1. Occur mainly in the winter months in India 2. Mostly Influence the north and north-western regions of India 3. Causes tropical cyclones in the coastal belts of India Select the correct answer code: a) 1, 2 b) 1, 3 c) 2, 3 d) 1, 2, 3

a The western cyclonic disturbances are weather phenomena of the winter months brought in by the westerly flow from the Mediterranean region. They usually influence the weather of the north and north-western regions of India.

Tropical cyclones occur during the monsoon as well as in October -November, and are part of the easterly flow.

Insights Dec-20 Geography 06/01/2021 1 32) The Roaring Forties are

a) Feeble easterly winds found in the Northern Hemisphere b) Strong westerly winds found in the Southern Hemisphere c) Equatorial counter-currents that move near to the eastern coastal shores in lower latitudes d) Thunderstorms often found near the equator

b The Roaring Forties are strong westerly winds found in the Southern Hemisphere, generally between the latitudes of 40 and 50 degrees. The strong west-to-east air currents are caused by the combination of air being displaced from the Equator towards the South Pole, the Earth's rotation, and the scarcity of landmasses to serve as windbreaks.

The Roaring Forties were a major aid to ships sailing the Brouwer Route from Europe to the East Indies or Australasia during the Age of Sail, and in modern usage are favoured by yachtsmen on round-the-world voyages and competitions. The boundaries of the Roaring Forties are not consistent, and shift north or south depending on the season. Similar but stronger conditions occurring in more southerly latitudes are referred to as the Furious Fifties and the Shrieking or Screaming Sixties.

Insights Dec-20 Geography 06/01/2021 1 33) Monsoon winds can be best described as

a) Local winds b) Easterly winds c) Seasonal winds d) Permanent winds

c Winds can be broadly divided into three types.

1. Permanent winds – The trade winds, westerlies and easterlies are the permanent winds. These blow constantly throughout the year in a particular direction. 2. Seasonal winds – These winds change their direction in different seasons. For example monsoons in India. 3. Local winds – These blow only during a particular period of the day or year in a small area. For example, land and sea breeze.

Insights Dec-20 Geography 06/01/2021 1 34) The major prevailing wind type resulting in hot desert landscape is

a) South-westerly winds throughout the year b) Light and variable winds in summer c) Westerlies that blow throughout the year d) North-east trade wind throughout the year

d
Insights Dec-20 Geography 06/01/2021 1 35) Consider the following statements.

1. Low pressure systems are usually characterized by dry and settled weather. 2. High pressure systems usually bring severe wind and precipitation. Which of the above statements is/are incorrect? a) 1 only b) 2 only c) Both 1 and 2 d) Neither 1 nor 2

c A low-pressure area is commonly associated with inclement weather, while a high-pressure area is associated with light winds and fair skies.
Insights Dec-20 Geography 06/01/2021 1 36) Consider the following statements.

1. Most of the cyclones that hit India originate in the Bay of Bengal, rather than the Arabian Sea. 2. Arabian Sea cyclones are also relatively weak compared to those emerging in the Bay of Bengal. Which of the above statements is/are correct? a) 1 only b) 2 only c) Both 1 and 2 d) Neither 1 nor 2

c Though cyclones are common in the month of June, very few of them originate in the Arabian Sea. Most of them are found in the Bay of Bengal. In the last 120 years for which records are available, just about 14% of all cyclonic storms, and 23% of severe cyclones, around India have occurred in the Arabian Sea. Arabian Sea cyclones are also relatively weak compared to those emerging in the Bay of Bengal.
Insights Dec-20 Geography 06/01/2021 1 37) A tropical cyclone is classified as making landfall when

1. The centre of the storm moves across the coast 2. The heat from condensation of vapour nuclei reaches its peak levels. Select the correct answer code: a) 1 only b) 2 only c) Both 1 and 2 d) Neither 1 nor 2

a A Landfall is the intersection of the center of a tropical cyclone with a coastline. Typically, in strong tropical cyclones, a landfall occurs when the eye of the cyclone moves over land.

When the cyclone makes landfall, it starts to lose speed and energy as it loses its source of both from the warm ocean waters. Therefore, at this point the condensation of the nuclei will drop, and not reach peak levels.

Insights Dec-20 Geography 06/01/2021 1 38) Places in the mountains are usually cooler than places on the plains due to the presence of

a) Moisture-laden winds on the mountains b) Leeward wind flow on the mountains c) Thin air on the mountains d) All of the above

c Temperature decreases with height. Due to thin air, places in the mountains are cooler than places on the plains. For example, Agra and Darjiling are located on the same latitude, but temperature of January in Agra is 16°C whereas it is only 4°C in Darjiling.
Insights Dec-20 Geography 06/01/2021 1 39) As per the Koeppen classification, which of the following regions of India has a Steppe Climate?

a) North-eastern India b) Parts of North-western India and leeward side of Western Ghats c) Coastal regions of south-eastern India d) Central India

b
Insights Dec-20 Geography 06/01/2021 1 40) Why are the western slopes of the Western Ghats covered with thick forests and not the eastern slopes?

1. Soil in eastern slope cannot support vegetation due to absence of nutrients. 2. Eastern side falls in rain shadow zone 3. No protected forests exist on the eastern slopes. Select the correct answer code: a) 1, 2 b) 2 only c) 1, 3 d) 2, 3

b When moisture laden winds cause rainfall in western slopes, upon moving to eastern sides they lose much of their moisture. Rainfall is less, and thus vegetation is less thick at eastern slopes.

Nilgiri Biosphere Reserve extends to Eastern Slopes.

Insights Dec-20 Geography 06/01/2021 1 41) The top layer of warm oceanic water is present throughout the year in

a) Tropical region b) Poles c) Temperate regions d) Southern Ocean

a The temperature structure of oceans over middle and low latitudes can be described as a three-layer system from surface to the bottom. The first layer represents the top layer of warm oceanic water and it is about 500m thick with temperatures ranging between 20° and 25° C. This layer, within the tropical region, is present throughout the year but in mid latitudes it develops only during summer.
Insights Dec-20 Geography 06/01/2021 1 42) Consider the following statements.

1. More fish come closer to the shore during the Low tide. 2. High tides help in navigation as they raise the water level close to the shores. Which of the above statements is/are correct? a) 1 only b) 2 only c) Both 1 and 2 d) Neither 1 nor 2

b High tides help in navigation. They raise the water level close to the shores. This helps the ships to arrive at the harbour more easily. The high tides also help in fishing. Many more fish come closer to the shore during the high tide. This enables fishermen to get a plentiful catch. The rise and fall of water due to tides is being used to generate electricity in some places.
Insights Dec-20 Geography 06/01/2021 1 43) Seas around Japan and the eastern coast of North America have some of the World’s best fishing grounds because

a) These areas do not experience strong ocean currents or strong ocean waves. b) They are not hit by tsunamis or hurricanes. c) These are the areas where the warm and cold ocean currents meet. d) These areas have high influx of fresh water from the Rivers.

The areas where the warm and cold currents meet provide the best fishing grounds of the world. Seas around Japan and the eastern coast of North America are such examples. The areas where a warm and cold current meet also experience foggy weather making it difficult for navigation.
Insights Dec-20 Geography 06/01/2021 1 44) Consider the following statements.

1. More fresh water can be found in the atmosphere than in the rivers on earth. 2. Rivers on earth contain less fresh water than Inland seas and salt lakes. 3. Ice-caps contain less fresh water than the fresh water available in the groundwater. Which of the above statements is/are correct? a) 1, 2 b) 1, 3 c) 1 only d) 2, 3

a
Insights Dec-20 Geography 06/01/2021 1 45) In deeper parts of oceans, primary production remains almost nil due to the absence of

a) Upwelling of water b) Excess of sulphides c) Ocean currents d) Sunlight

d The oceans below the photic zone are a vast space, largely dependent on food from photosynthetic primary producers living in the sunlit waters above.

This is because phytoplanktons (primary production) require abundant sunlight and nutrients to grow well.

Insights Dec-20 Geography 06/01/2021 1 46) Consider the following statements regarding Chota Nagpur Plateau.

1. The Chota Nagpur Plateau is a plateau in eastern India, which covers much of Jharkhand state as well as adjacent parts of Odisha, West Bengal and Chhattisgarh. 2. The Indo-Gangetic plain lies to the north and east of the plateau. 3. The plateau has been formed by continental uplift from forces acting deep inside the earth. 4. The Krishna River flows through Chota Nagpur Plateau. Which of the above statements is/are correct? a) 1, 2, 3 b) 1, 3, 4 c) 2, 3, 4 d) 1, 2, 3, 4

a The Chota Nagpur Plateau is a plateau in eastern India, which covers much of Jharkhand state as well as adjacent parts of Odisha, West Bengal and Chhattisgarh. The Indo-Gangetic plain lies to the north and east of the plateau, and the basin of the Mahanadi River lies to the south.

The Chota Nagpur Plateau is a continental plateau—an extensive area of land thrust above the general land. The plateau has been formed by continental uplift from forces acting deep inside the earth. The Damodar River flows through Chota Nagpur Plateau.

Insights Dec-20 Geography 06/01/2021 1 47) Which of these peaks of Himalayas, are situated in India?

1. Dhaulagiri 2. Kanchenjunga 3. Namcha Barwa 4. Nanda Devi 5. Annapurna Select the correct answer code: a) 1, 3, 5 b) 1, 2, 3, 4 c) 2, 4 d) 2, 3, 4, 5

c
Insights Dec-20 Geography 06/01/2021 1 48) Which of the following fall under the Peninsular block of India?

1. Aravali Range 2. Rajmahal Hills 3. Karbi Anglong Plateau 4. Meghalaya Plateau Select the correct answer code: a) 1, 2, 3 b) 1, 3, 4 c) 2, 3, 4 d) 1, 2, 3, 4

d The northern boundary of the Peninsular Block may be taken as an irregular line running from Kachchh along the western flank of the Aravali Range near Delhi and then roughly parallel to the Yamuna and the Ganga as far as the Rajmahal Hills and the Ganga delta. Apart from these, the Karbi Anglong and the Meghalaya Plateau in the northeast and Rajasthan in the west are also extensions of this block. The northeastern parts are separated by the Malda fault in West Bengal from the Chotanagpur plateau. In Rajasthan, the desert and other desert–like features overlay this block
Insights Dec-20 Geography 06/01/2021 1 49) Consider the following statements regarding Indian plate.

1. The Indian plate is tectonically separated from the Peninsular India plate. 2. It forms a convergent plate boundary with the Himalayas. 3. The plate extends to Pakistan and Myanmar as well. Which of the above statements is/are correct? a) 1, 2 b) 1, 3 c) 2, 3 d) 1, 2, 3

c The Indian plate includes Peninsular India and the Australian continental portions. The subduction zone along the Himalayas forms the northern plate boundary in the form of continent— continent convergence. In the east, it extends through Rakinyoma Mountains of Myanmar towards the island arc along the Java Trench. The eastern margin is a spreading site lying to the east of Australia in the form of an oceanic ridge in SW Pacific. The Western margin follows Kirthar Mountain of Pakistan.
Insights Dec-20 Geography 06/01/2021 1 50) Consider the following statements.

1. Bhur denotes an elevated piece of land situated along the banks of the Ganga river 2. Bhangar and Khadar are old and new alluvium soils respectively. Which of the above statements is/are incorrect? a) 1 only b) 2 only c) Both 1 and 2 d) Neither 1 nor 2

d ・ • The Bhangar is the older alluvium along the river beds forming terraces higher than the flood plain.

・ • The terraces are often impregnated with calcareous concretions known as ‘KANKAR’. ・ • ‘The Barind plains’ in the deltaic region of Bengal and the ‘bhur formations’ in the middle Ganga and Yamuna doab are regional variations of Bhangar. ・ • [Bhur denotes an elevated piece of land situated along the banks of the Ganga river especially in the upper Ganga-Yamuna Doab. This has been formed due to accumulation of wind-blown sands during the hot dry months of the year] ・ • The Khadar is composed of newer alluvium and forms the flood plains along the river banks.

Insights Dec-20 Geography 06/01/2021 1 51) Consider the following statements regarding Peninsular Rivers.

1. The Peninsular rivers have shorter and shallower courses as compared to their Himalayan counterparts. 2. A large number of the Peninsular rivers are seasonal. 3. Peninsular rivers carry much larger silt as compared to Himalayan rivers which carry fresh water. Which of the above statements is/are correct? a) 1, 2 b) 1, 3 c) 2, 3 d) 2 only

a The Peninsular rivers have shorter and shallower courses as compared to their Himalayan counterparts. Most of them come from Western Ghats, however, some of them originate in the central highlands and flow towards the west. Rest of them all flow to the east in the Bay of Bengal.

A large number of the Peninsular rivers are seasonal, as their flow is dependent on rainfall. During the dry season, even the large rivers have reduced flow of water in their channels. Moreover, due to flowing on hard terrain and short courses, they carry much lesser silt as compared to Himalayan rivers

Insights Dec-20 Geography 06/01/2021 1 52) Consider the following statements.

1. Braided channels occur in rivers with near flat slopes. 2. V-shaped valleys are a feature of youthful stage of Himalayan rivers. Which of the above statements is/are correct? a) 1 only b) 2 only c) Both 1 and 2 d) Neither 1 nor 2

b Braided streams tend to occur in rivers with high sediment loads and/or coarse grain sizes, and in rivers with steeper slopes than typical rivers with straight or meandering channel patterns.

The Himalayas mountains are tectonic in origin, dissected by fast-flowing rivers which are in their youthful stage. Various landforms like gorges, V-shaped valleys, rapids, waterfalls, etc. are indicative of this stage

Insights Dec-20 Geography 06/01/2021 1 53) Consider the following matches of rivers with their origin locations.

1. Sabarmati: Amarkantak 2. Brahmaputra: Kailash Range 3. Krishna: Mahabaleshwar 4. Godavari: Trimbakeshwar Which of the above is/are correctly matched? a) 1, 2, 3 b) 1, 3, 4 c) 2, 3, 4 d) 1, 2, 3, 4

c The Sabarmati river is one of the major west-flowing rivers in India. It originates in the Aravalli Range.

Brahmaputra river originates from the Kailash ranges of Himalayas. Krishna river's source is at Mahabaleshwar. The Godavari River rises from Trimbakeshwar in the Nashik district of Maharashtra.

Insights Dec-20 Geography 06/01/2021 1 54) Which of the following rivers drain into Bay of Bengal?

1. Mahi 2. Penner 3. Subarnrekha 4. Tapti Select the correct answer code: a) 1, 4 b) 2, 3 c) 1, 2, 3 d) 3, 4

b Usually, large Rivers flowing on the peninsular plateau have their origin in the Western Ghats and discharge their waters in the Bay of Bengal.

Indus, Tapti, Mahi, Narmada and Sabarmati drain in the Arabian Sea. Subarnrekha, Penner and others drain in the Bay of Bengal.

Insights Dec-20 Geography 06/01/2021 1 55) It is opined that in due course of time Indo– Brahma river was dismembered into many drainage systems. These include

1. Indus and its tributaries 2. Ganga and its Himalayan tributaries 3. The stretch of the Brahmaputra in Assam and its Himalayan tributaries Select the correct answer code: a) 1, 2 b) 1, 3 c) 2, 3 d) 1, 2, 3

d It is opined that in due course of time Indo– Brahma river was dismembered into three main drainage systems: (i) the Indus and its five tributaries in the western part; (ii) the Ganga and its Himalayan tributaries in the central part; and (iii) the stretch of the Brahmaputra in Assam and its Himalayan tributaries in the eastern part.
Insights Dec-20 Geography 06/01/2021 1 56) Which of the following have an influence on the movement of ocean water?

1. Salinity 2. Temperature 3. Sun 4. Moon 5. Wind Select the correct answer code: a) 2, 3, 4, 5 b) 1, 2, 4, 5 c) 1, 2, 5 d) 1, 2, 3, 4, 5

d The ocean water is dynamic. Its physical characteristics like temperature, salinity, density and the external forces like of the sun, moon and the winds influence the movement of ocean water